Human Phys. Lab - Renal Physiology and Acid-Base Balance, Physiology Chapter 21, Physiology Chapter 20, Physiology Chapter 19, Ch. 23 Final review, Urinary System, Human Physiology: An Integrated Approach, 6e (Silverthorn) Chapter 22 Metabolism and E...

Lakukan tugas rumah & ujian kamu dengan baik sekarang menggunakan Quizwiz!

If 140 liters of plasma are filtered in a day, and the filtration fraction is 20%: A. What is the renal plasma flow? B. If this person has a hematocrit of 30%[p. 517], what is the renal blood flow? C. If renal blood flow is 20% of this person's cardiac output, what is her cardiac output in L/min?

(a) 140 L/day is 20% of renal plasma flow (RBF), so plasma flow is 700 L/day. (b) Hematocrit is percent of blood occupied by packed red blood cells; the remainder (70%) is plasma. 700 L/day is 70% of RBF, so RBF is 1000 L/day. (c) If RBF is 20% of cardiac output (CO), then CO=5000 L/day or 3.47 L/min.

Match each of the following substances with its mode(s) of transport in proximal tubule reabsorption. A.Na+ B. glucose C. urea D. plasma proteins E. water 1. simple diffusion 2. primary active transport 3. indirect active transport 4. facilitated diffusion 5. movement through open channels 6. endocytosis 7. paracellular movement

(a) 2, 3, 5 (b) 3, 4 (c) 4, 7 (d) 6 (e) 5, 7

Match each of the following descriptions with the appropriate term(s): A. chyme is produced here B. organ where most digestion occurs C. initial section of small intestine D. this adds exocrine secretions to duodenum via a duct E. sphincter between stomach and intestine F. enzymes produced here G. distension of its walls triggers the defecation reflex 1. colon 2. stomach 3. small intestine 4. duodenum 5. ileum 6. jejunum 7. pancreas 8. pylorus 9. rectum 10. liver

(a) 2; (b) 3; (c) 4; (d) 7, 10; (e) 8; (f) 2, 3, 7; (g) 9

Hyperglycemia in a diabetic patient leads to osmotic diuresis and dehydration. Given the following information, answer the questions. Plasma glucose=400 mg/dL Normal urine flow=1 L per day GFR=130 mL/min Normal urine osmolarity=300 mOsM Glucose Tm=400 mg/min Molecular mass of glucose=180 daltons Renal plasma flow=500 mL/min How many milligrams of glucose filter into the nephron each minute? How many milligrams of glucose are reabsorbed each minute? How many grams of glucose are excreted in the urine each day? Assuming that dehydration causes maximal vasopressin secretion and allows the urine to concentrate to 1200 mOsM, how much additional urine does this diabetic patient excrete in a day?

(a) 400 mg glucose/100 mL×130 mL/min=520 mg glucose/ min filters. (b) Can reabsorb up to Tm so 400 mg/min reabsorbed. (c) Excreted=filtered−reabsorbed=120 mg/min×1440 min/ day=172.8 g/day excreted. (d) Convert grams to milliosmoles: 172.8 g×mole/180 g ×1000 mosmol/mole=960 mosmol glucose excreted/day. Concentration=amount/volume. 1200 mosmol/ L=960 mosmol/? liters. Will require 0.8 L additional volume.

Match each of the following cells with the product(s) it secretes. Items may be used more than once. A. parietal cells B. goblet cells C. brush border cells D. pancreatic cells E. D cells F. ECL cells G. chief cells H. G cells 1. enzymes 2. histamine 3. mucus 4. pepsinogen 5. gastrin 6. somatostatin 7. HCO3−HCO3− 8. HCl 9. intrinsic factor

(a) 8, 9; (b) 3; (c) 1, 3, 7; (d) 1, 7; (e) 6; (f) 2; (g) 4; (h) 5

A 45-year-old man visiting from out of town arrives at the emergency room having an asthma attack caused by pollen. (a) Blood drawn before treatment showed the following: HCO3 - = 30 meq>L (normal: 24), PCO2 = 70 mm Hg, pH = 7.24. What is the man's acid-base state? Is this an acute or a chronic situation? (b) Th e man was treated and made a complete recovery. Over the next ten years he continued to smoke a pack of cigarettes a day, and a year ago his family doctor diagnosed chronic obstructive pulmonary disease (emphysema). Th e man's most recent blood test showed the following: HCO3 - = 45 meq>L, PCO2 = 85 mm Hg, pH = 7.34. What is the man's acid-base state now? Is this an acute or a chronic situation? (c) Explain why in his second illness his plasma bicarbonate level and PCO2 are higher than in the fi rst illness but his pH is closer to normal.

(a) Acute respiratory acidosis (b) Chronic respiratory acidosis (c) Renal compensation has increased his pH by H+ excretion and HCO3 - reabsorption. His PCO2 is elevated because of his emphysema.

Define the following terms and explain their physiological significance: A.lipoprotein lipase B.amylin C.ghrelin D.neuropeptide Y E.apoprotein F.leptin G.osmotic diuresis H.insulin resistance

(a) Capillary endothelium enzyme that converts triglycerides into free fatty acids and monoglycerides. (b) Co-secreted with insulin; slows gastric emptying and gastric acid secretion. (c) "Hunger hormone" secreted by the stomach. (d) Hypothalamic peptide that increases food intake. (e) Protein components of lipoproteins. Apoprotein B on LDL-C facilitates transport into most cells. (f) "Satiety hormone" produced by adipocytes. (g) Loss of water in the urine due to high amounts of urine solutes. Hyperglycemia causes dehydration through osmotic diuresis. (h) Target cells fail to respond normally to insulin.

Liddle's syndrome is an inherited defect of apical ENaC sodium channels in P cells. It is characterized by high blood pressure and hypokalemia. A. Are the defective ENaC channel proteins increasing or decreasing apical Na+ movement? Explain and relate your answer to the characteristic hypokalemia. B. Liddle's syndrome is considered a form of pseudohyperaldosteronism {pseudo-, false}. What test(s) would you run to distinguish Liddle's syndrome from primary or secondary hyperaldosteronism?

(a) ENaC activity is increased, bringing more Na+ into the cell. This increases activity of the Na+-K+-ATPase, which pumps K+ into the cell. The K+ then leaks out the apical channels into the urine, resulting in increased excretion of K+ and hypokalemia. (a) Run aldosterone and ANG II levels. They will both be normal in Liddle's syndrome.

Define, compare, and contrast the items in the following sets of terms: A. filtration, secretion, and excretion B. saturation, transport maximum, and renal threshold C. probenecid, creatinine, inulin, and penicillin D. clearance, excretion, and glomerular filtration rate

(a) Filtration and secretion both move material from blood to tubule lumen, but filtration is a bulk flow process while secretion is a selective process. Excretion is also bulk flow but involves movement from the kidney lumen to the outside world. (b) Saturation—all transporter binding sites are occupied by ligand. Transport maximum—the maximum rate at which carriers are saturated by substrate. Renal threshold—plasma concentration at which saturation occurs. (c) Creatinine and inulin—compounds used to determine GFR. Penicillin and probenecid—xenobiotics that are secreted. (d) Clearance—rate at which plasma is cleared of a substance (mL plasma cleared of substance X/min). GFR—filtration rate of plasma (mL plasma filtered/min). Excretion—removal of urine, mL urine/min.

Identify the following structures, then explain their significance in renal physiology: A. juxtaglomerular apparatus B. macula densa C. mesangial cells D. podocytes E. sphincters in the bladder F. renal cortex

(a) Found where distal tubule passes between afferent and efferent arterioles. Composed of macula densa cells in the distal tubule and granular cells in arteriole wall. (b) Macula densa paracrine signals control autoregulation of GFR and renin secretion. (c) Alter the size of filtration slits. (d) Specialized epithelial cells that surround glomerular capillaries. Changes in slit size alter GFR. (e) An internal smooth muscle sphincter that is passively contracted and an external skeletal muscle sphincter that is tonically (actively) contracted. (f) Outer layer of the kidney that contains renal corpuscles, proximal and distal tubules, and parts of the loop of Henle and collecting ducts.

Define, compare, and contrast or relate the terms in each of the following sets: A. glucose, glycogenolysis, glycogenesis, gluconeogenesis, glucagon, glycolysis B. shivering thermogenesis, nonshivering thermogenesis, diet-induced thermogenesis C. lipoproteins, chylomicrons, cholesterol, HDL-C, LDL-C, apoproteins D. direct and indirect calorimetry E. conductive heat loss, radiant heat loss, convective heat loss, evaporative heat loss F. absorptive and postabsorptive states

(a) Glucose—monosaccharide. Glycogenolysis—glycogen breakdown. Glycogenesis—glycogen production from glucose. Gluconeogenesis—glucose synthesis from amino acids and fats. Glucagon—hormone that increases plasma glucose. Glycolysis—first pathway in glucose metabolism for ATP production. (b) Thermogenesis—heat production by cells. Shivering thermogenesis—muscle twitches produce heat as a by-product. Nonshivering thermogenesis occurs in all cells. Diet-induced thermogenesis—heat generated by digestive and anabolic reactions during the absorptive state. (c) Lipoproteins—transport molecules. Chylomicrons—lipoprotein complexes assembled in intestinal epithelium and absorbed into lymphatic system. Cholesterol—steroid component of cell membranes and precursor to steroid hormones. HDL-C—takes cholesterol into liver cells, where it is metabolized or excreted. LDL-C—elevated concentrations are associated with atherosclerosis. Apoprotein—protein component of lipoproteins. (d) Calorimetry—measurement of energy content and a means of determining metabolic rate. Direct calorimetry—measuring heat production when food is burned. Indirect calorimetry—measures oxygen consumption or CO2CO2 production. (e) Conductive heat loss—loss of body heat to a cooler object. Radiant heat loss—loss from production of infrared electromagnetic waves. Convective heat loss—upward movement of warm air and its replacement by cooler air. Evaporative heat loss—heat lost when water evaporates. (f) Absorptive state—following a meal, when anabolism exceeds catabolism. Postabsorptive state—catabolism exceeds anabolism.

Describe (or map) the physiological events that lead to the following signs or symptoms in a type 1 diabetic: A. hyperglycemia B. glucosuria C. polyuria D. ketosis E. dehydration F. severe thirst

(a) Hyperglycemia results from the lack of insulin production and failure of cells to take up and use glucose. (b) Glucosuria results when filtered glucose exceeds the kidney's capacity to reabsorb it. (c) Polyuria results from osmotic diuresis caused by glucosuria. (d) Ketosis results from increased fatty acid metabolism. (e) Dehydration is a consequence of polyuria due to osmotic diuresis. (f) Severe thirst is a consequence of dehydration.

How does each of the following factors affect digestion? Briefly explain how and where each factor exerts its effects. A. emulsification B. neural activity C. low pH D. size of food particles

(a) Increases surface area for enzymes to work; stomach and small intestine. (b) Motility and secretion along the length of the digestive tract. (c) Acidic pH in stomach helps break down food and digest microorganisms. Must be neutralized in the small intestine. (d) Size determines the surface area upon which enzymes can act.

Mary Littlefeather arrives in her physician's office complaining of severe, steady pain in the upper right quadrant of her abdomen. The pain began shortly after she ate a meal of fried chicken, French fries, and peas. Lab tests and an ultrasound reveal the presence of gallstones in the common bile duct running from the liver, gallbladder, and pancreas into the small intestine. A. Why was Mary's pain precipitated by the meal she ate? B. Which of the following processes will be affected by the gallstones: micelle formation in the intestine, carbohydrate digestion in the intestine, and protein absorption in the intestine. Explain your reasoning.

(a) Ingestion of a fatty meal triggers contraction of the gallbladder to release bile salts, but the blocked bile duct prevented bile secretion, causing pain. (b) Micelle formation—decreased due to lack of bile salts. Carbohydrate digestion—decreased because pancreatic secretions with amylase not able to pass blockage. Protein absorption—decreased slightly because of low pancreatic secretion; however, brush border enzymes also digest protein, so digestion does not stop completely when the bile duct is blocked. Therefore, some digested proteins will be absorbed.

You have been asked to study kidney function in a new species of rodent found in the Amazonian jungle. You isolate some nephrons and expose them to inulin. The following graph shows the results of your studies. (a) How is the rodent nephron handling inulin? Is inulin filtered? Is it excreted? Is there net inulin reabsorption? Is there net secretion? (b) On the graph, accurately draw a line indicating the net reabsorption or secretion. (Hint: excretion=filtration−reabsorption+secretion

(a) Inulin is filtered, secreted, and excreted. No evidence for reabsorption is presented. (b) The line indicating net secretion will be close to the filtration line until the slope changes, after which the secretion line is horizontal (no further increase in rate due to saturation).

Define, compare, and contrast the following pairs or sets of terms: A. mastication, deglutition B. microvilli, villi C. peristalsis, segmental contractions, migrating motor complex, mass movements D. chyme, feces E. short reflexes, long reflexes F. submucosal plexus, myenteric plexus, vagus nerve G. cephalic, gastric, and intestinal phases of digestion

(a) Mastication—chewing; deglutition—swallowing. (b) Villi—folds of intestine; microvilli—folds of cell membrane. Both increase surface area. (c) All patterns of GI muscle contraction. Migrating motor complex—move material from stomach to large intestine between meals. Peristalsis—progressive waves of contraction. Segmental contraction—contraction and relaxation of short intestinal segments. Mass movements—push material into rectum, triggering defecation. (d) Chyme—semidigested food and secretions, produced in the stomach. Feces—solid waste material that remains after digestion and absorption are complete; produced in the large intestine. (e) Short reflexes—integrated within the ENS. Long reflexes—integrated within the CNS. (f) Submucosal plexus—ENS in the submucosal layer. Myenteric plexus—ENS that lies between muscle layers of the GI tract wall. Vagus nerve—carries sensory and efferent signals between the brain and ENS. (g) Cephalic phase—digestive reflexes triggered by stimuli received in the brain. Gastric phase—short reflexes that begin with food entering the stomach. Intestinal phase—begins when chyme enters the small intestine.

Karen has bulimia, in which she induces vomiting to avoid weight gain. When the doctor sees her, her weight is 89 lb and her respiration rate is 6 breaths>min (normal 12). Her blood HCO3 - is 62 meq>L (normal: 24-29), arterial blood pH is 7.61, and PCO2 is 61 mm Hg. (a) What is her acid-base condition called? (b) Explain why her plasma bicarbonate level is so high. (c) Why is she hypoventilating? What eff ect does this have on the pH and total oxygen content of her blood? Explain your answers.

(a) Metabolic alkalosis, partially compensated. (b) After vomiting acid (H+),(H+), her body was left with HCO −3.HCO3 −. (c) Hypoventilation increases PCO2, HCO −3,PCO2, HCO3 −, and H+.H+. Increased H+H+ decreases pH (compensation). Hypoventilation also decreases arterial PO2PO2 and decreases the total oxygen content of blood

The Henderson-Hasselbalch equation is a mathematical expression of the relationship between pH, HCO −3 concentration, and dissolved CO2 concentration. One variant of the equation uses PCO2 instead of dissolved CO2 concentration: pH=6.1+log [HCO −3]/0.03×PCO2 A. If arterial blood has a PCO2 of 40 mm Hg and its HCO −3 concentration is 24 mM, what is its pH? (Use a log table or calculator with a logarithmic function capability.) B. What is the pH of venous blood with the same HCO −3 concentration but a PCO2 of 46 mm Hg?

(a) pH=6.1+log [24/(0.03×40)]=7.40pH=6.1+log [24/(0.03×40)]=7.40 (b) 7.34

What effect does insulin have on: A. glycolysis B. gluconeogenesis C. glycogenesis D. lipogenesis E. protein synthesis

(a) stimulates, (b) inhibits, (c) stimulates, (d) stimulates, (e) stimulates

Arrange the following structures in the order that a drop of water entering the nephron would encounter them: A. afferent arteriole B. Bowman's capsule C. collecting duct D. distal tubule E. glomerulus F. loop of Henle G. proximal tubule H. renal pelvis

(a), (e), (b), (g), (f), (d), (c), (h)

The following pressures exist at a glomerulus: PGC = 56 mmHg PBC = 14 mmHg πBC = 0 mmHg πGC = 32 mmHg What is the net glomerular filtration pressure?

+10 mmHg

Bowman's capsule

- a "globe-like" structure - jutting into it is a capillary network called the Glomerulus.

trigone of the bladder

- a triangular area of the posterior-inferior portion of the bladder wall - smooth walled as opposed to the rest of the mucous lining which appears to have many ridges & be very rough -outlined by 3 points - the entrance of ureter & exit point of the urethra.

Female urethra

- about 1 1/2 inches (short) - opens between clitoris & vagina opening within the vulva

Urinary Bladder

- hollow flask-shaped muscular organ - located anteriorly in the pelvic cavity - male: directly anterior to the rectum - female: directly anterior to the vagina

Kidneys

- reddish colored - kidney bean shaped - lies above the waist at the base of the posterior rib cage (about 1/2 covered by ribs) - lies posterior to peritoneum called "retroperitoneal" - about 5 inches & 2-3 inches wide - surrounding by 3 layers of connective tissue. - the concave portion of kidney-bean shape faces medially (toward aorta) called hilum. <- the area where blood vessels, lymph vessels, and nerve enter & exit.

juxtaglomerular cells

- secretory cells - the middle layer of the afferent arterioles - located adjacent to the macula dense

male urethra

- the average is 8 inches (longer) - first pass through the prostate gland, urogenital diaphragm, penis. - they serve to transport semen during ejaculation in addition to its purpose of transporting urine during micturition.

Macula Densa

- the distal convoluted tubule is in close proximity to the renal corpuscle. - cells become columnar - appear to have secretory characteristics

Nephron

- the functional unit of the kidney. - each kidney is composed of 1-4 million nephrons

Urethra

- the muscular tube leading from bladder to exterior of the body - transport urine out of the body - epithelium: area of pseudostratified columnar & stratified squamous - muscular contain smooth muscle

Renal clearance is something that you can quantify if you know certain variables associated with the substance in question. Renal clearance is helpful in determining the glomerular filtration rate. The glomerular filtration rate will tell you the volume of blood that is filtered per unit time and will allow you to infer the efficiency of the kidney and its ability to properly filter the appropriate volume of blood on a daily basis. Renal clearance will also help you to determine how well the kidney processes a particular substance, which is especially important in the area of pharmaceuticals. Which of the following variables must be known in order to calculate the clearance of a substance?

-The excretion rate of the substance -The concentration of solute in the plasma

Ureters

-renal pelvis connecting the kidney & bladder -transmits urine via peristalsis -lined by transitional epithelium -surrounding by a muscular layer composed of an inner circular & outer longitudinal layer of smooth muscle -the outermost fibrous coat of the ureter is connective tissue -ureter is retroperitoneal

Diagram the sodium-linked secondary active transport of glucose across the proximal tubule epithelium. (Fig. 19.8c)

...

Drag the labels onto the diagram to identify the processes of reabsorption in the tubular epithelium.

...

Given, Usodium = 10 mmole/L, Psodium = 140 mmole/L, V = 1.4 ml/min, what is the clearance of sodium?

0.1 ml/min ********FURTHER EXPLANATION OF THE ANSWER ****** Given, Usodium = 10 mmole/L, Psodium = 140 mmole/L, V = 1.4 ml/min, the clearance = 0.1 ml/min. Ux = urine concentration of a substance Px = plasma concentration of a substance V = rate of urine flow The formula for calculating clearance is: Clearance = (Usodium) X (V) (Psodium) Substituting in the appropriate numbers, we get, Clearance = (10 mmole/L) X (0.0014 L/min) = 0.0001 L/min or 0.1 ml/min (140 mmole/L) The clearance is less than 140 ml/min. The formula for calculating plasma clearance is Clearance = (Usodium) X (V) (Psodium) The clearance is less than 10 ml/min. The formula for calculating clearance is Clearance = (Usodium) X (V) (Psodium)

Darlene weighs 50 kg. Assume that her total blood volume is 8% of her body weight, that her heart pumps her total blood volume once a minute, and that her renal blood flow is 25% of her cardiac output. Calculate the volume of blood that flows through Darlene's kidneys each minute.

1 L/min

If plasma concentration of inulin=1 mginulin=1 mg inulin/mL plasma, plasma concentration of X=1 mg/mL,X=1 mg/mL, and GFR=125 GFR=125 (a) What is the filtration rate of inulin? Of X? (b) What is the excretion rate of inulin? Of X?

1 mg X/mL plasma×125 mL plasma/min=125 mg X filtered/ min. Same values for inulin. Inulin excretion=filtration= 125 mg inulin excreted/min. Cannot say what the excretion rate of X is because there is insufficient information.

List the six functions of the kidneys

1) Regulation of extracellular fluid volume and blood pressure 2) Regulation of osmolarity 3) Maintenance of ion balance 4) Homeostatic regulation of pH 5) Excretion of wastes 6) Production of hormones

What are the four major steps of hemostasis?

1) Vascular spam 2) Platelet response 3) Clotting response/ Coagulation 4) Clot removal

______1__ refers to the mass movement of fluid between the blood and the interstitial fluid as the result of ___2_____ or ___3____ pressure gradients. If the direction of bulk flow is into the capillary, the fluid movement is called ____4____. If the direction of flow is out of the capillary, the fluid movement is known as ___5_____

1) bulk flow 2) hydrostatic 3) oncotic 4) absorption 5) filtration

Sketch a concept map showing the factors that influence GFR.

1) net filtration pressure - renal blood flow and blood pressure 2) filtration coefficient - surface area of the glomerular capillaries and permeability of interface between capillary and Bowman's capsule

List the receptors that regulate osmolarity, blood volume, blood pressure, ventilation, and pH. Where are they located, what stimulates them, and what compensatory mechanisms are triggered by them?

1) osmolarity - osmoreceptors located in the hypothalamus-under high osmolarity, trigger thirst stimulation and vasopressin secretion-under low osmolarity, trigger decreased vasopressin secretion 2) decreased blood volume/pressure-carotid and aortic baroreceptors: - through the cardiovascular control center cause increased sympathetic output, decreased parasympathetic output; through the hypothalamus cause thirst stimulation and vasopressin secretion-atrial volume receptors - through the hypothalamus cause thirst stimulation and vasopressin secretion 3) increased blood pressure-carotid and aortic baroreceptors - through the cardiovascular control center causes decreased sympathetic output, increased parasympathetic output; through the hypothalamus cause thirst inhibition and vasopressin inhibition-atrial volume receptors - through the hypothalamus cause thirst inhibition and vasopressin inhibition 4) decreased pH (high plasma H+)-carotid and aortic chemoreceptors - respiratory control centers in the medulla - increase rate and depth of breathing-central chemoreceptors - respiratory control centers in the medulla - increase rate and depth of breathing

Match the nutrients on the left with the location in the GI tract where digestion of each nutrient begins: 1. carbohydrates a. mouth 2. lipids b. stomach 3. proteins c. duodenum

1-a, 2-a, 3-b

What do you see when the kidney is sliced open in a coronal section?

1. Cortex: outer region (1/3) consists of the filtering portion of nephrons. 2. Medula: inner, darker region (2/3) Medulla of collecting tubules where secretion and absorption occur. 3. renal pyramids: triangular arrangements containing straight renal tubules & blood vessels. 4. renal papillae: a piece of the pyramids. 5. renal pelvis: collecting area for all calyces prior to entering ureter. 6. major calyces: flow into the renal pelvis - collect urine from minor calyces - which in turn receive urine from collecting ducts in the pyramids. 7. renal columns: portions of cortical tissue which penetrates between the pyramid into the medulla. 8. Medullary rays: a portion of medullary tissue which appears to jut out into cortical portion of the kidney. Contains collecting tubules from nephrons in the cortex.

Function of urinary system

1. controls composition & volume of blood. 2. eliminates waste products in the blood. 3. made up of 2 kidneys, 2 ureters, bladder, and urethra.

What does endothelial-capsular membrane compose the?

1. endothelium of capillaries forming Glomerulus which contain pores 2. The basement membrane of Glomerulus which is glycoprotein matrix 3. Epithelium of visceral layer of Bowman's Capsule

What are the 2 sphincter of the bladder?

1. internal sphincter: involuntary smooth muscle continuous with detrusor 2. external sphincter: voluntary skeletal muscle which is at the level of the urogenital diaphragm.

what are the 3 layers of trigone?

1. mucosa (transitional epithelium): inner layer which can stretch 2. submucosa: the middle layer with dense CT 3. detrusor muscle: outer layer with the combination of 3 layers of smooth muscle.

The flow of fluid in Nephron:

1. only H2O & small solutes are filtered into the capsular space. 2. then to the proximal convoluted tubule in the cortex (simple cuboidal epithelium) (microvilli for reabsorption & secretion) 3. descending limb of Henle & dips down into the medulla (squamous epithelium) 4. Loop of Henle - squamous epithelium 5. tubule again become convoluted in the cortex = distal convoluted tubule cuboidal epithelium but no microvilli. 6. then flow to the collecting duct which flows through the renal pyramids of the medulla. (epithelium = cuboidal) 7. Duct empty into calyces via papillary ducts of Bellini which found in the renal papillae. (columnar) 8. depending on the distance of the nephron from the medulla, it is either called a cortical nephron or a juxtamedullary nephron.

What are the 3 layers of connective tissue in the kidneys?

1. renal (fibrous) capsule: fibrous CT coat continuous to the outermost layer of the ureter. 2. Adipose capsule: contains the perirenal fat-containing mass of adipose tissue surrounding the organ. May help to protect the kidney from trauma & temperature extreme. 3. renal facia: composed of fibrous CT. outermost layer that anchors the kidney to surrounding structures and posterior abdominal wall.

Each nephron consists of:

1. renal corpuscle or malpighion corpuscle composed of a glomerulus surrounding by Bowman's capsule. 2. proximal convoluted tubule 3. thin and think portions of Loop of Henle 4. distal convoluted tubule 5. collecting duct.

Blood supply to the kidney

1. right & left renal arteries 2. interlobar arteries 3. arcuate arteries 4.interlobular arteries 5. afferent arterioles 6. efferent arterioles 7. peritubular capillaries 8. Vasa recta 9. interlobular veins 10. arcuate veins 11. interlobar veains 12. renal veins 13. vena cava

How much urine on average leaves the body per day as urine?

1.5L/day

If the GFR = 150 mls/min, plasma concentration = 0.1 mmole/ml, the amount reabsorbed = 8 mmole/min, and the amount secreted = 3 mmole/min, what is the amount of solute excreted?

10 mmole/min

Compare the plasma entering the afferent arteriole to the composition and relative volume of the filtration fraction. (Fig. 19.4)

100% of plasma passes through the afferent arteriole. 20% of that gets filtered and < 1% us excreted into the environment

Mrs. Adler is participating in the clinical trials for a new memory drug tentatively called QuikSmart. Data collected from Mrs. Adler's trials have been placed in the table below. Use this data to answer the following two questions. What is Mrs. Adler's glomerular filtration rate (rounded to the nearest liter)?

126 L/day

Approximately how much plasma moves through the kidneys of an adult during a 24-hour period?

180 Liters

How much plasma on average enters the nephrons per day?

180L/day

Put the events associated with the mechanism of action of cholera in the correct sequential order. 1. Adenylyl cyclase is turned on, producing cAMP. 2. Cholera binds to intestinal cells. 3. CFTR channels are opened, increasing Cl-diffusion into the lumen of the intestine. 4. Cholera is brought into the cell via endocytosis.

2. Cholera binds to intestinal cells. 4.Cholera is brought into the cell via endocytosis. 1. Adenylyl cyclase is turned on, producing cAMP. 3. CFTR channels are opened, increasing Cl-diffusion into the lumen of the intestine.

At any given time, what percentage of cardiac output goes to the kidneys?

20-25%

Calculate the net filtration pressure if capillary hydrostatic pressure is 60 mm Hg, capillary osmotic pressure is 25 mm Hg, and capsular hydrostatic pressure is 10 mm Hg.

25 mm Hg (60 - (25 + 10) = 25 mm Hg. The two pressures that oppose filtration must be subtracted from the force favoring filtration.)

Calculate the net filtration pressure if capillary hydrostatic pressure is 60 mm Hg, capillary osmotic pressure is 25 mm Hg, and capsular hydrostatic pressure is 10 mm Hg.

25 mm Hg Yes, 60 - (25 + 10) = 25 mm Hg. The two pressures that oppose filtration must be subtracted from the force favoring filtration.

Match the following plasma proteins with the correct function. A. albumins B. globulins C. fibrinogen D. antibodies 36) contribute(s) significantly to osmotic pressure of plasma 37) help(s) defend the body against germs 38) essential to the process of blood clotting 39) a category that includes clotting factors and enzymes

36) A 37) D 38) C 39) B

In extreme dehydration, urine can reach a concentration of 1400 mOsM. If the minimum amount of waste solute that a person must excrete daily is about 600 milliosmoles, what is the minimum urine volume that is excreted in one day?

429 mL (600 mosmol/? L=1400 mosmol/L)

The urinary bladder can hold about ______________ mL.

500mL

The normal pH range for most body fluids is ________

7.38 to 7.42

In which segment of the nephron does most reabsorption take place? When a molecule or ion is reabsorbed from the lumen of the nephron, where does it go? If a solute is filtered and not reabsorbed from the tubule, where does it go?

70% occurs in the proximal tubule. Reabsorbed molecules go into the peritubular capillaries and the systemic venous circulation. If filtered and not reabsorbed, a molecule is excreted in the urine.

Given the following data, renal blood flow is __________. PPAH = 4 mmole/L UPAH = 950 mmole/L V = 2 mls/min hematocrit = 48%

913.5 mls/min

115) List the four basic processes of the digestive system, and describe each.

: 1. Digestion is the chemical and mechanical breakdown of foods into smaller units. 2. Absorption is the active or passive transfer of substances from the lumen of the GI tract to the extracellular fluid. 3. Motility is the movement of material in the GI tract as a result of muscle contraction. 4. Secretion refers to both transepithelial transfer of water and ions from the ECF to digestive tract lumen and the release of substances synthesized by GI epithelial cells. Section Title: Digestive Function and Processes Learning Outcome: 21.11

: Level II: Reviewing Concepts (Bloom's Taxonomy: Application) 72) What are the two ways of bicarbonate reabsorption in the proximal tubule?

: 1. convert HCO3- into carbon dioxide, then back into HCO3- 2. through the metabolism of glutamine (see Fig. 20.17) Section Title: Acid-Base Balance Learning Outcome: 20.15

: Level III: Problem Solving (Bloom's Taxonomy: Analysis) 83) Diabetes mellitus produces many homeostatic imbalances, including acidosis. The pH imbalance is due to ketoacidosis, which results from excessive accumulation of by-products of fat metabolism, as the body cannot meet energy needs from carbohydrate metabolism. Sally is a teenaged diabetic who sometimes rebels by not taking her insulin. Her body is beginning to develop ketoacidosis as a result. Create a chart to indicate and explain how her blood pH, HCO3-, and PCO2 react, by indicating "increase," "decrease," or "no change." 1. ketoacidosis has just developed 2. respiratory compensation occurs 3. renal compensation occurs

: 1. pH decreases by definition, HCO3- decreases as the carbonic acid reaction shifts to the right, PCO2 is unchanged because the excess is expelled by the lungs due to the higher gradient. 2. pH increases and HCO3- decreases as exhaled CO2 drives the carbonic acid reaction to the right, and PCO2 decreases as it is exhaled. 3. pH increases as H+ is excreted, HCO3- increases as it is reabsorbed, and increases as respiration returns to normal. Section Title: Acid-Base Balance Learning Outcome: 20.18

: Level II: Reviewing Concepts (Bloom's Taxonomy: Comprehension) 70) What three stimuli control vasopressin secretion? The most potent stimulus for vasopressin release is ________.

: 1. plasma osmolarity 2. blood volume 3. blood pressure; an increase in plasma osmolarity Section Title: Water Balance Learning Outcome: 20.2

125) One way to determine the total energy in calories of a food is to burn it in a device called a bomb calorimeter. The heat produced during combustion minus the heat added to begin the combustion is the total calories in the food. The typical human digestive tract does not absorb all of the calories in food; thus it is not 100% efficient. Furthermore, calories in undigestible organic compounds may not be absorbed at all, and thus can be considered to be zero calorie relative to human nutrition. Artificial sweeteners and fats are designed to produce desired sensation in the mouth without being absorbed. These are organic compounds that would release heat in a bomb calorimeter, thus they technically have calories. A. What does this suggest about the accuracy of the bomb calorimeter in determining available food calories for humans? What would be the results of bomb calorimetry of human feces? B. What is it about a particular food substance that may prevent absorption of its calories? C. What happens to the "zero-calorie" molecules as they pass through the digestive tract? What are the potential effects on defecation?

: A. Bomb calorimetry is not 100% accurate, because it does not take into account that some calories are not absorbed. Human feces would be determined to contain some calories. B. In order for nutrients in a food to be usable by the body, they must be chemically digested and absorbed. Digestive enzymes exhibit specificity, so they do not digest molecules that are different in structure from their substrate. If an artificial food additive is undigestible, this simply means that the body lacks the appropriate enzyme. C. Gut bacteria may be able to digest and absorb these compounds. Extra organic solute present in the gut may cause retention of water and production of bacterial gas as the bacteria digest the substance in question. This can lead to distention, discomfort, and diarrhea. Section Title: Digestion and Absorption Learning Outcome: 21.10

: Level I: Reviewing Facts and Terms (Bloom's Taxonomy: Knowledge) 70) A diet of pure carbohydrate would produce an RQ of ________.

: 1.0 Section Title: Energy Balance Learning Outcome: 22.2

: Level I: Reviewing Facts and Terms (Bloom's Taxonomy: Comprehension) 46) The normal pH range for most body fluids is ________.

: 7.35 to 7.45 Section Title: Acid-Base Balance Learning Outcome: 20.14

: Level I: Reviewing Facts and Terms (Bloom's Taxonomy: Comprehension) 42) A person who suffers from emphysema will exhibit signs of A) respiratory acidosis. B) respiratory alkalosis. C) metabolic acidosis. D) metabolic alkalosis. E) None of the answers are correct.

: A Section Title: Acid-Base Balance Learning Outcome: 20.18

: Level I: Reviewing Facts and Terms (Bloom's Taxonomy: Comprehension) Match the following structures with the appropriate description. A. appendix B. colon C. duodenum D. ileum E. jejunum 45) located at the ventral end of the cecum

: A Section Title: Anatomy of the Digestive System Learning Outcome: 21.1

: Level I: Reviewing Facts and Terms (Bloom's Taxonomy: Comprehension) A. pancreas B. pylorus C. rectum D. small intestine E. stomach 48) organ that adds exocrine secretions to the duodenum via a duct

: A Section Title: Anatomy of the Digestive System Learning Outcome: 21.11

8) The layer of loose connective tissue beneath the digestive epithelium is the A) lamina propria. B) muscularis mucosae. C) submucosa. D) submucosal plexus. E) myenteric plexus.

: A Section Title: Anatomy of the Digestive System Learning Outcome: 21.2

91) Put the following steps of fat digestion and absorption in order: 1. Bile salts coat fat droplets. 2. Cholesterol is transported into cells. 3. Chylomicrons are formed. 4. Chylomicrons are released into lymphatic system. 5. Monoglycerides and fatty acids move out of micelles. 6. Pancreatic lipase and colipase break down fats. A) 1, 6, 5, 2, 3, 4 B) 1, 2, 3, 4, 6, 5 C) 2, 6, 5, 1, 3, 4 D) 6, 3, 4, 1, 2, 5 E) None of the answers are correct.

: A Section Title: Digestion and Absorption Learning Outcome: 21.10

: Level I: Reviewing Facts and Terms (Bloom's Taxonomy: Comprehension) 40) Which of the following statements is true? A) Glucose and galactose absorption uses an apical Na+-glucose SGLT symporter. B) Fructose moves across the apical membrane by active transport. C) Glucose and galactose use different transporters in absorption. D) A basolateral GLUT5 transporter moves glucose out of the intestinal epithelial cell. E) None of the statements are true.

: A Section Title: Digestion and Absorption Learning Outcome: 21.10

A. gastrin B. cholecystokinin C. secretin D. motilin E. gastric inhibitory peptide 110) blocks cholesterol absorption

: A Section Title: Digestion and Absorption Learning Outcome: 21.10

: Level I: Reviewing Facts and Terms (Bloom's Taxonomy: Comprehension) A. enzymes B. HCl C. HCO3- D. mucus E. more than one of these 59) brush border

: A Section Title: Digestion and Absorption Learning Outcome: 21.2

: Level I: Reviewing Facts and Terms (Bloom's Taxonomy: Knowledge) 32) Bile is A) secreted by hepatocytes only. B) manufactured by the gallbladder only. C) released into the stomach only. D) secreted by hepatocytes and manufactured by the gallbladder. E) secreted by hepatocytes, manufactured by the gallbladder, and released into the stomach.

: A Section Title: Digestion and Absorption Learning Outcome: 21.8

137) In the body, 80% of all lymphocytes, a type of immune system cell, are thought to be present in the A) small intestine. B) large intestine. C) appendix. D) stomach. E) mouth.

: A Section Title: Digestive Function and Processes Learning Outcome: 21.1

: Level I: Reviewing Facts and Terms (Bloom's Taxonomy: Knowledge) 9) The respiratory quotient is ________ for consumed carbohydrates compared to proteins and fats. A) highest B) lowest C) the same

: A Section Title: Energy Balance Learning Outcome: 22.2

: Level I: Reviewing Facts and Terms (Bloom's Taxonomy: Knowledge) 4) Energy input equals energy output. This statement is known as the A) first law of thermodynamics. B) second law of thermodynamics. C) law of mass balance.

: A Section Title: Energy Balance Learning Outcome: 22.2

: Level I: Reviewing Facts and Terms (Bloom's Taxonomy: Knowledge) 54) wound healing, recovering from surgery

: A Section Title: Energy Balance Learning Outcome: 22.2

: Level I: Reviewing Facts and Terms (Bloom's Taxonomy: Knowledge) 55) enlarging one's muscles through body-building exercises

: A Section Title: Energy Balance Learning Outcome: 22.2

: Level II: Reviewing Concepts (Bloom's Taxonomy: Knowledge) 104) Two individuals are the same age, sex, and ethnic background. While being tested for their BMR, Bill consumes 20 liters of oxygen/hour and Randy consumes 16 liters of oxygen/hour. Which of the two needs to consume the most calories in order to maintain proper health and constant weight? A) Bill B) Randy

: A Section Title: Energy Balance Learning Outcome: 22.2

: Level I: Reviewing Facts and Terms (Bloom's Taxonomy: Knowledge) Match the term to its definition. A. metabolism B. catabolism C. anabolism 64) the sum of all the body's chemical reactions

: A Section Title: Energy Balance Learning Outcome: 22.4

: Level II: Reviewing Concepts (Bloom's Taxonomy: Knowledge) 96) In the fasted state, which of the following statements is FALSE? A) Skeletal muscle will increase gluconeogenesis. B) Pyruvate and lactate are transported to liver to make glucose. C) Some amino acids will be deaminated. D) Adipose breaks down its store of triglycerides.

: A Section Title: Fasted-State Metabolism Learning Outcome: 22.5

: Level II: Reviewing Concepts (Bloom's Taxonomy: Comprehension) 97) The level of ketone bodies in the blood increases when high levels of ________ are being metabolized. A) fatty acids B) glucose C) proteins D) amino acids E) carbohydrates

: A Section Title: Fasted-State Metabolism Learning Outcome: 22.7

: Level I: Reviewing Facts and Terms (Bloom's Taxonomy: Knowledge) 34) During the absorptive state, A) the liver forms glycogen. B) adipocytes contribute fatty acids to the circulation. C) skeletal muscles break down glycogen. D) glucagon levels are elevated. E) All of these answers are correct.

: A Section Title: Fed-State Metabolism Learning Outcome: 22.5

: Level I: Reviewing Facts and Terms (Bloom's Taxonomy: Knowledge) 5) Kidneys respond relatively ________ to changes in blood volume. A) slowly B) quickly

: A Section Title: Fluid and Electrolyte Homeostasis Learning Outcome: 20.1

: Level I: Reviewing Facts and Terms (Bloom's Taxonomy: Knowledge) 30) D cells in the islet of Langerhans secrete A) somatostatin. B) amylin. C) insulin. D) glucagon. E) pancreatic polypeptide.

: A Section Title: Homeostatic Control of Metabolism Learning Outcome: 22.11

: Level I: Reviewing Facts and Terms (Bloom's Taxonomy: Knowledge) 31) When blood glucose levels rise, as in the "fed" state, A) only insulin is released. B) only glucagon is released. C) only peripheral cells take up less glucose. D) only protein synthesis decreases. E) All of these answers are correct.

: A Section Title: Homeostatic Control of Metabolism Learning Outcome: 22.11

: Level I: Reviewing Facts and Terms (Bloom's Taxonomy: Comprehension) 44) Amylin A) is co-secreted with insulin. B) is co-secreted with glucagon. C) helps regulate glucose homeostasis by speeding up gastric emptying. D) is co-secreted with insulin and helps regulate glucose homeostasis by speeding up gastric emptying. E) is co-secreted with glucagon and helps regulate glucose homeostasis by speeding up gastric emptying.

: A Section Title: Homeostatic Control of Metabolism Learning Outcome: 22.12

: Level I: Reviewing Facts and Terms (Bloom's Taxonomy: Knowledge) 29) The beta cells of the pancreas produce A) insulin. B) glucagon. C) renin. D) cortisol. E) digestive enzymes.

: A Section Title: Homeostatic Control of Metabolism Learning Outcome: 22.12

: Level I: Reviewing Facts and Terms (Bloom's Taxonomy: Knowledge) 51) Pre-diabetes is a condition that will likely become diabetes if eating and exercise habits are not altered. A) True B) False

: A Section Title: Homeostatic Control of Metabolism Learning Outcome: 22.14

: Level II: Reviewing Concepts (Bloom's Taxonomy: Knowledge) 95) Drugs used to treat diabetes may A) stimulate beta-cell secretion of insulin. B) stimulate digestion and absorption of carbohydrates in the intestine. C) stimulate hepatic glucose output. D) decrease responsiveness of target tissue to insulin. E) None of these answers are correct.

: A Section Title: Homeostatic Control of Metabolism Learning Outcome: 22.14

: Level II: Reviewing Concepts (Bloom's Taxonomy: Comprehension) 62) Two hours before major surgery is to begin, the patient experiences "jitters," an elevated heart rate and blood pressure, increased rate of breathing, cold sweats, and an urge to urinate. These symptoms are the result of A) sympathetic activation. B) decreased levels of epinephrine in the blood. C) decreased activity of sympathetic centers in the hypothalamus. D) increased parasympathetic activity. E) all of these mechanisms.

: A Section Title: Integrated Control of Volume and Osmolarity Learning Outcome: 20.1

: Level I: Reviewing Facts and Terms (Bloom's Taxonomy: Knowledge) Match the structure to its function. A. mouth B. stomach C. rectum D. small intestine E. large intestine 51) Carbohydrate digestion begins here.

: A Section Title: Integrated Function: The Cephalic Phase Learning Outcome: 21.1

94) In response to the arrival of acidic chyme in the duodenum, the blood levels of A) secretin rise. B) cholecystokinin fall. C) gastrin rise. D) enterocrinin fall. E) All of these answers are correct.

: A Section Title: Integrated Function: The Cephalic Phase Learning Outcome: 21.11

97) In response to the hormone cholecystokinin, the pancreas secretes a fluid A) rich in enzymes. B) rich in bicarbonate. C) rich in bile. D) that contains only proteinases. E) that contains only amylase.

: A Section Title: Integrated Function: The Cephalic Phase Learning Outcome: 21.11

: Level I: Reviewing Facts and Terms (Bloom's Taxonomy: Comprehension) A. mouth B. stomach C. rectum D. small intestine E. large intestine 55) Fat digestion begins here.

: A Section Title: Integrated Function: The Cephalic Phase Learning Outcome: 21.11

Match the hormone with the correct statement. A. gastrin B. cholecystokinin C. secretin D. motilin E. gastric inhibitory peptide 102) secreted by cells in the stomach

: A Section Title: Integrated Function: The Cephalic Phase Learning Outcome: 21.11

13) Chief cells secrete A) pepsinogen. B) gastrin. C) mucus. D) hydrochloric acid. E) intrinsic factor.

: A Section Title: Integrated Function: The Gastric Phase Learning Outcome: 21.11

: Level I: Reviewing Facts and Terms (Bloom's Taxonomy: Knowledge) 60) reactions that result in the synthesis of large molecules

: A Section Title: Metabolism Learning Outcome: 22.4

: Level I: Reviewing Facts and Terms (Bloom's Taxonomy: Knowledge) Match the nutrient to its primary fate. A. carbohydrate B. protein C. fat 61) used immediately for energy

: A Section Title: Metabolism Learning Outcome: 22.7

: Level I: Reviewing Facts and Terms (Bloom's Taxonomy: Knowledge) 23) ________ are pacemakers for slow wave activity. A) Interstitial cells of Cajal B) Intrinsic neuron cells C) Extrinsic neuron cells D) G cells E) Chief cells

: A Section Title: Motility Learning Outcome: 21.3

: Level I: Reviewing Facts and Terms (Bloom's Taxonomy: Comprehension) 39) Which of the following statements is true about GI muscle contractions? A) Phasic contractions last only seconds and occur in the stomach and small intestine. B) Tonic contractions are sustained for minutes and occur in the small intestine. C) Cycles of smooth muscle contraction and relaxation are associated with fast wave potentials. D) Contractions of the smooth muscle do not depend on calcium. E) None of these statements are true.

: A Section Title: Motility Learning Outcome: 21.4

: Level I: Reviewing Facts and Terms (Bloom's Taxonomy: Knowledge) 48) Most heat is lost from the body through the process of A) radiation. B) conduction. C) convection. D) evaporation. E) concentration.

: A Section Title: Regulation of Body Temperature Learning Outcome: 22.16

3) Which function(s) is/are NOT continuously regulated by the GI tract? A) absorption only B) motility only C) secretion only D) All 3 functions are regulated. E) None of the 3 functions are regulated.

: A Section Title: Regulation of GI Function Learning Outcome: 21.6

: Level I: Reviewing Facts and Terms (Bloom's Taxonomy: Knowledge) 15) In the intestine, the epithelial cells have A) leaky junctions. B) tight junctions. C) electrical gap junctions.

: A Section Title: Secretion Learning Outcome: 21.12

: Level I: Reviewing Facts and Terms (Bloom's Taxonomy: Knowledge) 28) In the digestive system, HCl is released by ________, whereas HCO3- is secreted primarily from the ________. A) parietal cells of the stomach, pancreas B) the pancreas, parietal cells of the stomach C) parietal cells of the stomach, liver D) the liver, parietal cells of the stomach

: A Section Title: Secretion Learning Outcome: 21.5

: Level I: Reviewing Facts and Terms (Bloom's Taxonomy: Knowledge) 36) Decreased ECF volume causes A) sympathetic output from the cardiovascular control center to increase. B) parasympathetic output from the cardiovascular control center to increase. C) the force of ventricular contraction to decrease. D) arteriolar vasodilation. E) sympathetic output from the cardiovascular control center to increase and arteriolar vasodilation.

: A Section Title: Sodium Balance and ECF Volume Learning Outcome: 20.12

: Level II: Reviewing Concepts (Bloom's Taxonomy: Comprehension) 59) Renal tubule cells in the kidney medulla are constantly exposed to high extracellular osmolarity. How do they maintain normal cell volume? A) They synthesize organic solutes as needed to match the osmolarity. B) They synthesize water molecules through increased metabolism to offset volume loss. C) They maintain a water-impermeable membrane. D) They add or remove aquaporins as needed.

: A Section Title: Sodium Balance and ECF Volume Learning Outcome: 20.12

: Level I: Reviewing Facts and Terms (Bloom's Taxonomy: Comprehension) 25) Drugs that treat hypertension by preventing Angiotensin I from becoming Angiotensin II are called A) ACE inhibitors. B) beta blockers. C) calcium channel blockers. D) diuretics.

: A Section Title: Sodium Balance and ECF Volume Learning Outcome: 20.8

: Level I: Reviewing Facts and Terms (Bloom's Taxonomy: Comprehension) 33) The enzyme renin is responsible for the activation of A) angiotensin. B) cortisol. C) erythropoietin. D) atrial natriuretic peptide. E) adrenaline.

: A Section Title: Sodium Balance and ECF Volume Learning Outcome: 20.8

: Level I: Reviewing Facts and Terms (Bloom's Taxonomy: Comprehension) 8) Kidneys regulate A) water loss only. B) water gain only. C) both water loss and gain.

: A Section Title: Water Balance Learning Outcome: 20.1

: Level II: Reviewing Concepts (Bloom's Taxonomy: Comprehension) 63) Dan has been lost in the desert for two days with very little water. As a result, you would expect to observe A) elevated ADH levels. B) decreased blood osmolarity. C) normal urine production. D) increased blood volume. E) cells enlarged with fluid.

: A Section Title: Water Balance Learning Outcome: 20.1

: Level I: Reviewing Facts and Terms (Bloom's Taxonomy: Comprehension) 35) The osmolarity in the deepest part of the loop of Henle is ________ mOsM. A) 1200 B) 100 C) 300 D) 900 E) None of these answers are correct.

: A Section Title: Water Balance Learning Outcome: 20.12

: Level I: Reviewing Facts and Terms (Bloom's Taxonomy: Knowledge) 11) The hormone that directly controls water excretion by the kidneys is A) ADH. B) aldosterone. C) epinephrine. D) ANP. E) angiotensin.

: A Section Title: Water Balance Learning Outcome: 20.2

: Level III: Problem Solving (Bloom's Taxonomy: Application) 126) Describe a ketogenic diet. What are the pros and cons of such a diet? How is ketosis related to diabetes mellitus?

: A ketogenic diet is low in carbohydrate and high in protein and fat. Ketone bodies are produced as fat is metabolized. Pros: rapid initial weight loss and caloric restriction. Cons: initial weight loss represents loss of glycogen and water, rather than fat; dehydration, vitamin and mineral deficiency; and kidney problems. Untreated diabetics cannot metabolize carbohydrates normally, so their diets are functionally ketogenic. Section Title: Fasted-State Metabolism Learning Outcome: 22.10

: Level III: Problem Solving (Bloom's Taxonomy: Analysis) 121) The crew of an ocean-going racing sailboat must adhere to strict weight limits for both their bodies and the food they eat, yet their activity levels are very high, and weather conditions may vary to extreme cold, so they must consume many kilocalories. What kinds of nutrient-dense foods can you suggest to them to bring? In general, what type of energy nutrient should they choose to consume?

: A nutritionist's nightmare, this crew gets to eat foods everyone else is trying to avoid. Fat is high in kilocalories, but lightweight and doesn't occupy much space. Candy bars, granola with lots of nuts, and peanut butter are all good foods to have available, while vegetables, salads, and regular breakfast cereals are too bulky for the calories they provide. Section Title: Energy Balance Learning Outcome: 22.3

134) You are studying the toxic effects of a newly discovered tropical plant extract. From preliminary studies, it appears that the toxin prevents H+-dependent membrane transport. Explain which digestive processes may thus be impaired.

: Absorption of some amino acids, dipeptides, and tripeptides may be impaired, as well as iron and sodium absorption, because these are dependent upon an H+ exchanger or cotransporter. HCl secretion into the stomach, which acidifies the lumen for protein digestion, may also be impaired. Section Title: Digestive Function and Processes Learning Outcome: 21.5

128) Erica came down with the flu. She experienced severe vomiting, determined to be primarily stomach contents, for three days, and now is having chest pains. She is calling one of her symptoms "heartburn," but reported that it felt as if she had swallowed a small apple whole. Certain that she was having a heart attack, Erica rushed to the doctor. The doctor took her history and symptoms, and reassured her that she only had esophagitis. How did the physician conclude this? Is her blood pH higher or lower than normal?

: Acid reflux irritated the esophageal lining, causing it to swell. Her pH is higher than normal due to loss of H+ from her stomach. Section Title: Anatomy of the Digestive System Learning Outcome: 21.1

136) Chronic inflammation of the pancreas, or pancreatitis, impairs the digestive functions of this organ. What would be the effect on digestion and absorption of carbohydrates, fats, and proteins if acinar cells are impaired? What if duct cells are impaired? Which hormones may be used to treat pancreatitis?

: Acinar cells secrete digestive enzymes for all three classes of molecule, therefore nutrient digestion and consequent absorption would be decreased. Duct cells secrete bicarbonate, which counteracts the acidic nature of the chyme entering from the stomach and allows the pancreatic enzymes to function. The loss of these cells would also decrease digestion and absorption of the three classes of nutrient. Cholecystokinin stimulates pancreatic enzyme secretion and secretin stimulates bicarbonate secretion. Section Title: Secretion Learning Outcome: 21.11

: Level III: Problem Solving (Bloom's Taxonomy: Evaluation) 130) Describe anorexia nervosa and diabetes mellitus, and how they resemble each other.

: Anorexia nervosa is lack of adequate caloric intake due to distorted body image. Diabetes mellitus results from lack of insulin and consequent inability to utilize glucose. Both diseases result in starvation if left untreated. Section Title: Homeostatic Control of Metabolism Learning Outcome: 22.12

: Level II: Reviewing Concepts (Bloom's Taxonomy: Application) 115) Compare and contrast insulin and glucagon.

: Answers will vary but should include information in Tables 22.1 and 22.3 in the chapter. Section Title: Homeostatic Control of Metabolism Learning Outcome: 22.11

132) What would be the nutritional consequence of excessive antacid use?

: Antacids make the stomach pH higher than normal, which will interfere with protein digestion because the enzymes would not be as efficient. Section Title: Integrated Function: The Gastric Phase Learning Outcome: 21.5

: Level I: Reviewing Facts and Terms (Bloom's Taxonomy: Knowledge) 77) ________ are molecules that prevent damage to our cells by stopping the destructive effects of free radical formation.

: Antioxidants Section Title: Fasted-State Metabolism Learning Outcome: 22.8

: Level III: Problem Solving (Bloom's Taxonomy: Application) 77) Fred suffers from chronic emphysema. Blood tests show that his pH is normal but his bicarbonate levels are elevated significantly. How can this be?

: As long as the ratio of bicarbonate ion to carbonic acid is 20:1, the pH of body fluids will remain normal. Since Fred's condition is chronic (long term) his body has compensated for the excess carbonic acid (the result of hypercapnea due to poor ventilation) by increasing the amount of bicarbonate to match the elevated level of acid. This process involves the kidneys where some of the excess carbon dioxide is converted into carbonic acid and the carbonic acid is allowed to dissociate. The hydrogen ions are secreted and the newly formed bicarbonate is conserved to maintain a proper buffering capacity. Section Title: Acid-Base Balance Learning Outcome: 20.16

: Level III: Problem Solving (Bloom's Taxonomy: Analysis) 79) Mr. Gregory comes to the doctor with high blood pressure. Tests show that he also has elevated levels of renin in his blood and atherosclerotic plaques that have nearly blocked blood flow through his renal arteries. Mr. Gregory is puzzled. Explain to him how decreased blood flow in his renal arteries could cause renin secretion to increase. Map the pathways through which elevated renin causes high blood pressure for Mr. Gregory.

: Atherosclerotic plaques block blood flow, which decreases GFR and decreases pressure in the afferent arteriole. These are both stimuli for renin release. Renin secretion starts a cascade that produces angiotensin II, a vasoconstrictor. Vasoconstriction increases blood pressure, and the medullary control center responds to Ag II by also increasing ADH, aldosterone secretion, and thirst, collectively increasing blood pressure even further. Section Title: Sodium Balance and ECF Volume Learning Outcome: 20.8

: Level I: Reviewing Facts and Terms (Bloom's Taxonomy: Comprehension) 43) A person who suffers from hyperventilation will exhibit signs of A) respiratory acidosis. B) respiratory alkalosis. C) metabolic acidosis. D) metabolic alkalosis. E) None of the answers are correct.

: B Section Title: Acid-Base Balance Learning Outcome: 20.18

: Level I: Reviewing Facts and Terms (Bloom's Taxonomy: Comprehension) A. pancreas B. pylorus C. rectum D. small intestine E. stomach 49) location of the sphincter that prevents premature emptying of the stomach

: B Section Title: Anatomy of the Digestive System Learning Outcome: 21.1

: Level I: Reviewing Facts and Terms (Bloom's Taxonomy: Comprehension) Match the following structures with the appropriate description. A. appendix B. colon C. duodenum D. ileum E. jejunum 44) section where chyme is processed to remove water and electrolytes, leaving waste products of digestion

: B Section Title: Anatomy of the Digestive System Learning Outcome: 21.12

: Level I: Reviewing Facts and Terms (Bloom's Taxonomy: Comprehension) 16) The subepithelial connective tissue of the GI tract, immediately beneath the epithelium, is the A) muscularis mucosae. B) lamina propria. C) submucosa. D) serosa. E) submucosal plexus.

: B Section Title: Anatomy of the Digestive System Learning Outcome: 21.2

1) Adipocytes secrete the hormone A) ghrelin. B) leptin. C) neuropeptide Y. D) orexin. E) insulin.

: B Section Title: Appetite and Satiety Learning Outcome: 22.1

: Level I: Reviewing Facts and Terms (Bloom's Taxonomy: Knowledge) 2) Most obese humans are deficient in leptin. A) True B) False

: B Section Title: Appetite and Satiety Learning Outcome: 22.1

: Level I: Reviewing Facts and Terms (Bloom's Taxonomy: Knowledge) 5) The first law of thermodynamics states that A) the total amount of energy in the universe is variable. B) energy can be neither created nor destroyed and thus must be accounted for. C) what heats up must cool down. D) natural processes move from order to disorder, or entropy. E) there is no way a process can increase the amount of order.

: B Section Title: Appetite and Satiety Learning Outcome: 22.2

93) Decreased levels of bile salts in the bile would interfere with digestion of A) protein. B) fat. C) carbohydrates. D) nucleic acids. E) vitamins and minerals.

: B Section Title: Digestion and Absorption Learning Outcome: 21.10

A. gastrin B. cholecystokinin C. secretin D. motilin E. gastric inhibitory peptide 108) transports iron to the blood

: B Section Title: Digestion and Absorption Learning Outcome: 21.10

: Level I: Reviewing Facts and Terms (Bloom's Taxonomy: Knowledge) Classify each scenario below as to primarily which type of work is being done. A. chemical work B. transport work C. mechanical work 52) maintaining a concentration gradient across a membrane

: B Section Title: Energy Balance Learning Outcome: 22.2

: Level I: Reviewing Facts and Terms (Bloom's Taxonomy: Knowledge) 56) bringing glucose molecules inside brain cells so one can think

: B Section Title: Energy Balance Learning Outcome: 22.2

: Level I: Reviewing Facts and Terms (Bloom's Taxonomy: Knowledge) 66) large molecules broken into smaller ones

: B Section Title: Energy Balance Learning Outcome: 22.4

: Level I: Reviewing Facts and Terms (Bloom's Taxonomy: Comprehension) 23) Before converting amino acids into intermediates for energy metabolism, they must first undergo A) denaturation. B) deamination. C) detoxification. D) depeptidization. E) delousing.

: B Section Title: Fasted-State Metabolism Learning Outcome: 22.10

: Level II: Reviewing Concepts (Bloom's Taxonomy: Knowledge) 98) The Atkins and South Beach diets are considered ketogenic because A) Type I diabetics use them. B) they cause the body to burn calories from non-carbohydrate sources. C) they shift one's metabolism to increased levels of glycolysis. D) they have the potential to increase the body's pH. E) they require people to eat large quantities of ketones.

: B Section Title: Fasted-State Metabolism Learning Outcome: 22.10

: Level I: Reviewing Facts and Terms (Bloom's Taxonomy: Knowledge) 21) During the fasting state, the energy stores of the ________ become the major source of glucose for the whole body. A) brain B) liver C) muscles D) pancreas E) adipose tissues

: B Section Title: Fasted-State Metabolism Learning Outcome: 22.5

: Level I: Reviewing Facts and Terms (Bloom's Taxonomy: Knowledge) 35) During starvation, A) carbohydrate utilization increases. B) gluconeogenesis accelerates. C) there is a decline in circulating ketone bodies. D) structural proteins cannot be used as a potential energy source. E) All of these answers are correct.

: B Section Title: Fasted-State Metabolism Learning Outcome: 22.5

: Level I: Reviewing Facts and Terms (Bloom's Taxonomy: Knowledge) 15) An enzyme that digests fats is A) hydrolase. B) lipase. C) lyase. D) ligase. E) kinase.

: B Section Title: Fed-State Metabolism Learning Outcome: 22.10

: Level I: Reviewing Facts and Terms (Bloom's Taxonomy: Knowledge) 17) Which is NOT produced in the body for the purpose of storing extra calories? A) fat B) protein C) carbohydrate D) All of these are produced in the body for that purpose.

: B Section Title: Fed-State Metabolism Learning Outcome: 22.7

1) The primary route for water loss from the body is the ________ system. A) respiratory B) urinary C) digestive D) integumentary E) cardiovascular

: B Section Title: Fluid and Electrolyte Homeostasis Learning Outcome: 20.12

: Level I: Reviewing Facts and Terms (Bloom's Taxonomy: Knowledge) 2) The primary route for ion loss from the body is the ________ system. A) respiratory B) urinary C) digestive D) integumentary E) cardiovascular

: B Section Title: Fluid and Electrolyte Homeostasis Learning Outcome: 20.12

: Level I: Reviewing Facts and Terms (Bloom's Taxonomy: Knowledge) 6) Shrinkage of hepatocytes in the liver causes them to cause which of the following? A) glycogen production only B) glycogen breakdown only C) protein synthesis only D) both glycogen production and protein synthesis E) both glycogen breakdown and protein synthesis

: B Section Title: Fluid and Electrolyte Homeostasis Learning Outcome: 20.12

: Level I: Reviewing Facts and Terms (Bloom's Taxonomy: Knowledge) 32) When blood glucose levels fall, A) only insulin is released. B) only glucagon is released. C) only peripheral cells take up less glucose. D) only protein synthesis decreases. E) All of these answers are correct.

: B Section Title: Homeostatic Control of Metabolism Learning Outcome: 22.11

: Level II: Reviewing Concepts (Bloom's Taxonomy: Comprehension) 106) Both insulin and glucagon are peptide hormones that target liver cells. The responses of the target cells to these two hormones are exactly the opposite of each other. This information implies that A) one hormone binds to a receptor on the cell membrane and the other to an intracellular receptor. B) each of the two hormones uses a different second messenger. C) both hormones interact with receptors at the cell nucleus. D) one of the hormones does not interact with a membrane receptor.

: B Section Title: Homeostatic Control of Metabolism Learning Outcome: 22.11

: Level I: Reviewing Facts and Terms (Bloom's Taxonomy: Knowledge) 39) A primary target tissue for insulin is the A) brain only. B) liver only. C) intestine only. D) brain and liver. E) brain, liver, and intestine.

: B Section Title: Homeostatic Control of Metabolism Learning Outcome: 22.12

: Level I: Reviewing Facts and Terms (Bloom's Taxonomy: Knowledge) 28) The alpha cells of the pancreas produce A) insulin. B) glucagon. C) renin. D) cortisol. E) digestive enzymes.

: B Section Title: Homeostatic Control of Metabolism Learning Outcome: 22.13

: Level II: Reviewing Concepts (Bloom's Taxonomy: Comprehension) 102) In type 2 diabetes, insulin levels are frequently normal, yet the target cells are less sensitive to the effects of insulin. This suggests that the target cells A) are impermeable to insulin. B) may have a defect in their signal transduction pathway. C) cannot convert insulin to an active form. D) have adequate internal supplies of glucose. E) None of these answers are correct.

: B Section Title: Homeostatic Control of Metabolism Learning Outcome: 22.14

96) In response to the hormone secretin, the pancreas secretes a fluid A) rich in enzymes. B) rich in bicarbonate. C) rich in bile. D) that contains only proteases. E) that contains only amylase.

: B Section Title: Integrated Function: The Cephalic Phase Learning Outcome: 21.11

: Level I: Reviewing Facts and Terms (Bloom's Taxonomy: Comprehension) 25) The swallowing center in the brain, which coordinates the muscular reflexes, is located in the A) cerebrum. B) medulla oblongata. C) pons. D) cerebellum. E) hypothalamus.

: B Section Title: Integrated Function: The Cephalic Phase Learning Outcome: 21.11

: Level I: Reviewing Facts and Terms (Bloom's Taxonomy: Knowledge) 26) Which of the following events does NOT occur when you swallow? A) Respiration is inhibited. B) The upper esophageal sphincter closes. C) The glottis and epiglottis close. D) The lower esophageal sphincter relaxes.

: B Section Title: Integrated Function: The Cephalic Phase Learning Outcome: 21.11

A. gastrin B. cholecystokinin C. secretin D. motilin E. gastric inhibitory peptide 104) stimulates bile release

: B Section Title: Integrated Function: The Cephalic Phase Learning Outcome: 21.11

: Level I: Reviewing Facts and Terms (Bloom's Taxonomy: Comprehension) A. mouth B. stomach C. rectum D. small intestine E. large intestine 53) Protein digestion begins here.

: B Section Title: Integrated Function: The Gastric Phase Learning Outcome: 21.11

: Level I: Reviewing Facts and Terms (Bloom's Taxonomy: Comprehension) 36) The gastric phase of gastric secretion is triggered by A) the sight, thought, or smell of food. B) the entry of food into the stomach. C) the entry of chyme into the small intestine. D) the entry of chyme into the large intestine. E) the release of cholecystokinin and secretin by the small intestine.

: B Section Title: Integrated Function: The Gastric Phase Learning Outcome: 21.11

92) During defecation, A) stretch receptors in the rectal wall activate sympathetic centers in the spinal cord. B) stretch receptors in the rectal wall activate parasympathetic centers in the spinal cord. C) stretch receptors in the rectal wall activate both sympathetic and parasympathetic centers in the spinal cord. D) the internal anal sphincter is consciously relaxed. E) the external anal sphincter contracts.

: B Section Title: Integrated Function: The Intestinal Phase Learning Outcome: 21.11

99) Diarrhea often accompanies intestinal infections. Why? A) The immune system increases the production of enzymes for added protection. B) Bacterial toxins enhance the secretion of KCl from the cells, which causes water movement into the intestine. C) Dehydration of the body will kill the bacteria causing the infection. D) Loss of electrolytes will kill bacteria.

: B Section Title: Integrated Function: The Intestinal Phase Learning Outcome: 21.11

: Level I: Reviewing Facts and Terms (Bloom's Taxonomy: Knowledge) 11) Metabolism is a term that describes A) all work done by a living organism. B) all chemical reactions that take place within an organism. C) only chemical reactions that release ATP from living cells. D) the energy released from chemical bonds in living cells. E) the extraction of nutrients from biomolecules.

: B Section Title: Metabolism Learning Outcome: 22.2

: Level I: Reviewing Facts and Terms (Bloom's Taxonomy: Knowledge) Match each term to its definition. A. anabolism B. biochemical pathway C. catabolism 58) a series of interconnected chemical reactions

: B Section Title: Metabolism Learning Outcome: 22.4

: Level I: Reviewing Facts and Terms (Bloom's Taxonomy: Knowledge) 13) The brain relies solely on ________ as its energy source. If it is deprived of this substrate, the cells soon die. A) fatty acids B) glucose C) amino acids D) insulin

: B Section Title: Metabolism Learning Outcome: 22.7

: Level I: Reviewing Facts and Terms (Bloom's Taxonomy: Knowledge) 62) synthesis of tissues

: B Section Title: Metabolism Learning Outcome: 22.7

: Level I: Reviewing Facts and Terms (Bloom's Taxonomy: Comprehension) 22) Slow waves are A) cycles of smooth muscle contraction and relaxation. B) spontaneous cycles of depolarization and repolarization. C) reflexes that originate and are integrated in the enteric nervous system. D) peristaltic contractions. E) segmental contractions.

: B Section Title: Motility Learning Outcome: 21.3

11) The motor activity of the muscularis externa is controlled by the A) submucosal plexus. B) myenteric plexus. C) migrating motor complex. D) extrinsic neurons. E) motilin.

: B Section Title: Motility Learning Outcome: 21.7

: Level I: Reviewing Facts and Terms (Bloom's Taxonomy: Knowledge) 29) Excess potassium ions are eliminated from the body by the A) sweat glands. B) kidneys. C) liver. D) digestive system. E) spleen.

: B Section Title: Potassium Balance Learning Outcome: 20.10

: Level II: Reviewing Concepts (Bloom's Taxonomy: Comprehension) 64) What effect would a decrease in pH have on the amount of potassium ion in the urine? A) an increase in the amount of potassium in the urine B) a decrease in the amount of potassium in the urine C) no effect on the amount of potassium in the urine

: B Section Title: Potassium Balance Learning Outcome: 20.10

: Level I: Reviewing Facts and Terms (Bloom's Taxonomy: Knowledge) 49) Specific responses to changes in body temperature are regulated by the A) thermoreceptors. B) hypothalamus. C) medulla oblongata. D) skin. E) cardiac output.

: B Section Title: Regulation of Body Temperature Learning Outcome: 22.16

: Level I: Reviewing Facts and Terms (Bloom's Taxonomy: Knowledge) 47) Convective heat loss occurs when A) water evaporates from the skin's surface. B) warm air rises from the body's surface. C) a cooler object rests on the body's surface. D) one swims in water below body temperature. E) All of the answers are correct.

: B Section Title: Regulation of Body Temperature Learning Outcome: 22.16

101) A drug that blocks the action of the hormone cholecystokinin would affect A) the amount of bile produced by the liver. B) the composition of pancreatic secretions. C) the level of intestinal gastrin. D) secretions of the duodenal glands. E) All of the answers are correct.

: B Section Title: Regulation of GI Function Learning Outcome: 21.11

4) Which of the following is an accessory organ of digestion? A) stomach B) pancreas C) spleen D) colon E) esophagus

: B Section Title: Secretion Learning Outcome: 21.1

100) A drug that blocks the action of carbonic anhydrase in parietal cells would result in A) a lower pH during gastric digestion. B) a higher pH during gastric digestion. C) decreased production of pepsinogen by chief cells. D) increased protein digestion in the stomach. E) decreased gastrin production.

: B Section Title: Secretion Learning Outcome: 21.5

: Level I: Reviewing Facts and Terms (Bloom's Taxonomy: Comprehension) Match each product with the cell or region that secretes or contains it. A. enzymes B. HCl C. HCO3- D. mucus E. more than one of these 57) parietal cells

: B Section Title: Secretion Learning Outcome: 21.5

: Level I: Reviewing Facts and Terms (Bloom's Taxonomy: Comprehension) 26) Aldosterone A) is secreted in response to elevated levels of sodium in the blood. B) promotes sodium retention in the kidneys. C) helps decrease blood volume. D) increases the concentration of sodium in urine. E) functions in pH regulation.

: B Section Title: Sodium Balance and ECF Volume Learning Outcome: 20.8

: Level I: Reviewing Facts and Terms (Bloom's Taxonomy: Comprehension) 24) Which of the following is NOT true about angiotensin II? A) increases cardiac output B) activates parasympathetic output C) is a potent vasoconstrictor D) elevates blood pressure E) stimulates thirst

: B Section Title: Sodium Balance and ECF Volume Learning Outcome: 20.8

: Level II: Reviewing Concepts (Bloom's Taxonomy: Comprehension) 58) The RAAS pathway begins with A) secretion of angiotensin converting enzyme. B) secretion of the enzyme renin. C) secretion of angiotensinogen. D) secretion of aldosterone. E) secretion of antidiuretic hormone.

: B Section Title: Sodium Balance and ECF Volume Learning Outcome: 20.8

: Level I: Reviewing Facts and Terms (Bloom's Taxonomy: Comprehension) 18) When baroreceptors in the carotid and aortic bodies register increased blood pressure, this results in A) secretion of ADH. B) inhibition of ADH secretion. C) increased glomerular filtration. D) decreased urine production. E) increased thirst.

: B Section Title: Water Balance Learning Outcome: 20.1

: Level I: Reviewing Facts and Terms (Bloom's Taxonomy: Comprehension) 17) When venous return is increased, stretch receptors in the atria of the heart are activated. This results in A) secretion of ADH. B) inhibition of ADH secretion. C) increased glomerular filtration. D) decreased urine production. E) increased thirst.

: B Section Title: Water Balance Learning Outcome: 20.1

: Level I: Reviewing Facts and Terms (Bloom's Taxonomy: Knowledge) 15) Osmoreceptors depolarize after they ________ in response to ________ plasma osmolarity. A) shrink, decreased B) shrink, increased C) swell, decreased D) swell, increased

: B Section Title: Water Balance Learning Outcome: 20.3

: Level I: Reviewing Facts and Terms (Bloom's Taxonomy: Comprehension) 13) Why do patients taking loop diuretics need to take supplemental potassium? A) They cause active secretion of potassium in the loop of Henle. B) They inhibit the reabsorption of potassium as well as sodium in the loop of Henle. C) They inhibit intestinal absorption of potassium. D) They cause active reabsorption of potassium in the distal convoluted tubule.

: B Section Title: Water Balance Learning Outcome: 20.4

: Level I: Reviewing Facts and Terms (Bloom's Taxonomy: Knowledge) 84) Vitamin ________ must be complexed with a protein called ________ to be absorbed from the small intestine.

: B12, intrinsic factor Section Title: Digestion and Absorption Learning Outcome: 21.10

: Level III: Problem Solving (Bloom's Taxonomy: Analysis) 133) Calculate the body mass index of a woman who is 5'5" tall and weighs 180 lbs. Now calculate the BMI of a man with those dimensions. 1 kg = 2.2 lbs, 1 m = 39.24 in. Is either individual considered obese? Explain. Could BMI be misleading as a means of indicating obesity? Explain.

: BMI is kg/m2. 1 kg/2.2 lb × 180 lb = 81.8 kg. 5 ft × 12 in/ft = 60 in. 1 m/39.24 in × 65 in = 1.66 m. 81.8 kg/(1.66m)2 = 29.7. BMI calculation is the same for both genders, but the classification for obesity is not. For men, a BMI greater than 25 is obese; for women a BMI greater than 30 is obese. Based on this, only the man is considered obese, but the woman is on the border. BMI does not take into account the amount of lean weight versus fat weight, so a very fit and muscular person could be classified as obese based on BMI but will not have obesity-related health problems. Section Title: Energy Balance Learning Outcome: 22.2

: Level II: Reviewing Concepts (Bloom's Taxonomy: Comprehension) 118) Define basal metabolic rate (BMR) and list the six factors affecting BMR.

: BMR is an individual's lowest metabolic rate, which occurs when a person is sleeping. Factors affecting BMR are age and gender, amount of lean muscle mass, activity level, diet, hormones, and genetics. Section Title: Energy Balance Learning Outcome: 22.2

: Level III: Problem Solving (Bloom's Taxonomy: Application) 124) Charlie has a blood test that shows a normal level of LDLs but an elevated level of HDLs in his blood. Since his family has a history of cardiovascular disease, he wonders if he should modify his lifestyle. What would you tell him?

: Based just on the information given, Charlie would appear to be in good health, at least relative to his diet and exercise. Problems are associated with elevated levels of LDLs, which carry cholesterol to peripheral tissues and make it available for the formation of atherosclerotic plaques in blood vessels. High levels of HDLs indicate that a considerable amount of cholesterol is being removed from the peripheral tissues and carried to the liver for disposal. You should encourage Charlie not to change, and keep up the good work. Section Title: Fed-State Metabolism Learning Outcome: 22.7

114) List the key components of bile. Is bile action similar to the action of lipases? Explain.

: Bile salts, bile pigments, and cholesterol are the key components. Bile does not contain enzymes or have enzymatic action; rather bile emulsifies lipids so that lipases have sufficient access to their lipid substrates. Section Title: Digestion and Absorption Learning Outcome: 21.10

: Level III: Problem Solving (Bloom's Taxonomy: Application) 128) Kathy and Sally have been best friends since high school. Each is 5'5" tall and weighs about 160 lbs., which is considered to be overweight. Kathy's blood pressure is 125/82 and her blood analysis shows healthy levels of glucose and lipids. Sally's blood pressure is 135/87 and her blood analysis shows elevated glucose and triglycerides. Given that they are the same height and weight, would you expect their blood analyses to be more similar? Explain. Would you expect their body shapes to be similar? Explain. Which one is more likely to be diagnosed with metabolic syndrome? Explain.

: Blood amounts of glucose and lipids are more related to the location than to the amount of extra fat, so height and weight alone do not predict blood analysis results. The body is largely composed of bone, lean muscle mass, fat, and water. Because each of these substances has different densities, it is possible for two people to have the same weight, but a different body composition. Based on the blood work, Sally is more likely to have upper-body fat (apple shape) and/or less lean muscle mass and Kathy is more likely to have lower-body fat (pear shape) and/or more lean body mass. Sally is at greater risk for metabolic syndrome, and appears to have it, based on her weight, BP, and blood glucose and lipids. Section Title: Homeostatic Control of Metabolism Learning Outcome: 22.14

: Level I: Reviewing Facts and Terms (Bloom's Taxonomy: Comprehension) 39) The primary role of the carbonic acid-bicarbonate buffer system is A) to buffer stomach acid. B) to buffer carbonic acid formed by carbon dioxide. C) the prevention of pH changes caused by organic and fixed acids. D) to buffer the urine. E) to increase the amount of carbonic acid during ventilation.

: C Section Title: Acid-Base Balance Learning Outcome: 20.15

: Level II: Reviewing Concepts (Bloom's Taxonomy: Comprehension) 61) When the pH of the extracellular fluid declines, A) the kidneys excrete more sodium ions. B) the kidneys excrete more bicarbonate ions. C) the kidneys reabsorb more potassium ions. D) the pH of the urine increases. E) the kidneys reabsorb less water.

: C Section Title: Acid-Base Balance Learning Outcome: 20.17

95) A blockage of the ducts from the parotid glands would A) result in the production of more viscous saliva. B) impair the lubricating properties of saliva. C) interfere with carbohydrate digestion in the mouth. D) eliminate the sense of taste. E) cause all of these effects.

: C Section Title: Anatomy of the Digestive System Learning Outcome: 21.11

10) Rugae, plicae, and villi are all features that accomplish A) secretion of enzymes. B) secretion of hormones. C) an increase in surface area. D) immune function. E) mechanical digestion.

: C Section Title: Anatomy of the Digestive System Learning Outcome: 21.2

: Level I: Reviewing Facts and Terms (Bloom's Taxonomy: Knowledge) 18) The myenteric plexus is A) a layer of circular smooth muscle. B) a layer of longitudinal smooth muscle. C) a network of neurons. D) the mucus secreting layer of the digestive tract. E) primarily composed of connective tissue.

: C Section Title: Anatomy of the Digestive System Learning Outcome: 21.2

: Level I: Reviewing Facts and Terms (Bloom's Taxonomy: Knowledge) 33) The feeding and satiety centers are located in the A) medulla oblongata. B) pons. C) hypothalamus. D) cerebrum. E) cerebellum.

: C Section Title: Appetite and Satiety Learning Outcome: 22.1

: Level I: Reviewing Facts and Terms (Bloom's Taxonomy: Knowledge) 3) The brain neurotransmitter that is an important stimulus for food intake is A) ghrelin. B) leptin. C) neuropeptide Y. D) orexin. E) insulin.

: C Section Title: Appetite and Satiety Learning Outcome: 22.1

98) Digestion of a carbohydrate results in large amounts of the monosaccharides glucose and galactose. This carbohydrate was probably A) sucrose. B) maltose. C) lactose. D) cellulose. E) glycogen.

: C Section Title: Digestion and Absorption Learning Outcome: 21.10

: Level I: Reviewing Facts and Terms (Bloom's Taxonomy: Comprehension) 31) When eaten, nearly 90% of our dietary calories from fat are in the form of A) steroids. B) phospholipids. C) triglycerides. D) cholesterol. E) fat-soluble vitamins.

: C Section Title: Digestion and Absorption Learning Outcome: 21.10

: Level I: Reviewing Facts and Terms (Bloom's Taxonomy: Comprehension) 38) Most products of fat digestion are absorbed by A) capillaries. B) veins. C) lymphatic vessels. D) arterioles.

: C Section Title: Digestion and Absorption Learning Outcome: 21.10

A. gastrin B. cholecystokinin C. secretin D. motilin E. gastric inhibitory peptide 109) hormone that decreases iron absorption

: C Section Title: Digestion and Absorption Learning Outcome: 21.10

2) Chemical digestion refers to A) the progressive dehydration of indigestible residue. B) the input of food into the digestive tract. C) the enzymatic breakdown of food. D) the absorption of nutrients in the gut. E) the effects of chewing, churning, and segmentation.

: C Section Title: Digestion and Absorption Learning Outcome: 21.11

: Level I: Reviewing Facts and Terms (Bloom's Taxonomy: Comprehension) 29) Nutrient absorption occurs primarily in the A) stomach. B) liver. C) small intestine. D) large intestine. E) stomach and small intestine.

: C Section Title: Digestion and Absorption Learning Outcome: 21.12

: Level I: Reviewing Facts and Terms (Bloom's Taxonomy: Knowledge) 10) The most practical way to estimate a person's basal metabolic rate is to measure it when the person is A) sleeping. B) resting after a large meal. C) resting after a 12-hour fast.

: C Section Title: Energy Balance Learning Outcome: 22.2

: Level I: Reviewing Facts and Terms (Bloom's Taxonomy: Knowledge) 19) Diet-induced thermogenesis is highest after ingestion of A) spicy foods, like jalapeno peppers. B) fats. C) proteins. D) carbohydrates.

: C Section Title: Energy Balance Learning Outcome: 22.2

: Level I: Reviewing Facts and Terms (Bloom's Taxonomy: Knowledge) 53) working on a dock, loading and unloading boxes into trucks all day

: C Section Title: Energy Balance Learning Outcome: 22.2

: Level I: Reviewing Facts and Terms (Bloom's Taxonomy: Knowledge) 57) a thoroughbred horse, or greyhound dog, running around the track

: C Section Title: Energy Balance Learning Outcome: 22.2

: Level I: Reviewing Facts and Terms (Bloom's Taxonomy: Comprehension) 18) The nutrients that yield the most energy per gram when metabolized are A) carbohydrates. B) proteins. C) fats. D) nucleic acids. E) vitamins.

: C Section Title: Energy Balance Learning Outcome: 22.3

: Level I: Reviewing Facts and Terms (Bloom's Taxonomy: Knowledge) 65) large molecules synthesized from smaller ones

: C Section Title: Energy Balance Learning Outcome: 22.4

: Level I: Reviewing Facts and Terms (Bloom's Taxonomy: Knowledge) 24) When amino acids are metabolized for energy, the nitrogen is converted to urea and then is excreted from the body by the A) liver. B) skin. C) kidneys. D) sweat glands. E) spleen.

: C Section Title: Fasted-State Metabolism Learning Outcome: 22.10

: Level I: Reviewing Facts and Terms (Bloom's Taxonomy: Knowledge) 27) A harmful substance that the liver produces when metabolizing fatty acids is A) ammonia. B) urea. C) ketone bodies. D) rancid fatty acids. E) beta units.

: C Section Title: Fasted-State Metabolism Learning Outcome: 22.10

: Level II: Reviewing Concepts (Bloom's Taxonomy: Comprehension) 99) When blood levels of glucose, amino acids, and insulin are high, and glycogenesis is occurring in the liver, the body is in the A) fasting state. B) postabsorptive state. C) absorptive state. D) stress state. E) bulimic state.

: C Section Title: Fed-State Metabolism Learning Outcome: 22.5

: Level I: Reviewing Facts and Terms (Bloom's Taxonomy: Knowledge) 16) The liver and skeletal muscles store glucose as ________ for a ready energy source. A) lipids B) glucose C) glycogen D) urea E) ketone bodies

: C Section Title: Fed-State Metabolism Learning Outcome: 22.8

: Level I: Reviewing Facts and Terms (Bloom's Taxonomy: Comprehension) 7) Most body water is located A) in plasma. B) in interstitial fluid. C) inside cells. D) in lumens of organs open to the outside.

: C Section Title: Fluid and Electrolyte Homeostasis Learning Outcome: 20.12

: Level I: Reviewing Facts and Terms (Bloom's Taxonomy: Knowledge) 3) Cell volume (and therefore cell function) in most cells is dependent upon careful regulation of A) volume of extracellular fluid. B) blood pressure. C) osmolarity of extracellular fluid. D) permeability of cell membranes. E) resting membrane potential.

: C Section Title: Fluid and Electrolyte Homeostasis Learning Outcome: 20.12

: Level I: Reviewing Facts and Terms (Bloom's Taxonomy: Knowledge) 38) Insulin secretion A) is stimulated by sympathetic neurons. B) decreases in response to elevated amino acid concentrations. C) is stimulated by parasympathetic neurons. D) is inhibited by GLP-1. E) None of these answers are correct.

: C Section Title: Homeostatic Control of Metabolism Learning Outcome: 22.12

: Level I: Reviewing Facts and Terms (Bloom's Taxonomy: Knowledge) 45) In type I diabetes, a hyperglycemic hyperosmotic state may occur. Which of the following best describes this state? A) There are low levels of vasopressin (ADH). B) There is decreased water intake. C) Plasma glucose and blood osmolarity levels are above normal. D) ATP production is increased due to the increased levels of glucose. E) Blood osmolarity is below normal levels due to elevated glucose.

: C Section Title: Homeostatic Control of Metabolism Learning Outcome: 22.15

: Level I: Reviewing Facts and Terms (Bloom's Taxonomy: Comprehension) 35) During the cephalic phase of gastric secretion, A) the stomach responds to distention. B) secretin inhibits parietal and chief cells. C) there is an increased flow of action potentials along the vagus nerve to the stomach. D) the intestine reflexively inhibits gastric emptying. E) production of gastric juice slows down.

: C Section Title: Integrated Function: The Cephalic Phase Learning Outcome: 21.11

A. gastrin B. cholecystokinin C. secretin D. motilin E. gastric inhibitory peptide 103) inhibits gastric emptying

: C Section Title: Integrated Function: The Cephalic Phase Learning Outcome: 21.11

14) G cells of the stomach secrete A) cholecystokinin. B) secretin. C) gastrin. D) enterokinase. E) pepsin.

: C Section Title: Integrated Function: The Gastric Phase Learning Outcome: 21.11

: Level I: Reviewing Facts and Terms (Bloom's Taxonomy: Comprehension) Match the following structures with the appropriate description. A. appendix B. colon C. duodenum D. ileum E. jejunum 43) location of most peptic ulcers

: C Section Title: Integrated Function: The Intestinal Phase Learning Outcome: 21.1

: Level I: Reviewing Facts and Terms (Bloom's Taxonomy: Comprehension) 33) Functions of the large intestine include A) chemical digestion of chyme. B) temporary food storage. C) resorption of water and compaction of feces. D) absorption of the products of digestion. E) All of these answers are correct.

: C Section Title: Integrated Function: The Intestinal Phase Learning Outcome: 21.12

: Level I: Reviewing Facts and Terms (Bloom's Taxonomy: Knowledge) 59) reactions that result in the breakdown of large biomolecules

: C Section Title: Metabolism Learning Outcome: 22.4

: Level I: Reviewing Facts and Terms (Bloom's Taxonomy: Knowledge) 14) Gluconeogenesis refers to A) converting glucose to glycogen storage. B) removing glucose from storage as glycogen. C) creating glucose from glycerol, amino acids, or lactate. D) converting glucose to storage as adipose tissue. E) removing fatty acids from adipose storage.

: C Section Title: Metabolism Learning Outcome: 22.7

: Level I: Reviewing Facts and Terms (Bloom's Taxonomy: Knowledge) 63) storage

: C Section Title: Metabolism Learning Outcome: 22.7

: Level I: Reviewing Facts and Terms (Bloom's Taxonomy: Comprehension) 21) The ________ are double sheets of peritoneal membrane that hold some of the visceral organs in their proper position. A) serosa B) adventitia C) mesenteries D) fibrosa E) lamina propria

: C Section Title: Motility Learning Outcome: 21.2

: Level II: Reviewing Concepts (Bloom's Taxonomy: Comprehension) 66) Dehydration may cause some ions to become concentrated. If a person was suffering from severe hyperkalemia, you would expect A) the potassium ion concentration of the interstitial fluid to be less than normal. B) the membrane potential of nerves and muscles to be more negative. C) the skeletal muscles to be unresponsive and cardiac arrest could occur. D) muscle weakness and increased strength of twitch contractions. E) All of the answers are correct.

: C Section Title: Potassium Balance Learning Outcome: 20.10

: Level I: Reviewing Facts and Terms (Bloom's Taxonomy: Knowledge) 46) Homeothermic refers to A) natural treatments or remedies. B) shivering. C) regulating body temperature within a narrow range. D) warm relationships with others of the same gender.

: C Section Title: Regulation of Body Temperature Learning Outcome: 22.16

: Level I: Reviewing Facts and Terms (Bloom's Taxonomy: Knowledge) 21) ACE converts A) renin to angiotensinogen. B) angiotensinogen to angiotensin I. C) angiotensin I to angiotensin II. D) angiotensin II to aldosterone. E) renin to aldosterone.

: C Section Title: Sodium Balance and ECF Volume Learning Outcome: 20.8

: Level I: Reviewing Facts and Terms (Bloom's Taxonomy: Comprehension) 19) Why is sodium actively reabsorbed in the nephron? A) to decrease osmolarity inside the nephron B) to make urine less concentrated C) to increase passive reabsorption of water D) to decrease blood pressure

: C Section Title: Water Balance Learning Outcome: 20.1

: Level I: Reviewing Facts and Terms (Bloom's Taxonomy: Knowledge) 9) When a body is dehydrated, water in the urinary bladder A) can be returned to the circulation directly. B) can be returned to the circulation after moving back into the kidneys. C) will still be expelled from the body in the urine.

: C Section Title: Water Balance Learning Outcome: 20.1

: Level I: Reviewing Facts and Terms (Bloom's Taxonomy: Knowledge) 16) The hormone ADH A) is secreted by the anterior pituitary gland in response to changes in blood osmolarity. B) stimulates the kidneys to retain sodium ion. C) stimulates water conservation at the kidneys. D) causes the kidneys to produce a large volume of urine. E) All of the answers are correct.

: C Section Title: Water Balance Learning Outcome: 20.2

: Level III: Problem Solving (Bloom's Taxonomy: Application) 76) Devising new food recipes is more successful when the chefs understand basic food chemistry. For example, to create a new cake recipe, one must understand the role of both acids and bases in leavening (rising, or trapping of tiny gas bubbles as dough is baking, to provide the characteristic texture). The principle is to incorporate baking soda (NaHCO3) into an acidic batter, causing a chemical reaction resulting in gas production. Looking at the chemical formula, what gas do you think is formed? Summarize the chemical reaction that occurs in a cake batter that contains NaHCO3 and vinegar or acetic acid (produces acetate when ionized: CH3COO- + H+). What would happen to your cake if you didn't use enough baking soda? What would happen if you forgot to acidify your batter with vinegar or a similar food acid?

: CO2 gas is formed. NaHCO3 + CH3COO- + H+ → Na+ + CH3COO- + CO2 + H2O. Less gas would be produced, so the cake wouldn't rise normally. The chemical reaction would not occur, so the cake wouldn't rise. Section Title: Acid-Base Balance Learning Outcome: 20.15

135) Cholesterol is absorbed without being digested into smaller pieces. How does this compare to absorption of carbohydrates and proteins? What characteristic of cholesterol suggests it would be transported by simple diffusion? What is the evidence that transport proteins are involved in cholesterol absorption? Does this discovery rule out transport by simple diffusion? Explain.

: Carbohydrates are digested to monosaccharides before they are absorbed. Proteins are absorbed as amino acids, di- or tripeptides, or even oligopeptides. As a lipid, cholesterol can dissolve in the phospholipid bilayer and therefore cross membranes by simple diffusion. The drug ezetimibe inhibits cholesterol absorption, suggesting there must be a membrane transporter. This discovery does not rule out additional transport of cholesterol by simple diffusion, but does suggest there is a significant amount of transport dependent upon a transport molecule. Section Title: Digestion and Absorption Learning Outcome: 21.10

: Level I: Reviewing Facts and Terms (Bloom's Taxonomy: Knowledge) 74) ________ are lipoproteins that are formed in the intestine to carry lipids into circulation.

: Chylomicrons Section Title: Fed-State Metabolism Learning Outcome: 22.5

: Level II: Reviewing Concepts (Bloom's Taxonomy: Comprehension) 112) Explain the relationship among chylomicrons, cholesterol, and the various forms of lipoproteins (LDL, HDL, etc.). What is the role of each one in the body? Why are some considered "bad" and others "good"? Where does each come from? Where could each end up?

: Chylomicrons are complexes composed of triglycerides, cholesterol, and lipoprotein, assembled in the intestinal epithelium. Most lipoprotein in blood is low-density, or LDL; other forms include very low-density (VLDL) and high-density lipoprotein (HDL). The triglyceride portion of chylomicrons is a source of free fatty acids and monoglycerides for cells, which use them for energy or convert them back to triglycerides for energy storage. Cholesterol is a precursor for steroid hormones and bile components and, along with phospholipids synthesized from fatty acids, a structural component of cell membranes. LDLs are a means of transporting cholesterol to most cells, whereas HDLs transport cholesterol primarily to the liver. LDL is "bad" or "lethal" because elevated LDL is associated with atherosclerosis. HDL is "good" or "healthy" because it takes cholesterol to the liver, where it is metabolized or excreted. Section Title: Fed-State Metabolism Learning Outcome: 22.10

: Level I: Reviewing Facts and Terms (Bloom's Taxonomy: Comprehension) 83) ________ is a protein cofactor that is secreted by the pancreas and that allows lipases to access fats inside the bile coating.

: Colipase Section Title: Digestion and Absorption Learning Outcome: 21.10

113) The mucosa of the majority of the digestive tract contains simple columnar epithelium, while the esophagus and the anus have stratified squamous epithelium. Why is this the case? What purpose does each type of epithelium serve in the digestive tract?

: Columnar epithelium is primarily responsible for secreting and absorbing materials across its membranes whereas stratified squamous epithelium is primarily present for protection of the tissues beneath it. Since the stomach and intestines secrete enzymes and other chemicals important for digesting food and since they absorb nutrients and water, they have simple columnar epithelium. Since the esophagus is carrying undigested food from the mouth to the stomach and not absorbing anything, it makes sense that it would have stratified squamous epithelium to protect the tissues beneath it from the ingested food particles. Since the anus is primarily a passageway for feces to leave the body and it would not be beneficial for the feces to come into contact with the tissue beneath it, stratified squamous epithelium also makes sense in the anus. Section Title: Anatomy of the Digestive System Learning Outcome: 21.2

131) Essential nutrients are those that our cells require but cannot make, and thus they must be present in the diet. Only some amino acids and some fatty acids are essential. What does that suggest about extreme diets that eliminate fats or proteins? What does that suggest about our carbohydrate intake? Describe the typical American diet, in terms of relative amounts of ingested carbohydrates, fats, proteins, and nucleic acids. Propose some explanations for why one of those is predominant.

: Complete elimination of proteins or fats would eventually be fatal. Carbohydrates need not be consumed at all, from the standpoint of essential nutrients. In reality, however, a carbohydrate-free diet would cause metabolic problems because of consequences of manufacturing glucose from lipids and proteins, as described in Chapter 22 in the text. About half of the typical dietary intake is carbohydrate, with most of the remainder in the form of fats and proteins. Carbohydrates are the primary molecule in plant products, which makes up most of what we eat because of abundance and price. Section Title: Digestive Function and Processes Learning Outcome: 21.10

: Level I: Reviewing Facts and Terms (Bloom's Taxonomy: Knowledge) 92) ________ is the direct transfer of heat energy from one object to another.

: Conduction Section Title: Regulation of Body Temperature Learning Outcome: 22.16

: Level I: Reviewing Facts and Terms (Bloom's Taxonomy: Knowledge) 93) ________ is the transfer of heat energy to air.

: Convection Section Title: Regulation of Body Temperature Learning Outcome: 22.16

: Level I: Reviewing Facts and Terms (Bloom's Taxonomy: Comprehension) 37) Symptoms of low plasma pH may include A) CNS depression. B) confusion and disorientation. C) numbness, tingling, or muscle twitches. D) CNS depression and confusion and disorientation. E) CNS depression; confusion and disorientation; and numbness, tingling, or muscle twitches.

: D Section Title: Acid-Base Balance Learning Outcome: 20.14

: Level II: Reviewing Concepts (Bloom's Taxonomy: Comprehension) 60) When the pH of body fluids begins to fall, proteins will A) release a hydrogen from the carboxyl group. B) release a hydrogen from the amino group. C) bind a hydrogen at the carboxyl group. D) bind a hydrogen at the amino group. E) cause none of these actions.

: D Section Title: Acid-Base Balance Learning Outcome: 20.14

: Level I: Reviewing Facts and Terms (Bloom's Taxonomy: Comprehension) 38) The most important factor affecting the pH of plasma is the concentration of A) lactic acid. B) ketone bodies. C) organic acids. D) carbon dioxide. E) hydrochloric acid.

: D Section Title: Acid-Base Balance Learning Outcome: 20.18

: Level I: Reviewing Facts and Terms (Bloom's Taxonomy: Comprehension) 41) Prolonged vomiting of the stomach's contents can result in A) respiratory acidosis. B) respiratory alkalosis. C) metabolic acidosis. D) metabolic alkalosis. E) None of the answers are correct.

: D Section Title: Acid-Base Balance Learning Outcome: 20.18

: Level I: Reviewing Facts and Terms (Bloom's Taxonomy: Knowledge) Match the following structures with the appropriate description. A. appendix B. colon C. duodenum D. ileum E. jejunum 46) distal-most section of small intestine

: D Section Title: Anatomy of the Digestive System Learning Outcome: 21.1

: Level I: Reviewing Facts and Terms (Bloom's Taxonomy: Comprehension) A. pancreas B. pylorus C. rectum D. small intestine E. stomach 50) organ where most digestion occurs

: D Section Title: Anatomy of the Digestive System Learning Outcome: 21.12

19) Peyer's patches are characteristic of the A) stomach. B) esophagus. C) pancreas. D) small intestine. E) colon.

: D Section Title: Anatomy of the Digestive System Learning Outcome: 21.2

7) The lamina propria and mucous epithelium are components of the A) serosa. B) adventitia. C) muscularis mucosa. D) mucosa. E) submucosa.

: D Section Title: Anatomy of the Digestive System Learning Outcome: 21.2

: Level I: Reviewing Facts and Terms (Bloom's Taxonomy: Knowledge) 17) Sandwiched between the layer of circular and longitudinal muscle in the muscularis externa is the A) mucosa. B) submucosa. C) muscularis mucosa. D) myenteric plexus. E) submucosal plexus.

: D Section Title: Anatomy of the Digestive System Learning Outcome: 21.2

: Level I: Reviewing Facts and Terms (Bloom's Taxonomy: Knowledge) 20) Features of the submucosa include A) blood and lymph vessels. B) a major nerve network. C) Peyer's patches. D) blood, lymph vessels, and a major nerve network. E) blood, lymph vessels, a major nerve network, and Peyer's patches.

: D Section Title: Anatomy of the Digestive System Learning Outcome: 21.2

9) Contraction of the ________ alters the shape of the intestinal lumen and moves epithelial pleats and folds. A) mucosa B) submucosa C) submucosal plexus D) muscularis mucosa E) adventitia

: D Section Title: Anatomy of the Digestive System Learning Outcome: 21.4

: Level I: Reviewing Facts and Terms (Bloom's Taxonomy: Knowledge) 30) Thirst is A) controlled by centers in the hypothalamus. B) triggered by increased osmolarity. C) relieved only when plasma osmolarity is decreased. D) controlled by centers in the hypothalamus and triggered by increased osmolarity. E) controlled by centers in the hypothalamus, triggered by increased osmolarity, and relieved only when plasma osmolarity is decreased.

: D Section Title: Behavioral Mechanisms in Salt and Water Balance Learning Outcome: 20.11

5) The ________ is a significant site of absorption of water and electrolytes, but NOT of nutrients. A) mouth B) stomach C) small intestine D) large intestine E) None of the answers are correct.

: D Section Title: Digestion and Absorption Learning Outcome: 21.10

A. gastrin B. cholecystokinin C. secretin D. motilin E. gastric inhibitory peptide Match the structure or compound to its function. A. ezetimbe B. ferroportin C. hepcidin D. DMT1 E. NPC1L1 107) transporter for iron absorption

: D Section Title: Digestion and Absorption Learning Outcome: 21.10

: Level I: Reviewing Facts and Terms (Bloom's Taxonomy: Comprehension) 27) Mucus functions in A) protection only. B) lubrication only. C) enzyme activation only. D) protection and lubrication. E) protection, lubrication, and enzyme activation.

: D Section Title: Digestive Function and Processes Learning Outcome: 21.10

: Level I: Reviewing Facts and Terms (Bloom's Taxonomy: Knowledge) 7) A bomb calorimeter measures A) the carbon dioxide produced when a food sample is completely combusted. B) the oxygen consumed when a food sample is completely combusted. C) only the food calories available to the human body. D) the heat produced when a food sample is completely combusted. E) both carbon dioxide produced and oxygen consumed when a food sample is completely combusted.

: D Section Title: Energy Balance Learning Outcome: 22.2

: Level I: Reviewing Facts and Terms (Bloom's Taxonomy: Knowledge) 8) Which of the following can be measured to determine a person's metabolic rate? A) only heat released from body in sealed chamber B) only oxygen consumed by body in sealed chamber C) only carbon dioxide produced by body in sealed chamber D) heat released from body in sealed chamber, oxygen consumed by body in sealed chamber, and carbon dioxide produced by body in sealed chamber E) None of these answers are correct.

: D Section Title: Energy Balance Learning Outcome: 22.2

: Level I: Reviewing Facts and Terms (Bloom's Taxonomy: Knowledge) 6) One kilocalorie (kcal) is the amount of energy needed to raise one ________ of water by 1 degree Celsius. A) milliliter B) tablespoon C) cup D) liter E) gallon

: D Section Title: Energy Balance Learning Outcome: 22.3

: Level I: Reviewing Facts and Terms (Bloom's Taxonomy: Knowledge) 25) The reactions where fats are broken down into glycerol and fatty acids are called A) glycerolysis. B) glycolysis. C) gluconeogenesis. D) lipolysis. E) liposuction.

: D Section Title: Fasted-State Metabolism Learning Outcome: 22.10

: Level II: Reviewing Concepts (Bloom's Taxonomy: Knowledge) 100) If you were in a desert without a food source, which nutrient would you like to have stored in your body in a large amount? A) protein B) glycogen C) calcium D) fat E) vitamin B6

: D Section Title: Fasted-State Metabolism Learning Outcome: 22.5

: Level I: Reviewing Facts and Terms (Bloom's Taxonomy: Knowledge) 22) Lipoproteins that carry mostly cholesterol and phospholipids from peripheral tissues to the liver are called A) very low-density lipoproteins (VLDLs). B) low-density lipoproteins (LDLs). C) intermediate-density lipoproteins (IDLs). D) high-density lipoproteins (HDLs). E) very high-density lipoproteins (VHDLs).

: D Section Title: Fed-State Metabolism Learning Outcome: 22.7

: Level I: Reviewing Facts and Terms (Bloom's Taxonomy: Knowledge) 4) The two organ systems that work together to regulate most aspects of the body's water balance are A) digestive and respiratory. B) urinary and respiratory. C) cardiovascular and respiratory. D) urinary and cardiovascular. E) digestive and cardiovascular.

: D Section Title: Fluid and Electrolyte Homeostasis Learning Outcome: 20.1

: Level I: Reviewing Facts and Terms (Bloom's Taxonomy: Knowledge) 42) Glucagon A) only stimulates gluconeogenesis. B) primarily targets the liver. C) primarily targets skeletal muscle. D) stimulates gluconeogenesis and primarily targets the liver. E) stimulates gluconeogenesis and primarily targets skeletal muscle.

: D Section Title: Homeostatic Control of Metabolism Learning Outcome: 22.13

A. gastrin B. cholecystokinin C. secretin D. motilin E. gastric inhibitory peptide 106) smooth muscle of duodenum is a target

: D Section Title: Integrated Function: The Cephalic Phase Learning Outcome: 21.11

: Level I: Reviewing Facts and Terms (Bloom's Taxonomy: Comprehension) 34) The release of many GI tract hormones is stimulated by a particular food or substance. Which hormone is INCORRECTLY paired with its stimulus? A) CCK — fatty foods B) secretin — acid in the small intestine C) GIP — glucose in the small intestine D) motilin — acid in the stomach E) gastrin — peptides, increased sympathetic activity

: D Section Title: Integrated Function: The Cephalic Phase Learning Outcome: 21.8

: Level I: Reviewing Facts and Terms (Bloom's Taxonomy: Comprehension) A. mouth B. stomach C. rectum D. small intestine E. large intestine 54) Protein digestion is completed here.

: D Section Title: Integrated Function: The Intestinal Phase Learning Outcome: 21.11

: Level I: Reviewing Facts and Terms (Bloom's Taxonomy: Comprehension) A. mouth B. stomach C. rectum D. small intestine E. large intestine 56) Fat digestion is completed here.

: D Section Title: Integrated Function: The Intestinal Phase Learning Outcome: 21.11

: Level I: Reviewing Facts and Terms (Bloom's Taxonomy: Comprehension) A. mouth B. stomach C. rectum D. small intestine E. large intestine 52) Carbohydrate digestion is completed here.

: D Section Title: Integrated Function: The Intestinal Phase Learning Outcome: 21.12

: Level I: Reviewing Facts and Terms (Bloom's Taxonomy: Knowledge) 20) The synthesis of glucose from a noncarbohydrate precursor is referred to as A) glycogenesis. B) glycogenolysis. C) glycolysis. D) gluconeogenesis. E) glycogen.

: D Section Title: Metabolism Learning Outcome: 22.4

: Level I: Reviewing Facts and Terms (Bloom's Taxonomy: Comprehension) 28) An increase in plasma potassium levels is properly called A) hypernatremia. B) hyperpotassemia. C) hyperpotasseplasmia. D) hyperkalemia. E) hypercalcemia.

: D Section Title: Potassium Balance Learning Outcome: 20.10

: Level I: Reviewing Facts and Terms (Bloom's Taxonomy: Knowledge) 50) Heat loss is promoted by A) sweating. B) dilation of cutaneous blood vessels. C) nonshivering thermogenesis. D) sweating and dilation of cutaneous blood vessels. E) sweating, dilation of cutaneous blood vessels, and nonshivering thermogenesis.

: D Section Title: Regulation of Body Temperature Learning Outcome: 22.16

: Level I: Reviewing Facts and Terms (Bloom's Taxonomy: Comprehension) A. enzymes B. HCl C. HCO3- D. mucus E. more than one of these 58) goblet cells

: D Section Title: Secretion Learning Outcome: 21.11

: Level I: Reviewing Facts and Terms (Bloom's Taxonomy: Comprehension) 20) Juxtaglomerular cells in the nephron secrete A) angiotensinogen. B) angiotensin I. C) aldosterone. D) renin. E) angiotensin converting enzyme.

: D Section Title: Sodium Balance and ECF Volume Learning Outcome: 20.7

: Level I: Reviewing Facts and Terms (Bloom's Taxonomy: Knowledge) 22) Stimuli for the activation of the RAAS pathway include A) low blood pressure in arterioles in the nephron only. B) a decrease in fluid flow through the distal tubule only. C) high blood pressure in the renal artery only. D) low blood pressure in arterioles in the nephron and a decrease in fluid flow through the distal tubule. E) low blood pressure in arterioles in the nephron, a decrease in fluid flow through the distal tubule, and high blood pressure in the renal artery.

: D Section Title: Sodium Balance and ECF Volume Learning Outcome: 20.8

: Level I: Reviewing Facts and Terms (Bloom's Taxonomy: Comprehension) 27) Atrial natriuretic peptide A) increases the GFR. B) inhibits the release of renin. C) stimulates the release of renin. D) increases the GFR and inhibits the release of renin. E) increases the GFR and stimulates the release of renin.

: D Section Title: Sodium Balance and ECF Volume Learning Outcome: 20.9

: Level I: Reviewing Facts and Terms (Bloom's Taxonomy: Comprehension) 10) Water reabsorption by the kidneys is a result of A) both passive and active transport processes. B) cotransport with ions. C) exchange with ions. D) osmosis.

: D Section Title: Water Balance Learning Outcome: 20.2

: Level I: Reviewing Facts and Terms (Bloom's Taxonomy: Knowledge) 12) The hormone that regulates water excretion by the kidneys A) decreases water permeability throughout the kidney tubules. B) increases water permeability throughout the kidney tubules. C) only decreases water permeability in certain portions of the kidney tubules. D) only increases water permeability in certain portions of the kidney tubules.

: D Section Title: Water Balance Learning Outcome: 20.2

: Level I: Reviewing Facts and Terms (Bloom's Taxonomy: Comprehension) 14) The primary osmoreceptors are located in the A) pons. B) kidney. C) stomach. D) hypothalamus. E) medulla.

: D Section Title: Water Balance Learning Outcome: 20.3

: Level I: Reviewing Facts and Terms (Bloom's Taxonomy: Knowledge) 82) ________ is the amount of heat generated during the digestion of a meal.

: Diet-induced thermogenesis Section Title: Energy Balance Learning Outcome: 22.2

: Level I: Reviewing Facts and Terms (Bloom's Taxonomy: Comprehension) 40) As a result of respiratory alkalosis, A) the respiratory rate increases. B) the tidal volume increases. C) the kidneys conserve bicarbonate. D) the kidneys secrete fewer hydrogen ions. E) the body retains less carbon dioxide.

: E Section Title: Acid-Base Balance Learning Outcome: 20.18

: Level II: Reviewing Concepts (Bloom's Taxonomy: Comprehension) 65) In response to a rapid increase of organic acid in the body, you would expect to observe A) increased alveolar ventilation only. B) decreased blood pressure only. C) decreased heart rate only. D) decreased blood pH only. E) increased alveolar ventilation and decreased blood pH.

: E Section Title: Acid-Base Balance Learning Outcome: 20.18

: Level I: Reviewing Facts and Terms (Bloom's Taxonomy: Knowledge) Match the following structures with their functions. A. pancreas B. pylorus C. rectum D. small intestine E. stomach 47) chyme is released from here

: E Section Title: Anatomy of the Digestive System Learning Outcome: 21.1

6) Which of the following is the sequence of layers from the lumen to the outer wall of the digestive tract? A) serosa, submucosa, mucosa, muscularis externa B) submucosa, mucosa, serosa, muscularis externa C) mucosa, submucosa, serosa, muscularis externa D) submucosa, muscularis externa, serosa, mucosa E) mucosa, submucosa, muscularis externa, serosa

: E Section Title: Anatomy of the Digestive System Learning Outcome: 21.2

: Level I: Reviewing Facts and Terms (Bloom's Taxonomy: Comprehension) 31) A hormone that helps to regulate the sodium ion concentration of the blood is A) cortisol. B) parathormone. C) thymosin. D) somatotropin. E) aldosterone.

: E Section Title: Behavioral Mechanisms in Salt and Water Balance Learning Outcome: 20.11

: Level I: Reviewing Facts and Terms (Bloom's Taxonomy: Comprehension) 30) Amylases, the enzymes used to digest carbohydrates, are secreted by A) salivary glands into the mouth only. B) gastric glands into the stomach only. C) the pancreas into the intestine only. D) salivary glands into the mouth and gastric glands into the stomach. E) salivary glands into the mouth and the pancreas into the intestine.

: E Section Title: Digestion and Absorption Learning Outcome: 21.10

A. gastrin B. cholecystokinin C. secretin D. motilin E. gastric inhibitory peptide 111) transporter for cholesterol absorption

: E Section Title: Digestion and Absorption Learning Outcome: 21.10

12) Intestinal crypts A) only increase the surface area of the mucosa of the small intestine. B) only carry products of digestion that will not pass through the walls of blood capillaries. C) only produce new cells for the mucosa of the small intestine. D) only function in the absorption of nutrients. E) increase the surface area of the mucosa of the small intestine and produce new cells for the mucosa of the small intestine.

: E Section Title: Digestion and Absorption Learning Outcome: 21.2

1) Mechanical digestion refers to A) the progressive dehydration of indigestible residue. B) the input of food into the digestive tract. C) the enzymatic breakdown of food. D) the absorption of nutrients in the gut. E) the effects of chewing, churning, and segmentation.

: E Section Title: Digestion and Absorption Learning Outcome: 21.4

: Level I: Reviewing Facts and Terms (Bloom's Taxonomy: Comprehension) 37) An enzyme that will digest proteins into peptides is A) lipase. B) amylase. C) nuclease. D) maltase. E) trypsin.

: E Section Title: Digestion and Absorption Learning Outcome: 21.8

: Level I: Reviewing Facts and Terms (Bloom's Taxonomy: Knowledge) 37) Which of the following factors increases basal metabolic rate? A) only thyroid hormones B) only acetylcholine C) only insulin D) only epinephrine E) thyroid hormones and epinephrine

: E Section Title: Energy Balance Learning Outcome: 22.2

: Level I: Reviewing Facts and Terms (Bloom's Taxonomy: Knowledge) 26) The process of disassembling fatty acids into two-carbon units inside mitochondria is called A) oxidative phosphorylation. B) deamination. C) ketonization. D) chemiosmosis. E) beta-oxidation.

: E Section Title: Fasted-State Metabolism Learning Outcome: 22.10

: Level II: Reviewing Concepts (Bloom's Taxonomy: Knowledge) 105) On a tour of African countries, Don contracts a bad case of traveler's diarrhea. Because he can't eat very much, his body starts to use energy sources other than carbohydrates. This would result in A) increased levels of urea in the blood. B) ketosis. C) a decreased blood pH. D) increased gluconeogenesis in the liver. E) All of these answers are correct.

: E Section Title: Fasted-State Metabolism Learning Outcome: 22.5

: Level II: Reviewing Concepts (Bloom's Taxonomy: Comprehension) 103) Which of the following drugs would be an effective means for lowering plasma cholesterol levels? A) a drug that binds to bile acids and prevents them from being reabsorbed B) a drug that stimulates intestinal cholesterol transport C) a drug that stimulates the enzyme HMG coA reductase D) a drug that adds plant sterols and stanols to the diet E) a drug that binds to bile acids and prevents them from being reabsorbed or a drug that adds plant sterols and stanols to the diet

: E Section Title: Fed-State Metabolism Learning Outcome: 22.1

: Level I: Reviewing Facts and Terms (Bloom's Taxonomy: Knowledge) 41) Insulin A) only stimulates glycolysis. B) only stimulates lipogenolysis. C) only inhibits gluconeogenesis. D) stimulates glycolysis and lipogenolysis. E) stimulates glycolysis and inhibits gluconeogenesis.

: E Section Title: Homeostatic Control of Metabolism Learning Outcome: 22.12

: Level I: Reviewing Facts and Terms (Bloom's Taxonomy: Knowledge) 40) GLUT4 transporters are A) only stored in cytoplasmic vesicles. B) only inserted in response to glucagon. C) only inserted into the plasma membrane by endocytosis. D) only found in adipose and skeletal muscles. E) stored in cytoplasmic vesicles and found in adipose and skeletal muscles.

: E Section Title: Homeostatic Control of Metabolism Learning Outcome: 22.12

: Level I: Reviewing Facts and Terms (Bloom's Taxonomy: Knowledge) 43) Dehydration can result from A) heat exhaustion. B) CCK. C) diabetes. D) gluconeogenesis. E) heat exhaustion and diabetes.

: E Section Title: Homeostatic Control of Metabolism Learning Outcome: 22.15

: Level I: Reviewing Facts and Terms (Bloom's Taxonomy: Knowledge) 94) After ingestion of carbohydrates in an insulin-deficient diabetic, ________ would happen. A) protein degradation B) fat synthesis C) ketone production D) glycolysis E) protein degradation and ketone production

: E Section Title: Homeostatic Control of Metabolism Learning Outcome: 22.15

: Level II: Reviewing Concepts (Bloom's Taxonomy: Comprehension) 101) Which of the following symptoms would you expect to observe in a person suffering from type I diabetes mellitus? A) glucosuria B) ketoacidosis C) thirst and polydipsia D) hyperglycemia E) All of these answers are correct.

: E Section Title: Homeostatic Control of Metabolism Learning Outcome: 22.15

A. gastrin B. cholecystokinin C. secretin D. motilin E. gastric inhibitory peptide 105) stimulates insulin release

: E Section Title: Integrated Function: The Cephalic Phase Learning Outcome: 21.11

: Level I: Reviewing Facts and Terms (Bloom's Taxonomy: Knowledge) 12) Anabolism is a term that describes A) reactions that release energy. B) reactions that require a net input of energy. C) reactions that result in the synthesis of large biomolecules. D) reactions that release energy and reactions that result in the synthesis of large biomolecules. E) reactions that require a net input of energy and reactions that result in the synthesis of large biomolecules.

: E Section Title: Metabolism Learning Outcome: 22.4

: Level I: Reviewing Facts and Terms (Bloom's Taxonomy: Knowledge) 36) The brain can use ________ for energy. A) only glucose B) only fats C) only ketones D) only lactate E) both glucose and ketones

: E Section Title: Metabolism Learning Outcome: 22.7

: Level I: Reviewing Facts and Terms (Bloom's Taxonomy: Knowledge) 24) Powerful contractions that occur a few times each day in the colon are called A) segmentation. B) tonic contractions. C) phasic contractions. D) peristalsis. E) mass movements.

: E Section Title: Motility Learning Outcome: 21.3

: Level II: Reviewing Concepts (Bloom's Taxonomy: Comprehension) 108) Brown fat A) is found in infants. B) functions in nonshivering thermogenesis. C) contains a rich vascular supply. D) is innervated by the sympathetic nervous system. E) All of these answers are correct.

: E Section Title: Regulation of Body Temperature Learning Outcome: 22.16

: Level II: Reviewing Concepts (Bloom's Taxonomy: Comprehension) 107) Which of the following individuals would lose heat faster in a cold room? A) an adult man B) an adult woman C) an adolescent male D) a child E) a newborn infant

: E Section Title: Regulation of Body Temperature Learning Outcome: 22.16

: Level I: Reviewing Facts and Terms (Bloom's Taxonomy: Comprehension) A. enzymes B. HCl C. HCO3- D. mucus E. more than one of these 60) pancreatic cells

: E Section Title: Secretion Learning Outcome: 21.11

: Level I: Reviewing Facts and Terms (Bloom's Taxonomy: Comprehension) 41) Bicarbonate secretion A) neutralizes acid entering from the stomach into the duodenum. B) is secreted by apical Cl--HCO3- antiport. C) is not dependent on high levels of carbonic anhydrase to maintain bicarbonate production. D) is secreted by the acinar cells. E) neutralizes acid entering from the stomach into the duodenum and is secreted by apical Cl--HCO3- antiport.

: E Section Title: Secretion Learning Outcome: 21.5

: Level I: Reviewing Facts and Terms (Bloom's Taxonomy: Comprehension) 42) Saliva is A) slightly acidic, with a pH of 6-7. B) controlled by the autonomic nervous system. C) stimulated by sympathetic innervation. D) secreted from endocrine glands. E) slightly acidic, with a pH of 6-7 and controlled by the autonomic nervous system.

: E Section Title: Secretion Learning Outcome: 21.5

: Level I: Reviewing Facts and Terms (Bloom's Taxonomy: Comprehension) 34) Angiotensin I is converted to angiotensin II by enzymes primarily located in the A) kidneys. B) liver. C) heart. D) lungs. E) blood vessels.

: E Section Title: Sodium Balance and ECF Volume Learning Outcome: 20.8

: Level I: Reviewing Facts and Terms (Bloom's Taxonomy: Comprehension) 57) A rise in angiotensin II levels would result in A) elevated blood pressure. B) increased retention of sodium ions at the kidney. C) increased water retention. D) increased blood volume. E) all of these effects.

: E Section Title: Sodium Balance and ECF Volume Learning Outcome: 20.8

: Level I: Reviewing Facts and Terms (Bloom's Taxonomy: Comprehension) 23) Angiotensin II A) stimulates thirst only. B) causes widespread vasoconstriction throughout the body only. C) causes the synthesis and release of aldosterone from the adrenal cortex. D) stimulates thirst and causes widespread vasoconstriction throughout the body. E) stimulates thirst, causes widespread vasoconstriction throughout the body, and causes the synthesis and release of aldosterone from the adrenal cortex.

: E Section Title: Sodium Balance and ECF Volume Learning Outcome: 20.8

: Level I: Reviewing Facts and Terms (Bloom's Taxonomy: Comprehension) 32) Which of the following statements concerning the hormone atrial natriuretic peptide is FALSE? A) Atrial natriuretic peptide is produced by cells in the heart. B) Atrial natriuretic peptide promotes sodium loss at the kidneys. C) Atrial natriuretic peptide reduces the sensation of thirst. D) Atrial natriuretic peptide suppresses ADH secretion. E) Atrial natriuretic peptide increases aldosterone secretion.

: E Section Title: Sodium Balance and ECF Volume Learning Outcome: 20.9

86) What is the ENS, and when was it discovered? What is its significance, what is its nickname, and what are the characteristics that justify that nickname?

: ENS is the enteric nervous system, discovered over 100 years ago. The ENS controls reflexive peristalsis in the intestine, independent of the CNS. The ENS is nicknamed the "little brain" because it shares these characteristics with the actual brain: secretion of neurotransmitters and neuromodulators, and the presence of glial cells, a diffusion barrier, and integrating centers. Section Title: Anatomy of the Digestive System Learning Outcome: 21.7

: Level I: Reviewing Facts and Terms (Bloom's Taxonomy: Comprehension) 76) ________ digests terminal peptide bonds to release amino acids.

: Endopeptidase Section Title: Digestion and Absorption Learning Outcome: 21.8

: Level III: Problem Solving (Bloom's Taxonomy: Synthesis) 122) Damien, in training for the Boston marathon, finds his plasma cholesterol concentration is somewhat elevated. What other test results should Damien review before changing his training regime or diet?

: Exercise often increases the levels of HDL's, since that is how fats travel throughout the body in order to be mobilized and used. Thus, his total cholesterol may be high, but it is better to depend on his HDL levels and the ratio of HDLs to LDLs. Section Title: Fed-State Metabolism Learning Outcome: 22.7

116) Proteins must first be enzymatically degraded to single amino acids before entering the capillaries of the hepatic portal system. Is this true or false? What is the significance of this? Is absorption of carbohydrates and lipids restricted to monomers? Explain.

: False. Absorption of small peptides allows peptides to provoke food allergies. The transporters for peptides are also responsible for absorption of some drugs. Carbohydrates are digested to monosaccharides because there are no transporters for larger carbohydrates. Lipids are absorbed as cholesterol, fatty acids, and monoglycerides, primarily by simple diffusion because they are lipophilic. Section Title: Digestion and Absorption Learning Outcome: 21.10

: Level II: Reviewing Concepts (Bloom's Taxonomy: Comprehension) 68) Diagram the reactions and interactions of the renin-angiotensin-aldosterone pathway. What condition is the primary stimulus for its activation?

: Figure 20.10 diagrams these interactions. The primary stimulus is low blood pressure, detected in several ways. Section Title: Sodium Balance and ECF Volume Learning Outcome: 20.8

: Level II: Reviewing Concepts (Bloom's Taxonomy: Comprehension) 67) Draw a map that shows the renal compensation for acidosis. Draw another map for alkalosis.

: Figure 20.17 shows compensations for both acidosis and alkalosis. Section Title: Acid-Base Balance Learning Outcome: 20.17

: Level III: Problem Solving (Bloom's Taxonomy: Analysis) 131) Prior to the discovery of the role of insulin in metabolism and the development of insulin therapy to treat diabetes mellitus, a diagnosis of this disease was always fatal. What do untreated diabetics die from?

: Figure 22.20 in the chapter provides a good summary of the effects of untreated diabetes mellitus. Metabolic acidosis and circulatory failure from dehydration are the primary triggers of death. Section Title: Homeostatic Control of Metabolism Learning Outcome: 22.15

: Level I: Reviewing Facts and Terms (Bloom's Taxonomy: Comprehension) 68) ________ is intestinal gas produced by bacteria in the colon during the metabolism of undigestible carbohydrates.

: Flatus Section Title: Integrated Function: The Intestinal Phase Learning Outcome: 21.11

: Level I: Reviewing Facts and Terms (Bloom's Taxonomy: Knowledge) 76) ________ are unstable molecules that are formed during normal metabolism and are thought to contribute to aging, disease, and some cancers.

: Free radicals Section Title: Fasted-State Metabolism Learning Outcome: 22.8

: Level I: Reviewing Facts and Terms (Bloom's Taxonomy: Comprehension) 90) Name two functions of cholecystokinin.

: Functions include stimulation of pancreatic enzyme secretion, bicarbonate secretion, gallbladder contraction, sense of satiety, and inhibition of gastric emptying. See Table 21.1 in the chapter. Section Title: Regulation of GI Function Learning Outcome: 21.6

88) List and briefly describe the action of peptides in the GI tract.

: Generally, GI peptides stimulate or inhibit motility and secretion. The peptides classified as hormones include gastrin, cholecystokinin, secretin, GIP, motilin, and glucagon-like peptide I. Section Title: Regulation of GI Function Learning Outcome: 21.8

: Level II: Reviews Concepts (Bloom's Taxonomy: Application) 111) Explain why glucose or other simple sugars should NOT be the sole source of energy in our diets. What is the best approach to meeting energy needs?

: Given that glucose only meets energy rather than other essential nutrient needs, a person consuming only glucose would die of malnutrition. A mixture of fats, proteins, simple sugars, and complex carbohydrates optimizes the storage, nutrient, and caloric needs of the various organ systems. Glucose requires a relatively large amount of water for hydration and a large amount of space for storage. Glycogen is more compact, and fat is the most compact and efficient means of energy storage. Section Title: Energy Balance Learning Outcome: 22.6

: Level III: Problem Solving (Bloom's Taxonomy: Application) 75) "Glucose, glucose, every where, nor any speck to utilize" is a phrase made up by one of the authors working on this test question. Similar to the irony of not being able to prevent dehydration by drinking seawater, people with untreated diabetes mellitus are unable to prevent starvation despite the large amount of glucose surrounding their cells; as if that isn't bad enough, dehydration is also a problem. Explain why there is glucose in the urine of such people, why glucose is not present in the urine of normal people, and why diabetics become dehydrated.

: Glucose is in the urine simply because the plasma concentration exceeds the number of glucose transporters in the kidney tubules, because plasma glucose is unusually high. Normal individuals are able to reabsorb all the glucose in the filtrate because plasma glucose concentrations are normally low (normal body cells absorb glucose, diabetic cells do not). Glucose in the filtrate raises the osmolarity of the filtrate, which decreases the relative osmotic gradient for reabsorption of water; thus more water is lost in the urine leading to dehydration. Section Title: Water Balance Learning Outcome: 20.12

: Level I: Reviewing Facts and Terms (Bloom's Taxonomy: Knowledge) 86) ________ is a test used to measure insulin response.

: Glucose tolerance test Section Title: Homeostatic Control of Metabolism Learning Outcome: 22.14

: Level III: Problem Solving (Bloom's Taxonomy: Application) 81) Gossip, an undergraduate, has normal PCO2 levels, high H+ levels, low pH and bicarbonate levels. What type of disturbance is Gossip suffering from and what might cause this? If his PCO2 were elevated, would your answer change? Explain.

: Gossip is likely suffering from metabolic acidosis, since his PCO2 is normal. If his PCO2 was elevated, then he would likely be suffering from respiratory acidosis. Metabolic acidosis may be caused by ingestion of methanol, aspirin, and ethylene glycol. Respiratory acidosis can be caused by drugs or alcohol depression, increased resistance in asthma, impaired gas exchange in fibrosis or severe pneumonia, and muscle weakness in muscular dystrophy and other muscle diseases. The most common cause is chronic obstructive pulmonary disease. Section Title: Acid-Base Balance Learning Outcome: 20.18

: Level I: Reviewing Facts and Terms (Bloom's Taxonomy: Knowledge) 73) The lipoprotein most likely to be called "good (or healthy) cholesterol" is ________.

: HDL Section Title: Fed-State Metabolism Learning Outcome: 22.5

127) A condition known as lactose intolerance is characterized by painful abdominal cramping, gas, and diarrhea. The cause of the problem is an inability to digest the milk sugar, lactose. How would this cause the observed symptoms? Why is yogurt less likely to trigger the symptoms?

: If an individual cannot digest lactose, then the sugar will pass through to the large intestine in an undigested form. The presence of the extra sugar in the chyme increases the osmolarity of the chyme, resulting in less water being reabsorbed by the intestinal mucosa. The bacteria that inhabit the large intestine can metabolize the lactose, and in the process they produce large amounts of carbon dioxide. The gas overstretches the intestine, which stimulates local reflexes that increase peristalsis. The combination of more fluid contents and increased peristalsis produces the symptom of diarrhea. The overexpansion of the intestine by gas causes the severe pain and abdominal cramping, and of course, the increase in intestinal gas release is directly related to increased gas production by the bacteria. Yogurt contains bacteria that digest the lactose prior to the consumption of the yogurt. Section Title: Digestion and Absorption Learning Outcome: 21.10

: Level II: Reviewing Concepts (Bloom's Taxonomy: Application) 116) Why does insulin deficiency lead to metabolic acidosis?

: Insulin deficiency produces a fasted-state metabolism. The liver metabolizes fatty acids for its energy needs, producing ketones. The ketones enter the blood, then are absorbed by tissues and converted to acetyl CoA. Ketones themselves are strong acids, thus ketoacidosis results. Other tissues begin to produce lactic acid, due to their anaerobic state resulting from loss of access to glucose. Section Title: Homeostatic Control of Metabolism Learning Outcome: 22.15

: Level III: Problem Solving (Bloom's Taxonomy: Application) 120) Jill suffers from anorexia nervosa. One afternoon she is rushed to the emergency room because of cardiac arrhythmias. Her breath has the smell of an aromatic hydrocarbon (sweet, fruity odor), and blood and urine samples indicated ketonemia and ketonuria. Why do you think she is having the arrhythmias?

: It appears that Jill is suffering from ketoacidosis as a consequence of her anorexia. Because she is literally starving herself, her body is metabolizing large amounts of fatty acids and amino acids to provide energy and in the process is producing large quantities of ketone bodies (normal metabolites from these catabolic processes). One of the ketones that is formed is acetone, which can be eliminated through the lungs. This accounts for the smell of aromatic hydrocarbons on Jill's breath. The ketones are also converted into keto acids such as acetic acid. In large amounts this lowers the body's pH. As the kidneys excrete excess H+, they reabsorb K+, leading to a state of hyperkalemia. This is probably the cause of her arrhythmias. See the Running Problem throughout the chapter, including its conclusion. Section Title: Homeostatic Control of Metabolism Learning Outcome: 22.15

: Level I: Reviewing Facts and Terms (Bloom's Taxonomy: Comprehension) 56) How do kidneys alter urine concentration?

: Kidneys alter urine concentration by varying the amounts of water and sodium reabsorbed in the distal nephron. Section Title: Water Balance Learning Outcome: 20.12

: Level III: Problem Solving (Bloom's Taxonomy: Application) 125) Low leptin levels have been observed in athletes with chronic negative energy balance. What might be a possible explanation for this? Would you expect neuropeptide Y (NPY) levels to be high? Explain.

: Leptin is believed to provide negative feedback to the hypothalamus and result in decreased food intake. Therefore, in a state where food intake is too low, it would make sense that leptin levels would be low. NPY, however, appears to be a stimulus for food intake. Therefore, in these athletes, we would expect NPY to be high. Section Title: Appetite and Satiety Learning Outcome: 22.1

: Level III: Problem Solving (Bloom's Taxonomy: Application) 127) High protein (low carbohydrate) diets have become popular in recent years. Based on what you've learned about metabolism and the body's energy needs, explain what might be some of the disadvantages of this type of diet.

: Low carbohydrate diets shift the body to a fasting mode because glucose is needed for the brain, central nervous systems, and red blood cells. After glycogen stores have been depleted, the body will convert protein and fats into glucose and ketone bodies to provide the energy needed. The disadvantages of this shift are excess ketone bodies can seriously disrupt the body's pH balance (lead to ketosis) and loss of protein from body tissue (this occurs even when abundant protein from food is provided). Section Title: Fasted-State Metabolism Learning Outcome: 22.10

119) Draw a map of the enteric nervous system response and include: stimuli, receptors, integrating centers, efferent path, and tissue response.

: Maps will vary. See Figure 21.11 in the chapter. Section Title: Regulation of GI Function Learning Outcome: 21.7

121) Make a map of the following terms: amino acid chief cell D cell ECL cell enteric sensory neuron enteric plexus G Cell gastrin H+ histamine parietal cell pepsin pepsinogen somatostatin vagus nerve Where appropriate, indicate stimulatory or inhibitory effect. Terms may be added as needed.

: Maps will vary. See Figure 21.16 in the chapter. Section Title: Integrated Function: The Gastric Phase Learning Outcome: 21.11

: Level I: Reviewing Facts and Terms (Bloom's Taxonomy: Comprehension) 71) ________ are tiny droplets of fatty acids, monoglycerides, and bile salts.

: Micelles Section Title: Digestion and Absorption Learning Outcome: 21.10

: Level III: Problem Solving (Bloom's Taxonomy: Analysis) 74) What are oropharynx receptors, which hormone do they suppress, and how is it known that they exist? If a person stranded on a desert island drank seawater to try to quench his thirst, how would this affect the oropharynx receptors?

: Oropharynx receptors inhibit the sense of thirst when they come in contact with cool water. They also inhibit secretion of vasopressin (ADH). They have not been identified anatomically, but presence of even a small amount of cold water in the throat is known to satisfy thirst even if the water is not absorbed and thus does not actually alleviate dehydration. Presence of receptors that inhibit the thirst sensation are the best guess as to how this works. Presumably drinking seawater would temporarily reduce the sense of thirst by the same proposed mechanism. Section Title: Behavioral Mechanisms in Salt and Water Balance Learning Outcome: 20.11

89) List the six types of epithelial cells associated with gastric glands. Briefly describe each one's role in the stomach.

: Parietal cells secrete HCl and intrinsic factor; chief cells secrete pepsinogen; D cells secrete somatostatin; ECL cells secrete histamine; G cells secrete gastrin; mucous cells secrete mucus and bicarbonate. Section Title: Secretion Learning Outcome: 21.5

: Level II: Reviewing Concepts (Bloom's Taxonomy: Application) 71) The cellular mechanisms for renal handling of H+ and HCO3- involve several membrane transporters; name three.

: Possible answers include: 1. apical Na+-H+ exchanger 2. basolateral Na+- HCO3- symport 3. H+- ATPase 4. H+- K+- ATPase 5. Na+- NH4+ antiport Section Title: Acid-Base Balance Learning Outcome: 20.15

122) Name the proenzymes that the pancreas secretes. Which one is involved in the activation of the other proenzymes? What are their active forms?

: Proenzymes include: trypsinogen, chymotrypsinogen, procarboxypeptidase, procolipase, and prophospholipase. Trypsinogen is converted to trypsin, which activates the other proenzymes. The active forms of the enzymes can be found in Figure 21.17 in the chapter. Section Title: Digestion and Absorption Learning Outcome: 21.11

: Level II: Reviewing Concepts (Bloom's Taxonomy: Comprehension) 110) In the study by Morewedge et al., subjects that imagined eating 30 M&M's one at a time ate fewer real M&M's than subjects who thought about eating only 3 M&M's. What possible reason can you think of to explain this observation?

: Psychological factors, such as "imagined" satisfaction from pretending to eat the candy, (or imagined guilt from all the calories and sugar that would have been consumed). Physiological factors may include signals released by the cephalic phase of digestion, just thinking about food, which could then affect the body's perceived input. (Chapter 21) Section Title: Appetite and Satiety Learning Outcome: 22.1

87) What are short reflexes? What types of behaviors do they regulate? What are long reflexes? Give examples of each.

: Reflexes controlled by the enteric nervous system are short reflexes. They regulate motility, secretion, and growth. Long reflexes are digestive reflexes that are integrated in the CNS. Long reflexes include feedforward reflexes and emotional reflexes. Section Title: Regulation of GI Function Learning Outcome: 21.6

: Level III: Problem Solving (Bloom's Taxonomy: Analysis) 82) Diabetes mellitus produces many homeostatic imbalances, including acidosis. The pH imbalance is due to ketoacidosis, which results from excessive accumulation of by-products of fat metabolism, as the body cannot meet energy needs from carbohydrate metabolism. Sally is a teenaged diabetic who sometimes rebels by not taking her insulin. Her mother takes her to the hospital because her breathing has become deep and gasping. Explain Sally's breathing pattern. What other compensatory responses may occur, and would they occur earlier or later than the respiratory response?

: Sally's respiratory system is responding to her ketoacidosis. By hyperventilating, she removes more CO2 from her blood, which shifts the carbonic acid equation to the right, removing free H+ and raising pH. Chemical buffering in the blood occurs immediately after the initial pH disturbance, as this is an ongoing process. The kidneys buffer more slowly, so they are effective later. Section Title: Acid-Base Balance Learning Outcome: 20.18

: Level III: Problem Solving (Bloom's Taxonomy: Application) 123) Sandy has insulin-dependent diabetes mellitus. Sandy has an accident on his bike and breaks his arm. Would his insulin dosage change?

: Sandy will need increased levels of insulin. Insulin will promote the necessary healing by supporting anabolic reactions needed for regrowth of the bone. Students will learn in chapter 23 that prolonged stress (rather than the stress just at the time of the accident) will lead to increased blood glucose levels due to increased cortisol secretion, hence additional insulin will be needed to compensate for this as well. Section Title: Homeostatic Control of Metabolism Learning Outcome: 22.12

123) The enteric nervous system and the brain have several similarities. What are they?

: See "The Enteric Nervous System Can Act Independently" section of the chapter. Section Title: Regulation of GI Function Learning Outcome: 21.7

124) Draw a cell that secretes HCl into the lumen of the stomach, selecting transporters from the following list (you may not need them all): chloride channel CFTR channel GLUT4 H+-K+-ATPase Cl- - HCO3- Antiport Cl- - HCO3- Symport Na+-K+-ATPase Label the cell showing the lumen, interstitial fluid, the basolateral membrane, and the apical membrane.

: See Figure 21.5 in the chapter. Section Title: Secretion Learning Outcome: 21.5

: Level II: Reviewing Concepts (Bloom's Taxonomy: Application) 117) Draw a cell that secretes bicarbonate into the lumen of the pancreas, selecting transporters from the following list (you may not need them all): chloride channel CFTR channel GLUT4 H+-K+-ATPase Cl- - HCO3- Antiport Cl- - HCO3- Symport K+ leak channel leaky junctions Na+-K+-ATPase Label the cell showing the lumen, interstitial fluid, the basolateral membrane, and the apical membrane.

: See Figure 21.5 in the chapter. Section Title: Secretion Learning Outcome: 21.5

112) What are the end products of carbohydrate digestion? What are the enzymes involved; where are they located; and what reactions do they catalyze?

: See Figure 21.8 in the chapter. Amylases are secreted into the mouth and small intestine, and disaccharidases are located on the intestinal brush border. Section Title: Digestion and Absorption Learning Outcome: 21.10

120) Diagram peptide absorption from the lumen into the portal vein. Be sure to include transporters and label the lumen, portal vein, apical membrane, and basolateral membrane.

: See Figure 21.8d in the chapter. Section Title: Integrated Function: The Intestinal Phase Learning Outcome: 21.11

: Level III: Problem Solving (Bloom's Taxonomy: Analysis) 80) How does decreased blood pressure affect the following: granular cells, glomerulus, cardiovascular control center, hypothalamus? Indicate if decreased blood pressure directly affects the organ or tissue above or acts through a reflex pathway. If it acts through a reflex pathway, name the reflex.

: See Table 20.1 in the chapter. Section Title: Integrated Control of Volume and Osmolarity Learning Outcome: 20.7

118) Compare and contrast gastrin, CCK, secretin, GIP, motilin, and glucagon-like peptide 1. In your answer, include the following details for each term: secreted by, target(s), effects, and stimulus for release.

: See Table 21.1 in the chapter. Section Title: Integrated Function: The Cephalic Phase Learning Outcome: 21.11

: Level I: Reviewing Facts and Terms (Bloom's Taxonomy: Knowledge) 74) ________ are short segments of intestine that alternately contract and relax. They are responsible for ________.

: Segmental contractions; mixing Section Title: Motility Learning Outcome: 21.4

: Level III: Problem Solving (Bloom's Taxonomy: Analysis) 132) Misty is 16 and, with her family, is visiting her grandmother for Christmas. At the end of a week surrounded by tempting holiday food, Misty weighs herself and sees that she has gained five pounds. She confides to her mother that she ate all of a 1-lb box of chocolates on Christmas day, but didn't overeat in any other way. There are 500 kilocalories per 100 g of chocolate. 1 lb = 454 g. There are 3500 kilocalories in a pound of fat. How many calories did she consume? Does this account for the five pounds she gained? Explain.

: She consumed 454 g of chocolates. 454 g × 500 kcal/100g = 2270 kcal. The chocolates cannot account for more than about 2/3 of a pound. She must have consumed extra calories at her meals and other snacks as well, and she may have exercised less than normal. Section Title: Energy Balance Learning Outcome: 22.1

126) Cody has always carefully controlled her caloric intake, and is lean. During her unplanned pregnancy, she continued to eat the same, hoping that taking prenatal vitamins and minerals would ensure that her fetus would grow normally. Her only complication during pregnancy was persistent constipation and consequent hemorrhoids. She gained only the minimum weight suggested by her doctor, and delivered a healthy, full-term infant; her pre-pregnancy weight was rapidly reestablished. Propose an explanation for how she managed to gain enough weight, did not violate the law of conservation of mass, and for her only complication. What does this suggest about "eating for two" that some pregnant women do?

: She must have inefficiently absorbed calories prior to her pregnancy, losing many potential calories in her feces. When she became pregnant, one of her physiological changes included an increase in nutrient (and thus calorie) absorption. This would also contribute to her constipation, as her feces retained less water than normal and thus were more difficult to eliminate. It is not possible for her to have gained more weight than she ate; no one can violate the law of conservation of mass. Most obstetricians recommend only a slight increase in calories during pregnancy for this reason, and women who indulge in overeating too often are likely to gain unnecessary body fat and may produce a fetus that is larger than it should be. Section Title: Integrated Function: The Intestinal Phase Learning Outcome: 21.10

: Level II: Reviewing Concepts (Bloom's Taxonomy: Comprehension) 117) Explain how and why abnormal temperatures, either too high or too low, can be dangerous.

: The body's responses to high and low temperature are summarized in Figures 22.21 and 22.22 in the chapter and discussed in the section "Hyperthermia Includes Heat Exhaustion and Heat Stroke." Complications include severe dehydration, loss of enzyme function, and death. Hypothermia is associated with slowed metabolism and loss of consciousness, and death. Section Title: Regulation of Body Temperature Learning Outcome: 22.16

: Level IV: Quantitative Problems (Bloom's Taxonomy: Application) 84) The graph below has had its labels removed. One axis is plasma osmolarity. The other axis is EITHER plasma vasopressin concentration OR plasma aldosterone concentration. Write the correct label on each axis of the graph, and explain how you came to that conclusion.

: The dependent variable is typically used for the y-axis, and the independent variable for the x-axis. Plasma osmolarity is the independent variable, and it regulates the secretion of hormones. As osmolarity increases, vasopressin secretion increases to promote water retention, but aldosterone secretion is inhibited to promote sodium loss. Therefore vasopressin concentration is the correct label for the y-axis. See Figure 20.6 in the chapter. Section Title: Water Balance Learning Outcome: 20.3

: Level II: Reviewing Concepts (Bloom's Taxonomy: Knowledge) 119) The weather person gleefully tells us that the temperature is 95, but the heat index is 103 and a heat advisory is in effect. What does this mean? Is humidity high or low? Why does it matter? Who is most affected, and why? What are some steps one could take to adjust?

: The humidity is high, thus slowing evaporative heat loss; the heat advisory means take it slow and suggests you plan your day around the heat and try to get out of it. Elderly people are most likely to be affected physiologically and possibly economically and sociologically as well. Use fans and/or air conditioning, swim, take advantage of a mall or movie theater's air conditioning, decrease activity, do outdoor work only in the early morning or late evening. Section Title: Regulation of Body Temperature Learning Outcome: 22.16

: Level II: Reviewing Concepts (Bloom's Taxonomy: Comprehension) 109) List and discuss two different theories about the regulation of food intake and the role of signal molecules and the brain.

: The hypothalamus contains two centers that regulate food intake: the feeding center and the satiety center, which inhibits the feeding center. The glucostatic theory states that glucose utilization by hypothalamic centers regulates food intake. The lipostatic theory states that signals from the body's fat stores to the brain modulate eating behavior to maintain a particular weight. Leptin is a protein hormone synthesized in adipocytes and may signal the brain to halt feeding behavior. Neuropeptide Y is a brain neurotransmitter that appears to be the stimulus for food intake; this molecule is inhibited by leptin. Ghrelin is a peptide secreted by the stomach that stimulates hunger. Table 22.1 in the chapter summarizes peptides involved in feeding. Section Title: Appetite and Satiety Learning Outcome: 22.1

: Level II: Reviewing Concepts (Bloom's Taxonomy: Application) 73) "Water, water, every where, nor any drop to drink" is a phrase from the Rime of the Ancient Mariner by Samuel Taylor Coleridge. This poem describes an ocean ship unable to sail to land and running out of its supply of fresh water; sailors have long known that drinking seawater cannot prevent death from dehydration. What would result if the sailors attempted to alleviate their dehydration by drinking seawater? Justify your answer by describing kidney physiology. Why wouldn't reflexes, in response to dehydration, fully compensate? What does this illustrate about the force allowing kidneys to retain water under more normal conditions?

: The osmolarity of seawater is higher than that of the kidney medulla, thus the osmotic gradient that normally allows net reabsorption of water does not exist. Seawater osmolarity is largely a result of permeant ions such as sodium and chloride, so ingested seawater raises plasma osmolarity when it is absorbed. When the kidneys form a filtrate of this high-osmolarity solution, the descending limb fails to reabsorb water because the kidney medulla has a lower osmolarity than the filtrate (opposite of normal), along its entire length, and in fact the medulla would lose water as it moved into the descending limb. While ions would be reabsorbed by the ascending limb as usual, it is not enough to drive sufficient water reabsorption. Reflexes such as secretion of water-preserving hormones are ineffective because, ultimately, water reabsorption depends only on osmosis, and the normal osmotic gradient has been disrupted. Section Title: Sodium Balance and ECF Volume Learning Outcome: 20.12

: Level III: Problem Solving (Bloom's Taxonomy: Application) 129) There is only one way to lose weight without actually removing a body part: consume fewer calories than you expend. Is this statement true or false? Explain, citing a fundamental law of physics. Why is it difficult for many people to lose weight? Many weight-loss gimmicks work. Explain why they work, and why they may not work forever.

: The statement is true, and is a principle of physics known as the first law of thermodynamics. It is difficult for many people to lose weight because of a variety of factors, such as overeating due to psychological factors, sedentary lifestyles, and so on. Weight-loss gimmicks work because ultimately they either reduce calories or increase activity, or both. They may be difficult to maintain because the novelty wears off, people feel deprived of food, and a multitude of other factors. Answers to this question will vary considerably. Section Title: Energy Balance Learning Outcome: 22.1

130) In some severe cases, a person suffering from stomach ulcers may have surgery to cut the branches of the vagus nerve that innervates the stomach. How would this help the problem?

: The vagus nerve carries neurons of the parasympathetic nervous system. These neurons control gastric secretions, notably the secretion of acid and enzymes. Severing these branches would eliminate neural stimulation from the central nervous system, thus eliminating the release of gastric fluids in response to anxiety and other higher-order stimuli when there is no food in the stomach. Normal digestive function would still occur, governed by various hormones and intramural neural reflexes. Section Title: Regulation of GI Function Learning Outcome: 21.6

133) You and your lab partners in a human anatomy course have just removed the intestines from an adult cadaver, and cut away connective tissues as necessary to uncoil the intestines into a straight line that could be measured. Your professor told you that the intestinal tract of an adult of this size and gender is typically about 13 ft. Your measurement is 22 ft. One of the lab partners insists that everything is smaller in preserved cadavers due to dehydration. Address this suggestion, and propose some other explanations for the difference.

: There is no doubt of some individual variation, so the measurement is expected to be at least a little different. Most of the difference is due to loss of living muscle tone, which normally keeps the tract contracted. While it is true that preserved tissues may shrink from dehydration, this is more than compensated by the loss of muscle tone. Section Title: Anatomy of the Digestive System Learning Outcome: 21.1

: Level I: Reviewing Facts and Terms (Bloom's Taxonomy: Knowledge) 88) ________ monitor skin temperature and core body temperature.

: Thermoreceptors Section Title: Regulation of Body Temperature Learning Outcome: 22.16

: Level I: Reviewing Facts and Terms (Bloom's Taxonomy: Knowledge) 83) ________ is a condition of insulin deficiency from beta cell destruction.

: Type 1 diabetes mellitus Section Title: Homeostatic Control of Metabolism Learning Outcome: 22.14

: Level I: Reviewing Facts and Terms (Bloom's Taxonomy: Knowledge) 84) ________ is known as insulin-resistant diabetes.

: Type 2 diabetes Section Title: Homeostatic Control of Metabolism Learning Outcome: 22.14

: Level II: Reviewing Concepts (Bloom's Taxonomy: Application) 114) Compare and contrast the two types of diabetes mellitus.

: Type I diabetes mellitus is a condition of insulin deficiency due to pancreatic beta cell destruction. Type 2 diabetes mellitus is insulin-resistant, due to a variety of causes. Figure 22.20 in the chapter provides a detailed summary of the characteristics of type I diabetes. Section Title: Homeostatic Control of Metabolism Learning Outcome: 22.14

: Level II: Reviewing Concepts (Bloom's Taxonomy: Comprehension) 113) It is known that exercise is good for diabetics. Explain how GLUT4 transporters may be involved in this beneficial effect of exercise.

: When one exercises, skeletal muscles are utilized. Once they are active, they no longer require insulin to be able to take up glucose from the blood. This is due to the fact that exercise causes more GLUT4 transporters to be inserted into the membrane of the muscle cells. Thus, exercise is able to remove the excess glucose from the bloodstream of diabetics. Section Title: Homeostatic Control of Metabolism Learning Outcome: 22.15

: Level III: Problem Solving (Bloom's Taxonomy: Analysis) 78) Mary, a nursing student, has been caring for burn patients. She notices that they consistently show elevated levels of potassium in their urine and wonders why. What would you tell her?

: When tissues are burned, cells are destroyed and the contents of their cytoplasm leak into the interstitial fluid and then move into the plasma. Since potassium ion is normally found within the cell, damage to a large number of cells would release relatively large amounts of potassium into the blood. The elevated potassium levels would stimulate the cells of the adrenal cortex that produce aldosterone. The elevated levels of aldosterone would promote sodium retention and potassium secretion by the kidneys, thus accounting for the elevated levels of potassium in the patient's urine. Section Title: Potassium Balance Learning Outcome: 20.10

129) Eric has had trouble sleeping the last two nights and is currently experiencing heart burn which he attributes to the stress of being in finals week and eating too much pizza. His lab partner in Physiology class tells him he should take some diphenhydramine (Benadryl) before he goes to bed because that will help him sleep and his upset stomach. Is this a good suggestion or not.

: While longstanding heartburn or insomnia may both be symptoms of more severe underlying conditions that require a visit to a health professional, his lab partner is correct. Diphenhydramine is an antihistamine that is found in many over-the-counter sleep medications and it will block the effects of histamine in the stomach preventing acid production. Section Title: Secretion Learning Outcome: 21.5

: Level I: Reviewing Facts and Terms (Bloom's Taxonomy: Comprehension) 75) The exocrine portion of the pancreas consists of lobules called ________, which secrete ________.

: acini, digestive enzymes Section Title: Secretion Learning Outcome: 21.11

: Level I: Reviewing Facts and Terms (Bloom's Taxonomy: Knowledge) 47) When the pH rises above 7.45, a state of ________ exists.

: alkalosis Section Title: Acid-Base Balance Learning Outcome: 20.14

: Level I: Reviewing Facts and Terms (Bloom's Taxonomy: Comprehension) 69) The enzyme that digests starch into disaccharides and trisaccharides is ________.

: amylase Section Title: Secretion Learning Outcome: 21.11

: Level I: Reviewing Facts and Terms (Bloom's Taxonomy: Knowledge) 68) The ________ represents the minimum resting energy expenditures of an awake, alert individual.

: basal metabolic rate (or BMR) Section Title: Energy Balance Learning Outcome: 22.2

: Level I: Reviewing Facts and Terms (Bloom's Taxonomy: Comprehension) 45) A(n) ________ consists of a combination of a weak acid and its dissociation products.

: buffer system Section Title: Acid-Base Balance Learning Outcome: 20.15

: Level I: Reviewing Facts and Terms (Bloom's Taxonomy: Comprehension) 49) The enzyme that catalyzes the conversion of H2O and CO2 to H2CO3 is called ________.

: carbonic anhydrase Section Title: Acid-Base Balance Learning Outcome: 20.15

: Level I: Reviewing Facts and Terms (Bloom's Taxonomy: Knowledge) 71) Metabolism is often divided into ________, energy-producing reactions, and ________, energy-utilizing reactions.

: catabolism, anabolism Section Title: Metabolism Learning Outcome: 22.4

: Level I: Reviewing Facts and Terms (Bloom's Taxonomy: Comprehension) 66) After processing in the stomach, the gastric contents are referred to as ________.

: chyme Section Title: Anatomy of the Digestive System Learning Outcome: 21.11

: Level I: Reviewing Facts and Terms (Bloom's Taxonomy: Comprehension) 53) The anatomical arrangement of the kidney that allows transfer of solutes from one blood vessel to another is called the ________.

: countercurrent exchange system Section Title: Water Balance Learning Outcome: 20.4

85) Defects in ________ structure or function lead to the disease cystic fibrosis. What ion does this channel transport?

: cystic fibrosis transmembrane regulator (or CFTR chloride channel); it transports chloride. Section Title: Secretion Learning Outcome: 21.5

: Level I: Reviewing Facts and Terms (Bloom's Taxonomy: Knowledge) 67) The energy content of a food or similar substance is measured by a process called ________.

: direct calorimetry Section Title: Energy Balance Learning Outcome: 22.2

: Level I: Reviewing Facts and Terms (Bloom's Taxonomy: Comprehension) 50) Removal of excess water in urine is known as ________.

: diuresis Section Title: Water Balance Learning Outcome: 20.1

: Level I: Reviewing Facts and Terms (Bloom's Taxonomy: Comprehension) 63) The three sections of the small intestine, in order according to movement of its contents, are ________, ________, and ________.

: duodenum, jejunum, ileum Section Title: Anatomy of the Digestive System Learning Outcome: 21.1

: Level I: Reviewing Facts and Terms (Bloom's Taxonomy: Comprehension) 67) Bile salts aid in the digestion of fats by ________ large fat droplets.

: emulsifying Section Title: Secretion Learning Outcome: 21.10

: Level I: Reviewing Facts and Terms (Bloom's Taxonomy: Comprehension) 80) Short reflexes of the digestive system are integrated in the ________.

: enteric nervous system Section Title: Regulation of GI Function Learning Outcome: 21.6

: Level I: Reviewing Facts and Terms (Bloom's Taxonomy: Comprehension) 52) AQP2 water pores are added to the cell membrane by ________ and withdrawn by ________ in a process known as ________.

: exocytosis, endocytosis, membrane recycling Section Title: Water Balance Learning Outcome: 20.2

: Level I: Reviewing Facts and Terms (Bloom's Taxonomy: Knowledge) 79) Long reflexes that originate completely outside the digestive system include ________ and ________, which are called ________ reflexes.

: feedforward reflexes, emotional reflexes, cephalic Section Title: Regulation of GI Function Learning Outcome: 21.6

: Level II: Reviewing Concepts (Bloom's Taxonomy: Comprehension) 69) Water gain during a day comes from either ________ or ________. How is water lost during a day?

: food, drink; metabolism Also see Figure 20.2 in the chapter. Section Title: Water Balance Learning Outcome: 20.11

: Level I: Reviewing Facts and Terms (Bloom's Taxonomy: Knowledge) 81) The primary products of protein digestion are ________, ________, and ________.

: free amino acids, dipeptides, tripeptides Section Title: Digestion and Absorption Learning Outcome: 21.10

: Level I: Reviewing Facts and Terms (Bloom's Taxonomy: Knowledge) 79) The pancreatic hormone that raises blood glucose concentration is ________.

: glucagon Section Title: Homeostatic Control of Metabolism Learning Outcome: 22.13

: Level I: Reviewing Facts and Terms (Bloom's Taxonomy: Knowledge) 80) The process of synthesizing glucose from lipids, amino acids, or other carbohydrates is called ________.

: gluconeogenesis Section Title: Metabolism Learning Outcome: 22.4

: Level I: Reviewing Facts and Terms (Bloom's Taxonomy: Comprehension) 70) Maltose is broken down by maltase into two molecules of ________.

: glucose Section Title: Digestion and Absorption Learning Outcome: 21.11

: Level I: Reviewing Facts and Terms (Bloom's Taxonomy: Knowledge) 78) Lipids are broken down into ________, which feeds into glycolysis, and ________, which are metabolized to acetyl CoA.

: glycerol; fatty acids Section Title: Fasted-State Metabolism Learning Outcome: 22.10

: Level I: Reviewing Facts and Terms (Bloom's Taxonomy: Knowledge) 81) The process of glycogen formation is known as ________.

: glycogenesis Section Title: Metabolism Learning Outcome: 22.4

: Level I: Reviewing Facts and Terms (Bloom's Taxonomy: Knowledge) 64) Most absorbed nutrients first enter the blood of the ________ system.

: hepatic portal Section Title: Integrated Function: The Intestinal Phase Learning Outcome: 21.11

: Level I: Reviewing Facts and Terms (Bloom's Taxonomy: Knowledge) 69) Measuring oxygen consumption as a way of estimating a person's metabolic rate is a type of ________.

: indirect calorimetry Section Title: Energy Balance Learning Outcome: 22.2

: Level I: Reviewing Facts and Terms (Bloom's Taxonomy: Comprehension) 55) The ________ cells of the distal nephron are interspersed among the principal cells and contribute to acid-base regulation.

: intercalated Section Title: Acid-Base Balance Learning Outcome: 20.17

: Level I: Reviewing Facts and Terms (Bloom's Taxonomy: Comprehension) 72) Slow waves originate in modified smooth muscle cells called ________.

: interstitial cells of Cajal Section Title: Motility Learning Outcome: 21.3

: Level I: Reviewing Facts and Terms (Bloom's Taxonomy: Comprehension) 48) Abnormal fat and amino acid metabolism may lead to the condition called ________.

: ketoacidosis Section Title: Acid-Base Balance Learning Outcome: 20.18

: Level I: Reviewing Facts and Terms (Bloom's Taxonomy: Comprehension) 82) Fat digestion is carried out by the enzyme ________, which breaks down triglyceride into ________.

: lipase, two fatty acids and one monoglyceride Section Title: Digestion and Absorption Learning Outcome: 21.10

: Level I: Reviewing Facts and Terms (Bloom's Taxonomy: Knowledge) 72) The enzyme ________ converts triglycerides into free fatty acids and monoglycerides.

: lipoprotein lipase Section Title: Fed-State Metabolism Learning Outcome: 22.5

: Level I: Reviewing Facts and Terms (Bloom's Taxonomy: Comprehension) 77) Bile is produced in the ________ and stored in the ________.

: liver, gallbladder Section Title: Digestion and Absorption Learning Outcome: 21.10

: Level I: Reviewing Facts and Terms (Bloom's Taxonomy: Knowledge) 78) Digestive reflexes originating in the CNS are called ________.

: long reflexes Section Title: Regulation of GI Function Learning Outcome: 21.6

: Level I: Reviewing Facts and Terms (Bloom's Taxonomy: Knowledge) 75) Lipoproteins that contain large amounts of cholesterol for transport to peripheral tissues are called ________.

: low-density lipoproteins (LDLs) Section Title: Fed-State Metabolism Learning Outcome: 22.5

: Level I: Reviewing Facts and Terms (Bloom's Taxonomy: Comprehension) 61) The largest collection of ________ tissue in the body is the gut-associated lymphoid tissue (GALT).

: lymphoid Section Title: Immune Functions of the GI Tract Learning Outcome: 21.13

: Level I: Reviewing Facts and Terms (Bloom's Taxonomy: Comprehension) 54) Paracrine feedback from the ________ in the distal tubule to the granular cells stimulates release of ________.

: macula densa, renin Section Title: Sodium Balance and ECF Volume Learning Outcome: 20.8

: Level I: Reviewing Facts and Terms (Bloom's Taxonomy: Knowledge) 91) A genetic condition in which body temperature becomes abnormally elevated is ________.

: malignant hyperthermia Section Title: Regulation of Body Temperature Learning Outcome: 22.16

: Level I: Reviewing Facts and Terms (Bloom's Taxonomy: Comprehension) 51) The high ________ osmolarity allows urine to be concentrated.

: medullary interstitial Section Title: Water Balance Learning Outcome: 20.4

: Level I: Reviewing Facts and Terms (Bloom's Taxonomy: Knowledge) 87) The combination of type 2 diabetes, atherosclerosis, and high blood pressure is called ________.

: metabolic syndrome Section Title: Homeostatic Control of Metabolism Learning Outcome: 22.14

: Level I: Reviewing Facts and Terms (Bloom's Taxonomy: Knowledge) 73) The ________ is a "housekeeping function" that sweeps food remnants and bacteria out of the upper GI tract and into the large intestine.

: migrating motor complex Section Title: Motility Learning Outcome: 21.4

: Level I: Reviewing Facts and Terms (Bloom's Taxonomy: Knowledge) 85) The loss of water in the urine due to unreabsorbed solutes is known as ________.

: osmotic diuresis Section Title: Homeostatic Control of Metabolism Learning Outcome: 22.15

: Level I: Reviewing Facts and Terms (Bloom's Taxonomy: Knowledge) 90) Chemicals known as ________ are fever-producing cytokines that are part of the normal immune response.

: pyrogens Section Title: Regulation of Body Temperature Learning Outcome: 22.16

: Level I: Reviewing Facts and Terms (Bloom's Taxonomy: Knowledge) 89) The production of heat from rhythmic tremors of skeletal muscle is referred to as ________.

: shivering thermogenesis Section Title: Regulation of Body Temperature Learning Outcome: 22.16

: Level I: Reviewing Facts and Terms (Bloom's Taxonomy: Knowledge) 62) Digestion is essentially completed in the ________.

: small intestine Section Title: Integrated Function: The Intestinal Phase Learning Outcome: 21.10

: Level I: Reviewing Facts and Terms (Bloom's Taxonomy: Comprehension) 44) The principal ions in the ECF are ________.

: sodium, chloride, and bicarbonate Section Title: Sodium Balance and ECF Volume Learning Outcome: 20.12

: Level I: Reviewing Facts and Terms (Bloom's Taxonomy: Comprehension) 65) The primary complex carbohydrate ingested and digested by people is ________.

: starch Section Title: Digestion and Absorption Learning Outcome: 21.10

In a normal kidney, which of the following conditions would cause an increase in the glomerular filtration rate (GFR)?

A decrease in the concentration of plasma proteins in the blood

How will a diuretic (a substance that will increase the volume of urine produced) help alleviate edema in arms and legs?

A diuretic will make you urinate. This will -lower the hydrostatic pressure- and the more you urinate, the less fluid will leak to the interstitial fluid which would equal less edema. or the diuretic will -increase ^ the osmotic pressure- making the solution hyperosmotic, and water flows toward hyperosmotic areas.

The blood tests came back negative for liver and pancreatic enzymes, but the X-ray confirmed gallstones filling her gallbladder and blocking the lower biliary duct near the hepatopancreatic ampulla (ampulla of Vater). The doctor mentioned that this might also affect pancreatic function, scheduled a cholecystectomy for the next day, and decided to admit her to the hospital overnight. Why might a gallstone affect the pancreas?

A gallstone that blocks the hepatopancreatic ampulla (ampulla of Vater) will block the release of digestive enzymes from the pancreas. If you look back at the anatomy, the pancreas and gallbladder both drain through the hepatopancreatic ampulla, or ampulla of Vater. If a gallstone washes out of the gallbladder and blocks this ampulla, digestive enzymes produced in the pancreas will not be able to drain out. Even worse, when the person eats fat and cholecystokinin is released, causing the gall bladder to contract, bile will be pushed down the cystic duct. Where can it go if the ampulla is blocked? It will go up the pancreatic duct, into the pancreas. There it can increase pressure in the ducts and contribute to activating digestive enzymes, which then start to digest the pancreas.

What is a nutrient pool? What are the three primary nutrient pools of the body?

A group of nutrients (glucose, free fatty acids, and amino acids), mostly in the blood, available for cell use.

Juxtaglomerular complex (JGC)

A group of specialized cells in the walls of the distal part of the ascending limb of the nephron loop and afferent arteriole close to the glomerulus; involved in blood pressure regulation and autoregulation of renal blood flow.

The bicarbonate buffer system is one of the chemical buffer systems of the body. How would the bicarbonate buffer system work if sodium hydroxide were added to a solution?

A hydrogen on carbonic acid would dissociate and join the hydroxyl group on the base to form water and sodium bicarbonate a strong base (sodium hydroxide) would be converted to a weak base (sodium bicarbonate).

D) lymphatic vessel : villus

A lacteal is a ________ located in ________. A) nerve : myenteric plexus B) nerve : submucosal plexus C) capillary : villus D) lymphatic vessel : villus E) patch of immune tissue : submucosa

Osmotic pressure

A measure of the tendency of a solvent to move into a more concentrated solution.

What is a buffer? Lit three intracellular buffers. Name the primary extracellular buffer.

A molecule that moderates changes in pH. Intracellular: proteins, Pi, and hemoglobin. Extracellular: HCO3-

Angiotensin Ⅱ

A peptide hormone that is part of the renin-angiotensin-aldosterone system for regulating blood pressure. It is produced from angiotensin Ⅰ by angiotensin converting enzyme. It is a potent vasoconstrictor. It also triggers the release of aldosterone and antidiuretic hormone.

Autoregulation

A process in which a tissue regulates aspects of its own structure or functioning. For example, mechanisms within the nephron maintain a relatively constant glomerular blood flow over a range of systemic pressures.

Passive transport

A process that does not require cellular energy to transport substance across the cell membrane.

Hypotonic solution

A solution that is more dilute (containing more nonpenetrating solutes) than a reference solution.

What is the normal glomerular filtration rate? A) 125 mL/min B) 625 mL/min C) 3 L/day D) 30 L/day E) 1 gallon/day

A) 125 mL/min

ACE inhibitors prevent angiotensin converting enzyme (ACE) from performing its role in the body. Which statement best describes why doctors prescribe ACE inhibitors? A) ACE inhibitors reduce blood pressure by blocking the conversion of angiotensin I to angiotensin II. B) ACE inhibitors increase blood pressure by blocking the RAS pathway. C) ACE inhibitors block the production of renin in the juxtaglomerular cells in the kidneys reducing fluid volume and, therefore, blood pressure. D) ACE inhibitors slow down the heart rate by not allowing atrial natriuretic peptide (ANP) to be released from the heart, decreasing sodium production and, therefore, blood pressure. E) ACE inhibitors block baroreceptors and reduce sympathetic activity to the cardiovascular system, thereby reducing blood pressure.

A) ACE inhibitors reduce blood pressure by blocking the conversion of angiotensin I to angiotensin II.

What hormone activates the cAMP second messenger system in cells of the distal tubules and collecting ducts? A) ADH B) aldosterone C) angiotensin II D) atrial natriuretic peptide E) parathyroid hormone

A) ADH

What hormone increases water reabsorption by inserting aquaporin-2 proteins into the apical membrane of principal cells? A) ADH B) aldosterone C) angiotensin II D) atrial natriuretic peptide E) parathyroid hormone

A) ADH

Some athletes, in order to gain an advantage over their competitors, participate in blood doping. This process increases the number of red blood cells in their cardiovascular systems and in turn delivers more oxygen to their metabolically active tissues. Which statement best describes how athletes' blood dope and the dangers associated with blood doping? A) Erythropoietin is injected stimulating more red blood cell production. An increase in red blood cells increases the hematocrit which can increase blood pressure and weaken vessels making the chances of stroke increase. B) Athletes receive blood transfusions to increase the number of red blood cells and by doing so increase their chances of developing aids or hepatitis C. C) Hemoglobin is injected, increasing the oxygen carrying capacity of red blood cells. However, too much hemoglobin can cause hemolysis. D) Leukocytes are injected because they are larger than red blood cells. However, they make the body more susceptible to infection. E) Colony-stimulating factors are injected to increase the number of red blood cells. Too many, however, can clog arteries, causing a heart attack

A) Erythropoietin is injected stimulating more red blood cell production. An increase in red blood cells increases the hematocrit which can increase blood pressure and weaken vessels making the chances of stroke increase.

Olympic athletes train in Colorado Springs, Colorado because this location is at high altitude and near the 14,000 foot Pikes Peak. Which statement best describes why this location was chosen? A) Oxygen levels are low and, therefore, more red blood cells are produced, giving the athletes an advantage when competing at lower altitudes. B) The concentration difference for getting air into the lungs is greater at high altitudes. C) The thinner air causes it to be easier to move into and out of the lungs, which leads to higher oxygen transport. D) The thin air allows for less resistance when biking or running and, therefore, the athletes can run and bike faster, which builds up their endurance for future races E) Oxygen levels are low and therefore fewer red blood cells are available, which puts more physiological stress on the body. This leads to a greater efficiency of getting oxygen to the tissues.

A) Oxygen levels are low and, therefore, more red blood cells are produced, giving the athletes an advantage when competing at lower altitudes.

Which of the following statements on sodium movement in the renal tubules is TRUE? A) Sodium is actively transported across the basolateral membrane of both the proximal and distal tubule. B) Sodium is actively secreted from the collecting duct. C) Sodium is actively secreted from the proximal tubule. D) Sodium reabsorption follows water reabsorption in all segments of the renal tubules. E) Sodium channels are located on the basolateral membrane of epithelial tubular cells.

A) Sodium is actively transported across the basolateral membrane of both the proximal and distal tubule.

Which statement best describes the function of albumins? A) They primarily affect the movement of fluid into capillaries by creating an osmotic gradient. B) They aid in the structural form of the erythrocyte. C) They provide the nutrients needed to replicate erythrocytes. D) They transport lipids, playing a role in clotting and defending the body against foreign substances. E) They are the key substance in the formation of blood clots.

A) They primarily affect the movement of fluid into capillaries by creating an osmotic gradient.

Which statement best describes the function of globulins? A) They transport lipids, playing a role in clotting and defending the body against foreign substances. B) They primarily affect the movement of fluid into capillaries by creating an osmotic gradient. C) They provide the nutrients needed to replicate erythrocytes. D) They are the key substance in the formation of blood clots. E) They aid in the structural form of the erythrocyte.

A) They transport lipids, playing a role in clotting and defending the body against foreign substances.

Constriction of which vessel, by intrinsic or extrinsic mechanisms, decreases glomerular filtration rate? A) afferent arteriole B) peritubular capillary C) vasa recta D) glomerulus E) efferent arteriole

A) afferent arteriole

What is the most abundant plasma protein? A) albumin B) hemoglobin C) fibrinogen D) globulins E) carbonic anhydrase

A) albumin

Within the kidneys, what membrane has microvilli? A) apical membrane of renal tubule epithelial cells B) basolateral membrane of renal tubule epithelial cells C) basolateral membrane of peritubular capillary endothelial cells D) apical membrane of vasa recta endothelial cells E) glomerulus

A) apical membrane of renal tubule epithelial cells

The osmotic pressure gradient across the glomerular capillaries tends to move water A) back into the capillaries. B) into the filtrate. C) against its concentration gradient. D) into the collecting duct.

A) back into the capillaries.

The most important buffer system in the extracellular fluid is A) bicarbonate. B) monoprotonated phosphates. C) amino acids. D) carboxyl groups. E) chloride.

A) bicarbonate

How might a high-fat diet cause metabolic acidosis? A) breakdown of fat into fatty acids B) direct production of hydrogen ions C) loss of hydrogen ions D) loss of phosphate E) loss of bicarbonate

A) breakdown of fat into fatty acids

What hormone decreases plasma calcium levels? A) calcitonin only B) parathyroid hormone only E) both calcitonin and parathyroid hormone

A) calcitonin only

Antidiuretic hormone binds to receptors on cells in the collecting ducts and distal tubules where it ________. A) causes vesicles with aquaporin-2 to insert into the apical membrane B) causes vesicles with aquaporin-3 to insert into the apical membrane C) causes vesicles with aquaporin-2 to insert into the basolateral membrane

A) causes vesicles with aquaporin-2 to insert into the apical membrane

The primary function of atrial natriuretic peptide (ANP) is to A) decrease sodium reabsorption. B) increase sodium reabsorption. C) increase potassium secretion thereby decreasing potassium reabsorption. D) decrease potassium secretion thereby increasing potassium reabsorption. E) increase the stretching capacity of the atria.

A) decrease sodium reabsorption.

Which of the following is NOT a response to hemorrhage? A) decreased renin secretion B) increased sympathetic activity C) increased angiotensin II production D) increased ADH release

A) decreased renin secretion

Which chemical forms the meshwork of the blood clot? A) fibrin B) thrombin C) thromboxane A2 D) von Willebrand factor E) arachidonic acid

A) fibrin

What is the process whereby protein-free plasma moves by bulk flow from the glomerulus into Bowman's capsules? A) glomerular filtration B) reabsorption C) absorption D) secretion E) excretion

A) glomerular filtration

What is the sum of Starling's forces within the renal corpuscle called? A) glomerular filtration pressure B) glomerular filtration rate C) glomerular filtration volume D) glomerular oncotic pressure E) glomerular capillary hydrostatic pressure

A) glomerular filtration pressure

At high concentrations of antidiuretic hormone, the extent of water reabsorption in the collecting ducts is ________, causing urine output to ________. A) high : decrease B) low : decrease C) high : increase D) low : increase E) unaffected : remain unchanged

A) high : decrease

A greater plasma volume than normal is called A) hypervolemia. B) hypovolemia. C) normovolemia. D) equilibrium. E) fluid balance.

A) hypervolemia

In the renal tubules, where is the Na+/K+ pump located? A) in the basolateral membrane of the proximal tubules B) in the apical membrane of the descending limb of the loop of Henle C) in the apical membrane of the collecting ducts D) in both the apical and basolateral membranes of distal tubules E) in both the apical and basolateral membranes of proximal tubules

A) in the basolateral membrane of the proximal tubules

If hydrogen ions are taken out of a solution, the pH will A) increase. B) decrease. C) remain the same.

A) increase

In order to compensate for a hyperventilation, the A) kidneys decrease the reabsorption of bicarbonate. B) kidneys increase the conversion of glutamine to bicarbonate. C) kidneys increase the secretion of hydrogen ions. D) lungs increase ventilation rate. E) kidneys increase the reabsorption of hydrogen ions.

A) kidneys decrease the reabsorption of bicarbonate.

Which of the following conditions triggers the release of renin? A) low blood pressure B) high blood pressure C) increased parasympathetic stimulation D) atrial stretching due to increased plasma volume E) increased mean arterial pressure (MAP)

A) low blood pressure

The hydrogen ion concentration or pH of arterial blood is regulated by the combined actions of the ________ and ________. A) lungs : kidneys B) lungs: buffer systems C) kidneys : buffer systems D) kidneys : cardiovascular system E) kidneys : lymphatic system

A) lungs : kidneys

Which cells of the distal tubule secrete a paracrine in response to changes in the filtrate flow rate? A) macula densa cells B) endothelial cells C) mesangial cells D) granular cells E) podocytes

A) macula densa cells

A patient is exhibiting several signs of acid-base imbalance. Blood tests reveal that blood pH is 7.3, and bicarbonate and carbon dioxide levels in blood are both low. What is the state of this patient? A) metabolic acidosis with respiratory compensation B) respiratory acidosis with renal compensation C) metabolic alkalosis with respiratory compensation D) respiratory alkalosis with renal compensation E) Blood pH is normal.

A) metabolic acidosis with respiratory compensation

The diffusion of water across a tubule is driven by differences in ________ across the membrane. A) osmolarity B) partial pressure C) active transport D) volume E) potassium concentration

A) osmolarity

A decrease in plasma calcium will initiate an increase in the release of which of the following? A) parathyroid hormone B) calcitonin C) both parathyroid hormone and calcitonin

A) parathyroid hormone

Which of the following ions acts as a buffer to minimize changes in intracellular pH? A) phosphate B) bicarbonate C) chloride D) potassium E) sodium

A) phosphate

What chemical dissolves blood clots? A) plasmin B) heparin C) fibrin D) protein C E) thromboxane A2

A) plasmin

How might heavy exercise cause metabolic acidosis? A) production of lactic acid B) direct production of hydrogen ions C) loss of hydrogen ions D) loss of phosphate E) loss of bicarbonate

A) production of lactic acid

Carbonic anhydrase is located where? A) proximal tubule apical membrane B) intercalated cells basolateral membrane C) proximal tubule basolateral membrane D) principal cells basolateral membrane E) principal cells apical membrane

A) proximal tubule apical membrane

Sodium-glucose cotransporters are located where? A) proximal tubule apical membrane B) proximal tubule basolateral membrane

A) proximal tubule apical membrane

What is the secondary defense against changes in pH that requires minutes to be activated? A) respiratory compensation B) buffering of hydrogen ions C) renal compensation D) cardiac compensation E) hepatic compensation

A) respiratory compensation

What solute is primarily responsible for producing the osmotic gradient that drives water reabsorption? A) sodium B) potassium C) calcium D) phosphate E) chloride

A) sodium

Fluid and electrolyte balance occur when A) solutes and water enter and exit the plasma at the same rate. B) solutes and water enter the plasma at a greater rate than it exits. C) solutes and water exit the plasma at a greater rate than it enters. D) the charges are neutral and happen to be in an aqueous solution. E) the amount of solute in the plasma is the same as the amount in the interstitial fluid.

A) solutes and water enter and exit the plasma at the same rate.

Platelets become sticky when exposed to which of the following? A) von Willebrand factor B) prostacyclin C) fibrin D) thrombin E) serotonin

A) von Willebrand factor

A decrease in glomerular filtration rate tends to decrease urine output. A. True B. False

A. True

A metabolic alkalosis will cause a decrease in ventilation and an increased excretion of bicarbonate. A. True B. False

A. True

Acidosis results in potassium retention, whereas alkalosis results in potassium depletion. A. True B. False

A. True

Aldosterone stimulates an increase in sodium reabsorption at the same time it stimulates an increase in potassium secretion. A. True B. False

A. True

Alterations in acid-base balance that originate from the respiratory system involve alterations in CO2 content within the blood. A. True B. False

A. True

Consumption of salty foods without drinking water will increase plasma osmolarity and cause cells to shrink. A. True B. False

A. True

Glomerular capillary hydrostatic pressure is greater than systemic capillary hydrostatic pressure. A. True B. False

A. True

Material that enters the lumen of the renal tubules is excreted unless it is reabsorbed. A. True B. False

A. True

Metabolic acidosis is a disturbance in blood pH caused by something other than an abnormal PCO2. A. True B. False

A. True

The epithelial cells of the distal tubules contain receptors for hormones that can modify reabsorption or secretion of substances including water. A. True B. False

A. True

The intrinsic and extrinsic controls of glomerular filtration rate ultimately involve changes in resistance at the afferent arteriole. A. True B. False

A. True

The reabsorption of sodium and water occurs in a relatively unregulated manner in the proximal tubules. A. True B. False

A. True

Under normal conditions, the total solute concentration of the plasma and the interstitial fluid is approximately 300 mOsm. A. True B. False

A. True

If a substance is freely filtered and is neither reabsorbed nor secreted, then the amount of that substance in the urine must be equal to the amount filtered. A. True B. False

A. true

If the amount of a solute excreted is greater than filtered load, then that solute was secreted. A. True B. False

A. true

If the amount of solute excreted per minute is less than the filtered load, then the solute was reabsorbed in the renal tubules. A. True B. False

A. true

Juxtaglomerular cells are located in the walls of the distal tubule. A. True B. False

A. true

Kidneys compensate for changes in plasma volume and osmolarity by adjusting the rate of water reabsorption only. A. true B. false

A. true

Once glucose concentration in the plasma exceeds the renal threshold, glucose will be excreted in the urine. A. True B. False

A. true

The glomerular capillaries are composed of a fenestrated surface that allows protein-free plasma to exit the vasculature. A. True B. False

A. true

The obligatory water loss refers to the lowest volume of urine that must be produced to eliminate solutes. A. True B. False

A. true

Drugs that treat hypertension by preventing Angiotensin I from becoming Angiotensin II are called ________

ACE inhibitors

In severe dehydration or blood loss, what would be the levels of ADH and what would be the urine flow rate?

ADH - high; low urine flow rate (0.25 ml/min) Yes, ADH would be high, causing a large percentage of the water to be reabsorbed. Therefore, the urine flow rate would be low.

In overhydration, what would be the levels of ADH (high, normal, or low) and what would be the osmolarity of the urine?

ADH - low; 100 mOsm (urine) Yes, in overhydration, ADH would be very low and the cells of the collecting duct would be relatively impermeable to water and urea. Thus, the final urine would be dilute, having an osmolarity of about 100 mOsm.

Write out the words for the following abbreviations: ADH, ANP, ACE, ANG II, JG apparatus, P cell, I cell

ADH = antidiuretic hormone ANP = atrial natriuretic peptide ACE =angiotensin-converting enzyme ANG II = angiotensin II JG (apparatus) = justaglomerular P cell = principal cell I cell = intercalated cell

Which of the following statements about ADH (antidiuretic hormone) is correct?

ADH is released by the posterior pituitary gland.

When mean arterial pressure decreases, what may increase in response?

ADH levels and water reabsorption

An individual is severely dehydrated. How would this affect their ADH release and the volume of their urine?

ADH release would increase and urine volume would decrease ADH causes the kidney to reabsorb more water, minimizing water loss from the body.

Hemorrhage would tend to increase the secretion of what?

ADH, aldosterone, and erythropoietin

What are the main differences between metabolism in the absorptive and postabsorptive states?

Absorptive state—anabolic reactions and nutrient storage. Postabsorptive—mobilizes stored nutrients for energy and synthesis.

Which is more likely to accumulate in the body, acids or bases? List some sources of each

Acids from CO2, metabolism, and food are more likely. Sources of bases include some foods

What is the order of the blood vessels that carry blood to and within the kidney? (order in which blood passes through them)

Afferent arteriole -> glomerulus -> efferent arteriole -> peritubular capillary

What is the path of water through the urinary system, beginning with the plasma in the afferent arteriole?

Afferent arteriole > glomerulus > bowman's capsule > proximal tubule > descending and ascending loop of Henle > distal tubule > collecting duct > renal papilla > minor caylax > major caylax > renal pelvis > ureter > bladder > urethra

If the afferent arteriole of a nephron constricts, what happens to GFR in that nephron? If the efferent arteriole of a nephron constricts, what happens to GFR in that nephron? Assume that no autoregulation takes place.

Afferent arteriole constricts, GFR decreases. Efferent arteriole constricts, GFR increases.

hormone responsible for the absorption of sodium in the distal tubule and the collecting duct

Aldosterone

Which of the following statements about aldosterone is NOT correct?

Aldosterone increases sodium reabsorption by increasing the number of Na+-K+ ATPase pumps in the luminal membrane of the proximal tubule. Yes, this is the incorrect statement. Aldosterone increases the number of Na+-K+ ATPase pumps in the basolateral membrane of the distal tubules and collecting ducts.

True of Aldosterone

Aldosterone is stimulated by decreased plasma sodium levels and increased plasma potassium levels. Aldosterone is produced in the adrenal cortex. Aldosterone increases the number of passive sodium channels in the luminal membrane of the distal tubule and collecting duct, thus aiding sodium reabsorption.

During severe dehydration, the adrenal cortex receives two competing stimuli: high plasma osmolarity and decreased volume. In response to this, aldosterone is not released. Why is this beneficial?

Aldosterone would increase sodium retention, leading to shrinkage of cells throughout the body.

What are the advantages of a kidney that filters a large volume of fluid and then reabsorbs 99% of it?

Allows rapid removal of foreign substances that are filtered but not reabsorbed.

Renin

An enzyme released by the granular cells of the juxtaglomerular apparatus. Converts angiotensinogen into angiotensin Ⅰ.

What is a zymogen? What is a proenzyme? List two examples of each.

An inactive digestive proenzyme. Must have a segment of protein chain removed to activate. Examples: pepsinogen-pepsin, trypsinogen-trypsin.

Which hormone will directly act on the collecting duct of the kidney to enhance the reabsorption of water?

Antidiuretic hormone

Toxins, drugs, and metabolic wastes are cleared from the body by secretion into the filtrate. Which parts of the nephron could these items be secreted into?

Any part of the nephron except the nephron loop Secretion of substances takes place in the proximal and distal tubule, and in the collecting duct.

Using what you have learned about epithelial transport, draw a picture of the salivary duct cells and lumen. Arrange the following membrane channels and transporters on the apical and basolateral membranes so that the duct cell absorbs Na+ and secretes K+: ENaC, Na+-K+-ATPase, and K+ leak channel. With neural stimulation, the flow rate of saliva can increase from 0.4 mL/min to 2 mL/min. What do you think happens to the Na+ and K+ content of saliva at the higher flow rate?

Apical membrane has ENaC (Na+Na+ leak channel) and K+K+ leak channels. Basolateral membrane has the Na+-K+-ATPase.Na+-K+-ATPase. At high flow, saliva has more Na+Na+ and less K+.

Tight junction

Area where plasma membranes of adjacent cells are tightly bound together, forming an impermeable barrier.

Which part of the nephron always has low permeability to water, regardless of hormone levels?

Ascending loop of Henle

In which of the following locations is the osmolarity of the filtrate the greatest?

At the base of the loop of Henle

Over what range of arterial pressure does glomerular filtration rate remain stable due to intrinsic regulation? A) 50-150 mm Hg B) 80-180 mm Hg C) 70-110 mm Hg D) 80-100 mm Hg E) 80-120 mm Hg

B) 80-180 mm Hg

Aldosterone is released from the ________ in response to ________. A) adrenal medulla : decreases in plasma sodium B) adrenal cortex : increases in plasma potassium C) posterior pituitary : increases in plasma osmolarity D) pancreas : increases in plasma glucose E) macula densa : increased flow of tubular fluid in the distal tubule

B) adrenal cortex : increases in plasma potassium

During myogenic regulation of glomerular filtration rate, an increase in mean arterial pressure will cause the A) efferent arterioles to constrict and thereby maintain a relatively constant glomerular filtration pressure. B) afferent arterioles to constrict and thereby maintain a relatively constant glomerular filtration pressure. C) efferent arterioles to dilate and thereby maintain a relatively constant glomerular filtration pressure. D) afferent arterioles to dilate and thereby maintain a relatively constant glomerular filtration pressure. E) afferent arterioles to dilate and thereby allow glomerular filtration pressure to increase.

B) afferent arterioles to constrict and thereby maintain a relatively constant glomerular filtration pressure.

Release of what hormone is stimulated by high levels of potassium? A) ADH B) aldosterone C) angiotensin II D) atrial natriuretic peptide E) parathyroid hormone

B) aldosterone

Erythrocytes are synthesized in what organ and under the control of what chemical? A) liver, under control of bilirubin B) bone marrow, under control of erythropoietin C) kidney, under control of transferrin D) liver, under control of erythropoietin E) bone marrow, under control of bilirubin

B) bone marrow, under control of erythropoietin

What is the most rapid defense against changes in blood pH? A) respiratory compensation B) buffering of hydrogen ions C) renal compensation D) cardiac compensation E) hepatic compensation

B) buffering of hydrogen ions

Osmotic pressure resulting from presence of plasma proteins in blood is called ________ pressure. A) oncotic B) colloid osmotic C) hydrostatic D) oncotic and colloid osmotic E) colloid osmotic and hydrostatic

B) colloid osmotic

If hydrogen ions are added to a solution, the pH will A) increase. B) decrease. C) remain the same.

B) decrease

What triggers erythropoietin release? A) increased blood carbon dioxide levels B) decreased blood oxygen levels C) infection in the tissues D) infection in the blood E) damage to a blood vessel wall and subsequent exposure of collagen

B) decreased blood oxygen levels

What stimulates atrial natriuretic peptide release? A) distension of the atrial wall due to an increase in blood pressure B) distension of the atrial wall due to an increase in plasma volume C) changes in the concentration of sodium in the blood of the atrium D) changes in the concentration of potassium in the blood of the atrium E) renin

B) distension of the atrial wall due to an increase in plasma volume

Stretching of the bladder reflexively causes all of the following EXCEPT A) excitation of parasympathetic neurons, causing the detrusor muscles to contract. B) excitation of sympathetic neurons, causing the internal urethral sphincter to contract. C) inhibition of somatic neurons to the external urethral sphincter, causing the sphincter to open. D) micturition. E) opening of the urethral sphincters.

B) excitation of sympathetic neurons, causing the internal urethral sphincter to contract.

Both the intrinsic and extrinsic pathways converge on activation of what factor, which then activates prothrombin? A) calcium B) factor X C) platelet factor D) factor IV E) tissue factor

B) factor X

Due to the differences in opposing forces, there is net ________ occurring at the arteriolar end of most capillaries, coupled with net ________ at the venous end. A) absorption, filtration B) filtration, absorption

B) filtration, absorption

What Starling force, which drives fluid out of the glomerulus, is relatively high (~60 mm Hg) due to the resistance of the efferent arterioles? A) Bowman's capsule oncotic B) glomerular capillary hydrostatic C) Bowman's capsule hydrostatic D) glomerular oncotic E) glomerular filtration

B) glomerular capillary hydrostatic

Changes in mean arterial pressure have the potential to alter glomerular filtration rate by directly altering which of the following? A) glomerular oncotic pressure B) glomerular capillary hydrostatic pressure C) Bowman's capsule oncotic pressure D) Bowman's capsule hydrostatic pressure E) efferent arteriole resistance

B) glomerular capillary hydrostatic pressure

Renin is released by ________ cells of the ________. A) principal : afferent arteriole B) granular : afferent arteriole C) principal : distal tubule D) granular : distal tubule E) intercalated : afferent arteriole

B) granular : afferent arteriole

Which cells of the afferent arteriole secrete renin? A) podocytes B) granular cells C) macula densa cells D) mesangial cells E) endothelial cells

B) granular cells

Which of the following is associated with actions of aldosterone on tubular cells? A) increased sodium secretion B) increased potassium secretion C) decreased blood pressure D) decreased plasma osmolarity E) increased calcium reabsorption

B) increased potassium secretion

A substance is in positive balance when A) it enters plasma at the same rate it exits plasma. B) it enters plasma at a greater rate than it exits plasma. C) it exits plasma at a greater rate than it enters plasma. D) its usage by cells is increased. E) its production by cells is decreased.

B) it enters plasma at a greater rate than it exits plasma.

The primary organ where erythropoietin is produced is the A) liver. B) kidney. C) spleen. D) bone marrow. E) endothelial cells throughout the body.

B) kidney

The role of a buffer is to A) reduce the reabsorption of hydrogen ions in the kidneys. B) limit the change in pH with changing hydrogen ion concentrations. C) reduce the secretion of hydrogen ions in the kidneys. D) reduce the secretion of bicarbonate ions in the kidneys. E) enhance the ability of an increase in hydrogen ions to increase pH.

B) limit the change in pH with changing hydrogen ion concentrations.

During tubuloglomerular feedback of glomerular filtration rate, paracrines secreted from what structure(s) cause constriction of the afferent arteriole? A) glomerulus B) macula densa C) mesangial cells D) proximal tubule E) juxtaglomerular cells

B) macula densa

Most reabsorption occurs in the ________ and is ________. A) proximal tubule : regulated B) proximal tubule : not regulated C) distal tubule and collecting duct : regulated D) distal tubule and collecting duct : not regulated E) loop of Henle : regulated

B) proximal tubule : not regulated

Solutes that enter the lumen of the renal tubules are excreted unless they are A) secreted. B) reabsorbed. C) filtered. D) degraded. E) bound to receptors.

B) reabsorbed.

What would the following blood values indicate? Blood pH = 7.3, PCO2 is high, [HCO3-] is high. A) metabolic alkalosis with respiratory compensation B) respiratory acidosis with renal compensation C) respiratory alkalosis with renal compensation D) metabolic acidosis with respiratory compensation E) metabolic acidosis only

B) respiratory acidosis with renal compensation

To voluntarily control urination, the ________ nervous system innervates the ________. A) somatic : internal urethral sphincter B) somatic : external urethral sphincter C) somatic : detrusor muscle D) autonomic : internal urethral sphincter E) autonomic : external urethral sphincter

B) somatic : external urethral sphincter

What is a hematocrit measuring? A) the amount of hemoglobin in blood B) the percentage of blood that is comprised of red blood cells only C) the percentage of blood that is comprised of red and white blood cells D) the percentage of blood that is comprised of plasma E) the amount of oxygen that can be transported by blood

B) the percentage of blood that is comprised of red blood cells only

What converts fibrinogen to fibrin? A) prostacyclin B) thrombin C) von Willebrand factor D) arachidonic acid E) plasmin

B) thrombin

In activated platelets, arachidonic acid is converted to ________, which ________ formation of blood clots. A) thromboxane A2 : inhibits B) thromboxane A2 : stimulates C) prostacyclin : inhibits D) prostacyclin : stimulates E) nitric oxide : inhibits

B) thromboxane A2 : stimulates

What is the primary barrier to reabsorption in the renal tubules? A) capillary endothelial cell B) tubule epithelial cell C) capillary basement membrane D) tubule basement membrane E) peritubular space

B) tubule epithelial cell

As the body becomes dehydrated, the osmolarity of the body fluids decreases. A. True B. False

B. False

In a state of negative balance, the quantity of a substance in the plasma tends to increase. A. True B. False

B. False

Sodium movement in the proximal tubule: Sodium is transported across the apical membrane by the Na+/K+ pump and across the basolateral membrane by diffusion through sodium channels. A. True B. False

B. False

The distal tubule functions as a mass absorber. A. True B. False

B. False

The external urethral sphincter is controlled in an involuntary manner by the sympathetic nervous system. A. True B. False

B. False

Glomerular filtration involves the bulk flow of plasma (with all of its solutes) into Bowman's space. A. True B. False

B. false

Glomerular filtration is approximately 3 L/day. A. True B. False

B. false

What vitamin must be complexed with intrinsic factor in order to be absorbed?

B12

Which vitamin must be complexed with intrinsic factor in order to be absorbed?

B12

Vitamin ________ must be complexed with a protein called ________ to be absorbed from the small intestine.

B12; intrinsic factor

Define basal metabolic rate (BMR). Under what conditions is it measured? Why does the average BMR differ in adult males and females? List at least four factors other than sex that may affect BMR in humans.

BMR—an individual's lowest metabolic rate, measured at rest after sleep and a 12-hour fast. Higher in adult males because females have more adipose tissue with a lower respiration rate. Factors that affect BMR: age, physical activity, lean muscle mass, diet, hormones, and genetics.

Diarrhea often accompanies intestinal infections. Why?

Bacterial toxins enhance the secretion of Cl- from the cells, which causes water movement into the intestine

Describe the structure and function of the basal lamina. (Fig. 19.5d)

Basal lamina acts like a sieve excluding most plasma proteins Acellular, extracellular matrix between capillary endothelium and epithelium of Bowman's capsule

Because Mrs. F's cells are______to her plasma. water will move_________by ___________, causing her tissues to ____________________

Because Mrs. F's cells are hypertonic to her plasma, water will move into the cells by osmosis, causing her tissues to swell.

If blood pH increases, which of the following may increase to compensate?

Bicarbonate synthesis in the distal tubule H+ secretion Bicarbonate reabsorption

The liver further converts it into , which is stored in the

Bile gallbladder

Fat Digestion and Absorption events associated with fat digestion in the correct sequential order.

Bile salts from liver coat fat droplets Lipase and colipase break down fats Digested fats are stores in micelles Monoglycerides, free fatty acids, and cholesterol are absorbed along the enterocyte border Chylomicron is formed and transported out of the enterocyte

__________ is (are) a key component of bile. Its (Their) function is __________.

Bile salts; to emulsify fats during digestion

__________ is/are a key component of bile. Its function is __________.

Bile salts; to emulsify fats during digestion

Antimuscarinic drugs are the accepted treatment for an overactive bladder. Explain why they work for this condition.

Bladder smooth muscle contracts under parasympathetic control, so blocking muscarinic receptors decreases bladder contraction.

B) mesenteric vein

Blood from the intestinal capillaries drains into what blood vessel before entering the hepatic portal vein? A) hepatic artery B) mesenteric vein C) hepatic vein D) coeliac vein E) intestinal vein

Which of the following correctly indicates the direction of the different forces causing filtration in the renal corpuscle (of fluid flow into or out of Bowman's capsule from the glomerular capillaries)?

Blood hydrostatic pressure - (out), blood colloid osmotic pressure - (in), hydrostatic fluid pressure of the Bowman's capsule - (in)

Hannah, a 31-year-old woman, decided to have colonic irrigation, a procedure during which large volumes of distilled water were infused into her rectum. During the treatment, she absorbed 3000 mL of water. About 12 hours later, her roommate found her in convulsions and took her to the emergency room. Her blood pressure was 140/90, her plasma Na+Na+ concentration was 106 mEq/L (normal: 135 mEq/L), and her plasma osmolarity was 270 mOsM. In a concept map or flowchart, diagram all the homeostatic responses her body was using to attempt compensation for the changes in blood pressure and osmolarity.

Blood pressure is high, plasma Na+Na+ and osmolarity are low.

Chyme

Bolus + gastric juice once it enters the stomach

If plasma calcium levels decrease, which of the following may increase in response?

Bone resorption, PTH (parathyroid hormone) and reabsorption of calcium by the kidneys

Compare the enteric nervous system with the cephalic brain. Give some specific examples of neurotransmitters, neuromodulators, and supporting cells in the two.

Both use similar neurotransmitters and neuromodulators (serotonin, VIP, NO). Enteric support cells are similar to CNS astroglia. GI capillaries are not very permeable, like blood-brain barrier. Both act as integrating centers.

Trace a drop of filtered fluid from Bowman's capsule through the tubule of the nephron, ending in the renal pelvis. (Fig. 19.1c, g, i)

Bowman's capsule => proximal tubule => loop of henle => distal tubule => collecting duct => renal calyx => renal pelvis

Which of the following organ systems will be most negatively affected by the hyponatremic state?

Brain

Chemical digestion

Breaking chemical bonds by enzymes

Digestion

Breaking down

What are the three general fates of biomolecules in the body?

Broken down for energy, used for synthesis, or stored.

If a human drinks 200 ml of seawater, how will the body correct for the increase in plasma osmolarity?

By increasing the release of vasopressin

Tubular epithelial cells of the collecting duct and distal tubule contain receptors for what hormone that stimulates sodium reabsorption? A) antidiuretic hormone B) renin C) aldosterone D) atrial natriuretic peptide E) erythropoietin

C) aldosterone

What hormone stimulates the release of aldosterone? A) ADH B) aldosterone C) angiotensin II D) atrial natriuretic peptide E) parathyroid hormone

C) angiotensin II

What enzyme converts angiotensinogen into angiotensin I? A) carbonic anhydrase B) atrial natriuretic peptide C) angiotensin converting enzyme (ACE) D) renin E) mucin

C) angiotensin converting enzyme (ACE)

Bicarbonate crosses the apical membrane of the renal tubule by A) HCO3-/Cl- countertransporters. B) Na+/HCO3- cotransporters. C) conversion to carbon dioxide. D) HCO3- active transporters. E) Na+/HCO3- countertransporters.

C) conversion to carbon dioxide.

The process of fibrinolysis A) activates fibrinogen. B) draws torn edges of damaged tissue closer together. C) dissolves clots. D) forms emboli. E) forms thrombi.

C) dissolves clots

Aquaporin-3 is located on what membrane in the absence of ADH? A) proximal tubule apical membrane B) proximal tubule basolateral membrane C) distal tubular and collecting duct cells basolateral membrane D) distal tubular and collecting duct cells apical membrane

C) distal tubular and collecting duct cells basolateral membrane

Receptors for ADH are located where? A) proximal tubule apical membrane B) proximal tubule basolateral membrane C) distal tubular and collecting duct cells basolateral membrane E) distal tubular and collecting cells apical membrane

C) distal tubular and collecting duct cells basolateral membrane

The iron-containing ring in hemoglobin is called ________ and it binds ________. A) heme : carbon dioxide B) spectrin : carbon dioxide C) heme : oxygen D) globin : oxygen E) heme : carbon dioxide or oxygen

C) heme : oxygen

What is the strongest stimulus for the release of antidiuretic hormone from the posterior pituitary? A) decrease in plasma osmolarity B) decrease in blood pressure C) increase in plasma osmolarity D) increase in blood pressure E) decrease in sodium content of the blood

C) increase in plasma osmolarity

Which of the following is an effect of aldosterone on distal tubular cells? A) decreased number of open Na+/K+ channels in the basolateral membrane B) decreased number of Na+/K+ pumps in the basolateral membrane C) increased number of open Na+/K+ channels in the basolateral membrane D) increased number of open Na+/K+ channels in the apical membrane E) increased number of Na+/K+ pumps in the apical membrane

C) increased number of open Na+/K+ channels in the basolateral membrane

A substance is in negative balance when A) it enters plasma at the same rate it exits plasma. B) it enters plasma at a greater rate than it exits plasma. C) it exits plasma at a greater rate than it enters plasma. D) its usage by cells is decreased. E) its production by cells is increased.

C) it exits plasma at a greater rate than it enters plasma.

Angiotensinogen is synthesized by what organ? A) kidney B) bone marrow C) liver D) adrenal gland E) pituitary gland

C) liver

Most coagulation factors are synthesized by what organ? A) bone marrow B) kidney C) liver D) thymus E) heart

C) liver

How does severe vomiting cause a metabolic alkalosis? A) production of bicarbonate B) production of hydrogen ions C) loss of hydrogen ions D) loss of phosphate E) loss of bicarbonate

C) loss of hydrogen ions

Which of the following is NOT a mechanism whereby glomerular filtration rate is regulated? A) tubuloglomerular feedback B) myogenic regulation C) parasympathetic nervous system D) sympathetic nervous system

C) parasympathetic nervous system

Hyperkalemia refers to an excess of what? A) water B) sodium C) potassium D) calcium E) chloride

C) potassium

In healthy endothelial cells, arachidonic acid is converted to ________, which ________ formation of blood clots. A) thromboxane A2 : inhibits B) thromboxane A2 : stimulates C) prostacyclin : inhibits D) prostacyclin : stimulates E) nitric oxide : inhibits

C) prostacyclin : inhibits

Most sodium reabsorption is driven by the Na+/K+ pump located in what region? A) proximal tubule apical membrane B) intercalated cells basolateral membrane C) proximal tubule basolateral membrane D) principal cells basolateral membrane E) principal cells apical membrane

C) proximal tubule basolateral membrane

The extrinsic pathway of coagulation is activated by the A) sticking of platelets to damaged tissue. B) activation of a proenzyme exposed to collagen. C) release of tissue factor by a damaged endothelium. D) release of heparin from the liver. E) conversion of prothrombin to thrombin.

C) release of tissue factor by a damaged endothelium.

What is plasma from which clotting factors have been removed called? A) buffy white coat B) thrombocytes C) serum D) colony-stimulating factors E) platelets

C) serum

A hemorrhage-induced decrease in mean arterial pressure will decrease glomerular filtration rate by intrinsic and extrinsic mechanisms; the extrinsic mechanisms act through A) tubuloglomerular feedback. B) the parasympathetic nervous system. C) the sympathetic nervous system. D) myogenic responses.

C) the sympathetic nervous system

Which of the following intestinal hormones stimulates the release of bile from the gall bladder?

CCK (cholecystokinin) CCK causes the gall bladder to contract, releasing bile and an enzyme-rich pancreatic juice to be secreted from the pancreas.

Symptoms of low plasma pH may include

CNS depression and confusion and disorientation

The most important factor affecting the pH of plasma is the concentration of

CO2

Write the equation that shows how CO2 is related to pH. What enzyme increases the rate of this reaction? Name two cell types that possess high concentrations of this enzyme.

CO2 + H2O -> H2CO3 -> H+ + HCO3- Carbonic anhydrase high in renal tubule cells and RBCs

Respiratory acidosis/alkalosis deals with...

CO2 levels

What force(s) promote(s) glomerular filtration? What force(s) oppose(s) it? What is meant by the term net driving force?

Capillary hydrostatic pressure promotes filtration. Fluid pressure in Bowman's capsule and colloid osmotic (oncotic) pressure of plasma oppose it. Net driving force is the sum of these pressures.

B. False

Carbohydrates are absorbed as disaccharides. A. True B. False

The respiratory system is one of the three systems that regulate acid-base balance in the body. How does it work to decrease an acidosis?

Carbonic acid is broken down into water and CO2; the CO2 is then exhaled. CO2 can be blown off, thus decreasing acidity.

E) both parietal cells and lumen of the small intestine

Carbonic anhydrase is located in which of the following digestive structures? A) salivary glands only B) parietal cells only C) lumen of the small intestine only D) both salivary glands and parietal cells E) both parietal cells and lumen of the small intestine

Which of the following polysaccharides cannot be digested in the human GI tract?

Cellulose

Which of the following is a similarity between respiratory acidosis and respiratory alkalosis?

Changes in pH are the result of changes in the amount of carbon dioxide in the body

Bolus

Chewed up food mixed with saliva

What information about renal handling can clearance provide? What information about renal handling can clearance not provide?

Clearence can be used to determine how the nephron handles a filtered solute. Can show only net handling of solute Doesn't say if a molecule is both absorbed and secreted

________ is a protein cofactor secreted by the pancreas that allows lipases an access to fats inside the bile salt coating.

Colipase

Fluid is considered urine when it enters the __________________.

Collecting duct

What is the renal corpuscle? What happens here?

Consists of the glomerulus and Bowman's capsule Filtration of mostly protein-free plasma from the capillaries into the capsule

During the micturition reflex, what does activation of parasympathetic efferent neurons cause the bladder to do?

Contract

A person must void what volume of urine per day? A) 0 mL B) 15 mL C) 120 mL D) 440 mL E) 1200 mL

D) 440 mL

A normal adult hematocrit would be approximately ________%. A) 100 B) 75 C) 66 D) 45 E) 10

D) 45

Which of the following pressures across the glomerular capillaries approaches zero? A) glomerular filtration pressure B) glomerular oncotic pressure C) glomerular capillary hydrostatic pressure D) Bowman's capsule oncotic pressure E) Bowman's capsule hydrostatic pressure

D) Bowman's capsule oncotic pressure

Which statement best describes the function of fibrinogen? A) It transports lipids, playing a role in clotting and defending the body against foreign substances. B) It primarily affects the movement of fluid into capillaries by creating an osmotic gradient. C) It provides the nutrients needed to replicate erythrocytes. D) It is the key substance in the formation of blood clots. E) It aids in the structural form of the erythrocyte.

D) It is the key substance in the formation of blood clots.

Which of the following correctly describes water movement across the epithelial cells lining the collecting duct? A) Water can permeate the lipid bilayer at all times. B) Water can permeate the phospholipid bilayer only in the presence of ADH. C) Water can permeate the phospholipid bilayer only in the presence of aldosterone. D) Water can permeate the apical membrane through aquaporin-2 and the basolateral membrane through aquaporin-3, but aquaporin-2 is only present in the presence of ADH. E) Water can permeate the basolateral membrane through aquaporin-2 and the apical membrane through aquaporin-3, but aquaporin-2 is only present in the presence of aldosterone.

D) Water can permeate the apical membrane through aquaporin-2 and the basolateral membrane through aquaporin-3, but aquaporin-2 is only present in the presence of ADH.

The common pathway of coagulation begins with the A) sticking of platelets to damaged tissue. B) activation of a proenzyme exposed to collagen. C) release of tissue factor by a damaged endothelium. D) activating of a clotting factor that converts prothrombin to thrombin. E) activation of a clotting factor that converts fibrinogen to fibrin.

D) activating of a clotting factor that converts prothrombin to thrombin.

Which of the following is a source of water input? A) digestive tract only B) renal tubules only C) metabolism only D) both digestive tract and metabolism E) both the digestive tract and renal tubules

D) both digestive tract and metabolism

Substances can enter the renal tubules by what renal processes? A) filtration only B) reabsorption only C) secretion only D) both filtration and secretion E) both reabsorption and secretion

D) both filtration and secretion

Which of the following is the most accurate description of differences between epithelial cells of the proximal and distal tubules? A) less prominent microvilli in the epithelial cells of the distal tubule B) receptors for hormones on the epithelial cells of the distal tubule C) both statements are inaccurate D) both statement are accurate

D) both statement are accurate

Which muscle(s) that regulate(s) micturition is/are under involuntary control? A) detrusor B) internal urethral sphincter C) external urethral sphincter D) detrusor and the internal urethral sphincter E) detrusor and the external urethral sphincter

D) detrusor and the internal urethral sphincter

Aldosterone increases the number of potassium channels where? A) proximal tubule apical membrane B) proximal tubule basolateral membrane C) distal tubular cells basolateral membrane D) distal tubular cells apical membrane

D) distal tubular cells apical membrane

The process of red blood cell production is called A) erythrocytosis. B) erythropenia. C) hemocytosis. D) erythropoiesis. E) hematopenia.

D) erythropoiesis

Which of the following is NOT a metabolic disturbance that can result in a metabolic acidosis? A) high-protein diet B) high-fat diet C) heavy exercise D) excessive vomiting E) severe diarrhea

D) excessive vomiting

A hemoglobin molecule is composed of A) just two protein chains. B) just three protein chains. C) just four protein chains and nothing else. D) four protein chains and four heme groups. E) four heme groups but no proteins

D) four protein chains and four heme groups.

Under severe acidic conditions, the renal tubules can convert ________ into bicarbonate. A) glutamate B) carbon monoxide C) lysine D) glutamine E) glycine

D) glutamine

Which of the following accurately describes the thick ascending limb of the loop of Henle? A) permeable to water in the presence of specific hormones B) permeable to water and does NOT contain Na+/K+/Cl- cotransporters C) impermeable to water and does NOT contain Na+/K+/Cl- cotransporters D) impermeable to water and contains Na+/K+/Cl- cotransporters E) permeable to water and contains Na+/K+/Cl- cotransporters

D) impermeable to water and contains Na+/K+/Cl- cotransporters

When an increase in hydrogen ions remains after one hour, this will lead to a(n) A) decrease in bicarbonate reabsorption in the kidneys. B) increased hydrogen reabsorption in the kidneys. C) increase in bicarbonate secretion in the kidneys. D) increase in hydrogen ion secretion from the kidneys. E) failure of the kidneys to filter any more hydrogen ions.

D) increase in hydrogen ion secretion from the kidneys.

Most filtered solutes and water are reabsorbed in what region? A) loop of Henle B) collecting duct C) distal tubule D) proximal tubule E) Bowman's capsule

D) proximal tubule

A patient is exhibiting several signs of acid-base imbalance. Blood tests reveal that blood pH is 7.5, and bicarbonate and carbon dioxide levels in blood are both low. What is the state of this patient? A) metabolic acidosis with respiratory compensation B) respiratory acidosis with renal compensation C) metabolic alkalosis with respiratory compensation D) respiratory alkalosis with renal compensation E) Blood pH is normal

D) respiratory alkalosis with renal compensation

What is the process whereby molecules are selectively transported from the peritubular fluid to the lumen of the renal tubules? A) glomerular filtration B) reabsorption C) absorption D) secretion E) excretion

D) secretion

When the solute concentration is high enough in the lumen of the renal tubules such that all of the carrier proteins or pumps for that solute are occupied, the system is operating at A) diffusional saturation. B) tubular threshold. C) renal threshold. D) transport maximum. E) tubular maximum.

D) transport maximum.

Which of the following is the first step of hemostasis? A) formation of a platelet plug B) formation of a blood clot C) formation of a fibrin clot D) vascular spasm E) secretion of histamine

D) vascular spasm

Vasoconstriction

Decrease in diameter of a blood vessel due to contraction of smooth muscle cells in its wall.

Lisa suffers from kidney stones. One small stone manages to get lodged in the ascending limb of the nephron loop of one of her nephrons. Predict what would happen to the amount of filtrate produced over time by this particular nephron.

Decrease in filtrate production due to an increase in hydrostatic pressure in the capsular space

What happens if reabsorption of a substance increases (and filtration and secretion stay the same)?

Decreased excretion of the substance and an increased amount of the substance in blood

A patient has a genetic defect that decreases the number of GLUT transporters in the nephron tubule. Which would you expect?

Decreased transport maximum and increased glucose in urine

How do the two limbs of the loop of Henle differ in their permeability to water? What makes this difference in permeability possible?

Descending limb: permeable to water but lacks transporters for salts. Ascending limb: impermeable to water but reabsorbs NaCl

Read the box on hemodialysis on p. 603 and see if you can create a model system that would work for dialysis. Draw two compartments (one to represent blood and one to represent dialysis fluid) separated by a semipermeable membrane. In the blood compartment, list normal extracellular fluid solutes and their concentrations (see the table with normal values of blood components inside the back cover of this book). What will happen to the concentrations of these solutes during kidney failure? Which of these solutes should you put in the dialysis fluid, and what should their concentrations be? (Hint: Do you want diffusion into the dialysis fluid, out of the dialysis fluid, or no net movement?) How would you change the dialysis fluid if the patient was retaining too much water?

Dialysis fluid should resemble plasma without waste substances, such as urea. This will allow diffusion of solutes and water from the blood into the dialysis fluid, but diffusion will stop at the desired concentration. To remove excess water from the blood, you can make the dialysis fluid more concentrated than plasma.

Osmosis

Diffusion of a solvent through a selectively permeable membrane from a dilute solution to a more concentrated one.

D) micelles.

Digestion products of lipases are readily exchanged in the lumen with A) cholesterol. B) polysaccharides. C) chylomicrons. D) micelles. E) low-density lipoproteins

Define the four basic processes of the digestive system and give an example of each.

Digestion—chemical or mechanical breakdown of nutrients (proteins). Absorption—transport from lumen to ECF (water). Secretion—transport from ECF to lumen (enzymes). Motility—movement of material through the digestive tract.

Diagram the tertiary active transport of organic anions to demonstrate secretion across the proximal tubule epithelium. (Fig. 19.12)

Direct active Transport - Na+/K -ATPase keeps intracellular sodium low Secondary indirect active transport - Na+ dicarboxylate cotransporter (NaDC) concentrates a dicarboxylate inside the cell using energy of sodium gradient Tertiary indirect active transport - basolateral organic anion transporter (OAT) concentrates organic anions (OA-) inside the cell, using the energy stored in the dicarboxylate gradient

An osmotic gradient develops in the renal medulla because various parts of the loop of Henle differ from each other in which of the following characteristics?

Direction of fluid movement, active transport of solutes, and permeability to water

Hormonal regulation of Na+ reabsorption occurs in which part of the nephron?

Distal convoluted tubule and collecting duct

The portion of the nephron that attaches to the collecting duct is the ____

Distal tubule

The cystic duct

Duct leading to and away from the gallbladder

Where in the GI tract would you find chyme, bicarbonate, amylase, bile, lipase, and nucleases?

Duodenum

Which of the following occurs during micturition? A) The detrusor muscles relax, muscles of the internal urethral sphincter relax, and muscles of the external urethral sphincter relax. B) The detrusor muscles contract, muscles of the internal urethral sphincter contract, and muscles of the external urethral sphincter contract. C) The detrusor muscles relax, muscles of the internal urethral sphincter contract, and muscles of the external urethral sphincter contract. D) The detrusor muscles relax, muscles of the internal urethral sphincter contract, and muscles of the external urethral sphincter relax. E) The detrusor muscles contract, muscles of the internal urethral sphincter relax, and muscles of the external urethral sphincter relax

E) The detrusor muscles contract, muscles of the internal urethral sphincter relax, and muscles of the external urethral sphincter relax

Which of the following is True concerning glucose transport across the proximal tubule? A) Glucose is normally 100% reabsorbed. B) Glucose is actively transported across the apical membrane by cotransport with sodium. C) Glucose is passively transported across the basolateral membrane by facilitated diffusion. D) Glucose transport requires energy. E) All the above statements are correct.

E) all of the above statements are correct

The regulation of water and ion excretion occurs primarily within the A) proximal tubules only. B) late distal tubules only. C) collecting ducts only. D) both proximal tubules and late distal tubules. E) both collecting ducts and late distal tubules.

E) both collecting ducts and late distal tubules.

Located entirely within the cortex, reabsorption of water and certain solutes from which region is under hormonal control? A) loop of Henle B) collecting duct C) proximal tubule D) Bowman's capsule E) distal tubule

E) distal tubule

Hemophilia is a genetic clotting disorder usually characterized by the absence of what clotting factor? A) von Willebrand factor B) factor X C) factor XIII D) thrombin E) factor VIII

E) factor VIII

Which of the following describes movement from glomerular capillaries into Bowman's capsule? A) excretion B) reabsorption C) secretion D) micturition E) filtration

E) filtration

What is the GFR/renal plasma flow equal to? A) filtered load B) renal blood flow C) excretion rate D) clearance E) filtration fraction

E) filtration fraction

Which of the four Starling forces for glomerular filtration is incorrectly paired with the direction it favors filtration or absorption (back to glomerulus)? A) glomerular capillary oncotic pressure : absorption B) Bowman's capsule oncotic pressure : filtration C) glomerular capillary hydrostatic pressure : filtration D) Bowman's capsule hydrostatic pressure : absorption E) glomerular capillary osmotic pressure : filtration

E) glomerular capillary osmotic pressure : filtration

How does severe diarrhea cause a metabolic acidosis? A) production of bicarbonate B) production of hydrogen ions C) loss of hydrogen ions D) loss of phosphate E) loss of bicarbonate

E) loss of bicarbonate

Substances that are reabsorbed from the proximal tubule enter which blood vessel? A) vasa recta B) efferent arteriole C) afferent arteriole D) glomerulus E) peritubular capillary

E) peritubular capillary

Which of the following substances is NOT secreted at any point into the renal tubules? A) choline B) creatinine C) potassium D) hydrogen E) sodium

E) sodium

Which of the following chemicals involved in the development of a platelet plug is secreted by activated platelets in response to ADP? A) arachidonic acid B) serotonin C) epinephrine D) prostacyclin E) thromboxane A2

E) thromboxane A2

When carrier proteins are saturated, they are said to be working at what? A) diminished capacity B) active reabsorptive state C) increased affinity D) increased capacity E) transport maximum

E) transport maximum

Formation of a platelet plug is initiated by ________ binding to ________. A) thrombin : platelets B) arachidonic acid : collagen C) fibrin : erythrocytes D) ADP : platelets E) von Willebrand factor : collagen

E) von Willebrand factor : collagen

In renal secretion, molecules move from the ______________ to the ______________. (Fig. 19.2)

ECF into the lumen

Which ion is a primary determinant of ECF volume? Which ion is the determinant of extracellular pH?

ECF volume - Na+pH - H+

What is the enteric nervous system, and what is its function?

ENS: network of neurons within the GI tract that can sense a stimulus, integrate information, and create an appropriate response without integration or input from the CNS. Also interacts with the CNS through sensory and autonomic neurons.

If one is acidic, you would want to _____ H+.

EXCRETE

Liver decreases its protein synthesis and no longer breaks down aldosterone effectively.

Edema (swelling)

What is an electrolyte? Name five electrolytes whose concentrations must be regulated by the body

Electrolytes are ions, which can conduct electric current through a solution. Examples: Na+, K+, Ca2+, H+, HPO4 2- and HCO3 -.

Defecation

Eliminating undigestible residues (feces)

A) increasing the surface area of the fat globule exposed to enzymes.

Emulsification aids in the digestion of fats by A) increasing the surface area of the fat globule exposed to enzymes. B) beginning the process of degrading the triglycerides within the fat globule. C) increasing the number of fat globules. D) the ability of bile salts to activate lipases that digest fats. E) increasing the number of hydrophobic residues on the surface of the fat globule.

Which of the following terms can be associated with bile?

Emulsification and amphipathic

Compare the routes by which water enters the body with the routes by which the body loses water

Entry: ingested and a small amount from metabolism. Loss: exhaled air, evaporation and perspiration from skin, excreted by kidneys, and in feces

Distinguish between epithelial transport and the paracellular pathway. (Fig. 19.8a)

Epithelial transport is trans cellular meaning we go through the epithelial cell Paracellular goes between the cells, through cell-cell junctions

The walls of the gastrointestinal tract are constituted entirely of smooth muscle, except for which of the following?

Esophagus and anus

Under what circumstances are ketone bodies formed? From what biomolecule are ketone bodies formed? How are they used by the body, and why is their formation potentially dangerous?

Excessive breakdown of fatty acids, as occurs in starvation. Can be burned as fuel by neurons and other tissues. Many ketone bodies are strong acids and can cause metabolic acidosis.

In which of these processes is fluid entering the body's external environment? [Tbl. 1.1]

Excretion

The elimination of urine from the body into the external environment

Excretion

Which of the following nutrients is specifically digested in the mouth, the stomach, and the small intestine?

Fats

Describe glomerular capillaries and their pores.

Fenestrated capillaries => large pores to allow filtration through endothelium. Small enough to prevent blood cells from entering filtrate. Negative charge on pore proteins repels negative charged plasma proteins Epithelium of Bowman's capsule surrounds glomular capillaries and has podocytes.

The first process in urine formation that pushes a fluid similar in composition to plasma into the renal tubule

Filtration

Which of these processes occurs by means of bulk flow?

Filtration

List and briefly describe the three processes of the kidney.

Filtration - movement from blood to lumen Reabsorption - from lumen to blood Secretion - from blood to lumen Excretion - from lumen to outside the body

Graphing question: You are given a chemical Z and told to determine how it is handled by the kidneys of a mouse. After a series of experiments, you determine that (a) Z is freely filtered; (b) Z is not reabsorbed; (c) Z is actively secreted; and (d) the renal threshold for Z secretion is a plasma concentration of 80 mg/mL plasma, and the transport maximum is 40 mg/min. The mouse GFR is 1 mL/min. On a graph similar to the one in question 22, show how filtration, secretion, and excretion are related. One axis will be plasma concentration of Z (mg/mL) with a range of 0-140, and the other axis will show rates of kidney processes (mg/min) with a range of 0-140.

Filtration line: Use several plasma concentrations of Z (0-140 mg Z/mL plasma)×GFR. The line will be a straight line beginning at the origin and extending upward to the right. Secretion reaches its maximum rate of 40 mg/min at 80 mg Z/mL plasma. Plot that point. Draw the secretion line from the origin to that point. Above the renal threshold, secretion rate does not change, so the line becomes horizontal. Excretion line: Add the filtration rate and secretion rate at a number of plasma concentrations of Z.

Potassium is filtered, secreted and reabsorbed by the nephrons of the kidney. In what order does the majority of each process occur?

Filtration, reabsorption, secretion

Urea in the proximal tubule can only move by diffusion. If the urea concentration of filtrate is equal to the urea concentration of extracellular fluid, how is a urea concentration gradient created? (Fig. 19.8a)

First ions such as sodium move from the lumen to the intersitial fluid, this causes water to follow changing the concentration relative to the water of urea so a urea gradient is formed and urea is reabsorbed

Dwight was competing for a spot on the Olympic equestrian team. As his horse, Nitro, cleared a jump, the footing gave way, causing the horse to somersault, landing on Dwight and crushing him. The doctors feared kidney damage and ran several tests. Dwight's plasma creatinine level was 2 mg/100 mL. His 24-hour urine specimen had a volume of 1 L and a creatinine concentration of 20 mg/mL. A second specimen taken over the next 24 hours had the same plasma creatinine value and urine volume, but a urine creatinine concentration of 4 mg/mL. How many milligrams of creatinine are in each specimen? What is Dwight's creatinine clearance in each test? What is his GFR? Evaluate these results and comment on Dwight's kidney function.

First specimen clearance=1000 L plasma/day. Normally, creatinine clearance=GFR. However, this value is not at all realistic for GFR (normal average is 180 L/day). The repeat test has 4000 mg of creatinine and gives a clearance of 200 L/day, which is within normal limits. The abnormal values on the first test were probably a laboratory error. Dwight's kidney function is normal.

You are a physiologist taking part in an archeological expedition to search for Atlantis. One of the deep-sea submersibles has come back with a mermaid, and you are taking a series of samples from her. You have determined that her GFR is 250 mL/min and that her kidneys reabsorb glucose with a transport maximum of 50 mg/min. What is her renal threshold for glucose? When her plasma concentration of glucose is 15 mg/mL, what is its glucose clearance?

For any solute that filters: plasma concentration × GFR = filtration rate. At the transport maximum: filtration rate = reabsorption rate of Tm. By substitution: plasma concentration×GFR=Tm. The renal threshold represents the plasma concentration at which the transporters are working at their maximum (Tm). By substitution: renal threshold×GFR=Tm. Mermaid's GFR is 250 mL/min and Tm is 50 mg/min, so renal threshold is 0.2 mg glucose/mL plasma. Clearance=excretion rate/plasma concentration. At 15 mg glucose/mL plasma, 3750 mg/min filter and 50 mg/min reabsorb, so 3700 mg/min are excreted.

Increase gastric activity

Function of gastrin

Make digestive enzymes

Function of pancreatic acinar cells

Produce pepsinogen and gastric lipase

Function of the gastric chief cell

Produce hydrochloric acid, intrinsic factor and activates pepsin

Function of the gastric parietal cell

Segmentation

Function of the inner circular layer of muscularis externa

Peristalsis

Function of the outer longitudinal layer of muscularis externa

Which of the following cell types found in the stomach are endocrine cells?

G

The amount of filtrate entering the proximal tubules of the kidney each minute is the

GFR

How can you determine GFR from measuring only the plasma concentration of inulin and the excretion rate of inulin?

GFR = excretion rate of inulin/ [inulin] plasma

When mean arterial pressure increases, what may increase in response?

GFR and urine volume

When the macula densa detects an increase in NaCl concentration in the renal filtrate, what happens to the glomerular filtration rate (GFR)?

GFR decreases.

What does the abbreviation GFR stand for? What is a typical numerical value for GFR in milliliters per minute? In liters per day?

GFR—glomerular filtration rate. 125 mL/min or 180 L/day.

D) lower esophageal sphincter

Gastric reflux occurs when material flows backwards through which of the following structures? A) glottis B) epiglottis C) upper esophageal sphincter D) lower esophageal sphincter E) pyloric sphincter

Which of the following statements about type 2 diabetes is true?

Genetics and obesity play a role in the development of type 2 diabetes.

How does respiratory compensation by hyperventilation work?

Gets rid of excess CO2

Name the three filtration barriers that solutes must cross as they move from plasma to the lumen of Bowman's capsule. What components of blood are usually excluded by these layers?

Glomerular capillary endothelium, basal lamina, and epithelium of Bowman's capsule. Blood cells and most plasma proteins are excluded.

If the efferent arteriole is vasoconstricted, which of the following will increase?

Glomerular capillary pressure GFR Glomerular filtration pressure All answers are correct

Define and give an average value for GFR.

Glomerular filtration rate - Volume of fluid that filters into Bowman's capsule per unit time Average 125mL/min or 180L/day

Examine the graphs of insulin and glucagon secretion in Figure 22.14c. Why have some researchers concluded that the ratio of these two hormones determines whether glucose is stored or removed from storage?

Glucagon and insulin cycle according to food intake, but both hormones are always present in some amount. So it appears that the ratio rather than an absolute amount of hormone determines the direction of metabolism.

Which of the following carbohydrates can be absorbed in the human GI tract?

Glucose

Which of the following molecules is(are) moved across the apical membranes of intestinal epithelial cells by cotransport mechanisms with sodium?

Glucose and amino acids

Barbiturate poisoning

H+ is high, pH is low, Respiratory acidosis, Renal compensation by H+ excretion

Thoracic or abdominal surgery

H+ is high, pH is low, Respiratory acidosis, Renal compensation by H+ excretion

Emphysema

H+ is high, pH is low, Respiratory acidosis, Renal compensation by H+ excretion

Diabetic ketoacidosis:

H+ is high, pH is low, metabolic acidosis, Respiratory compensation by hyperventilation

Kidney failure

H+ is high, pH is low, metabolic acidosis, respiratory compensation by hyperventilation

Prolonged vomiting

H+ is low, pH is high, Metabolic alkalosis, Respiratory compensation by hypoventilation

Hyperventilation

H+ is low, pH is high, Respiratory alkalosis, Renal compensation by H+ reabsorption

Antacid overdose

H+ low, pH is high, metabolic alkalosis, Respiratory compensation by hypoventilation

The excretion of which substances is primarily regulated in the late distal tubule and collecting duct?

H2O, H+ and Na+

Match "parietal cells" with the cell or region that secretes or contains it:

HCl

In the disease state called hemochromatosis, the hormone hepcidin is either absent or not functional. Use your understanding of iron homeostasis to predict what would happen to intestinal iron uptake and plasma levels of iron in this disease.

Hepcidin causes enterocytes to destroy ferroportin transporters. If hepcidin is absent or not functional, intestinal iron uptake cannot be down-regulated when iron levels become too high, so these patients have elevated plasma levels of iron.

Liver no longer breaks down aldosterone effectively.

High blood pressure Low plasma K+

______________ interstitial osmolarity allows urine to be concentrated.

High medullary

4

How many lobes does the liver have?

Which of the following molecules or processes can be found in both red blood cells and parietal cells in the stomach?

Hydrogen ion (H+) production Countertransport of bicarbonate (HCO3-) and chloride (C-) ions. Carbonic anhydrase enzyme All of the answers are correct

metabolic acidosis deals with ...

Hydrogen levels

Which is unlikely to be a consequence of kidney disease?

Hyperglycemia

If a person with a normal body fluid osmolarity drinks a large quantity of water, what would occur if the kidneys could not excrete any of the water?

Hypervolemia and decreased plasma osmolarity

Both insulin and glucagon are released following ingestion of a protein meal that raises plasma amino acid levels. Why is the secretion of both hormones necessary?

If a person ingests a pure protein meal and only insulin is released, blood glucose concentrations might fall too low. Glucagon co-secretion ensures that blood glucose remains within normal levels.

Dinitrophenol (DNP) is an uncoupler, or has the ability to separate the flow of electrons and the pumping of H+ ions for ATP synthesis. This means that the energy from electron transfer cannot be used for ATP synthesis. Fifty years ago, DNP was given as a drug to help patients lose weight. Why does this work? Why would this be dangerous?

If electron transport doesn't produce ATP, then much more sugar must be metabolized for energy needs. Very low production of ATP would be lethal.

If the previous person's PCO2 (carbon dioxide) was elevated, how would the type of disturbance change?

If the PCO2 levels were elevated, the person would then be suffering from respiratory acidosis. Resp. acidosis occurs when alveolar hyperventilation results in CO2 retention and ELEVATED PLASMA PCO2.

Compare and contrast the terms: c) respiratory acidosis and metabolic acidosis, including causes and compensations

In both, body pH falls below 7.38. Respiratory - results from CO2 retention (from any number of causes); metabolic - results from excessive production of metabolic acids Respiratory compensation - renal H+ excretion and HCO3- retention. Metabolic compensation - increased ventilation, renal H+ excretion, and HCO3- retention Respratory - arterial Pco2 is elevated metabolic - Pco2 usually decreased

C) emulsified by bile salts.

In order to begin the process of fat digestion, the fat globules must be A) emulsified by lipases. B) emulsified by trypsin. C) emulsified by bile salts. D) coalesced by bile salts. E) coalesced by lipase.

C) cholecystokinin

In response to the presence of food within the duodenum, the increased release of what hormone is the predominant stimulus for an increase in enzyme secretion from the exocrine pancreas? A) insulin B) gastrin C) cholecystokinin D) secretin E) glucose-dependent insulinotropic peptide

D) secretin

In response to the presence of food within the duodenum, the increased release of what hormone will predominantly stimulate the increase of bicarbonate secretion from the exocrine pancreas? A) insulin B) gastrin C) cholecystokinin D) secretin E) glucose-dependent insulinotropic peptide

B) absorbed : secreted

In the jejunum, bicarbonate is ________, while in the ileum and colon, bicarbonate is ________. A) absorbed : absorbed B) absorbed : secreted C) secreted : secreted D) secreted : absorbed E) absorbed : not transported in either direction

Net filtration pressure

In the kidney, the difference between the glomerular hydrostatic pressure, which tends to force fluid out of the glomerulus, and the two opposing forces (colloid osmotic pressure and capsular hydrostatic pressure).

A new drug was found to decrease Hepatitis B virus. The drug is an analogue of one of the nucleic acid bases of DNA and probably works by being incorporated into the virus and disrupting viral genes during viral DNA replication. However, patients in a clinical trial of the drug began to experience drastic overproduction of lactic acid and liver failure leading to death. The most likely explanation for the problem:

Incorporation of the drug into mitochondrial DNA disrupts the ability of mitochondria to make ATP.

Vasodilation

Increase in diameter of a blood vessel due to relaxation of smooth muscle cells in its wall.

Name two stimuli that increase insulin secretion, and one stimulus that inhibits insulin secretion.

Increased plasma glucose or amino acids and parasympathetic input stimulate, sympathetic input inhibits.

What causes the internal urethral sphincter to open?

Increasing pressure due to contraction of the full bladder

The electron transport chain is located predominantly in the:

Inner membrane of the mitochondria

Name the two hormones that regulate glucose metabolism, and explain what effect each hormone has on blood glucose concentrations.

Insulin decreases blood glucose and glucagon increases it.

Which of the following statements about insulin is FALSE?

Insulin promotes fat degradation.

Acid-base balance in the distal tubule and collecting duct is regulated by what epithelial cell type?

Intercalated

A) parietal cell

Intrinsic factor is produced where? A) parietal cell B) chief cell C) acinar cell D) G cell E) neck cell

Why is inulin administration an effective way of measuring renal clearance rates?

Inulin is neither secreted nor reabsorbed.

Proximal tubule processes

Isosmotic reabsorption of organic nutrients, ions, and water. Secretion of metabolites and xenobiotic molelcules

How does angiotensin II (ANG II) directly affect the kidneys?

It increases sodium reabsorption in the proximal tubule. (it also stimulates vasopressin but this is not in direct contact with kidneys)

What happens to the OH- group that results from the hydrolysis of water to create H+ for HCl secretion by parietal cells?

It is converted to bicarbonate and shuttled out of the parietal cell by an HCO3-/Cl- exchanger

Does GFR change often?

It is relatively constant

Why is the regulation of extracellular fluid osmolarity so important?

It strongly influences cell volume.

Substances that are actively secreted into the renal tubule may include __________.

K+ and H+

Starting at the kidneys, follow a drop of urine to the external enviroment (Fig. 19.1a)

Kidney => ureter => Urinary bladder => Urethra

Name two ways the kidneys alter plasma pH. Which compounds serve as urinary buffers?

Kidneys excrete or reabsorb H+ or HCO3-Ammonia and phosphates

Which of the following is most likely to be observed in a patient with compensated respiratory alkalosis?

Kidneys secrete fewer hydrogen ions

Which type of blood lipoprotein has been implicated in the formation of atherosclerotic plaques, which can cause coronary artery disease?

LDL

__________ indicates that the person is at risk for coronary heart disease.

LDL-C levels > 160 mg/dl

Explain why in anaerobic cells the ratio of pyruvate/ lactate is much less than 1 while under aerobic conditions the ratio of pyruvate/ lactate is much greater than 1.

Lactate is produced from pyruvate only under anaerobic conditions.

List the four layers of the GI tract walls. What type of tissue predominates in each layer?

Layers (lumen outward): mucosa (epithelium, connective tissue, and smooth muscle), submucosa (connective tissue), musculature (smooth muscle), serosa (connective tissue).

Mouth, pharynx, esophagus, stomach, small intestine, large intestine, liver, gall bladder, pancreas,

Layers of the alimentary canal

Approximately what percent of the total filtered volume is excreted as urine each day?

Less than 1%

Which of the following enzymes produce a product that enters the absorptive cells of the small intestine via simple diffusion?

Lipases

Which of the following enzymes produces a product that enters the absorptive cells of the small intestine via simple diffusion?

Lipases

D) chylomicrons : lacteals

Lipids are absorbed as ________ into ________. A) fatty acids and monoglycerides : mesenteric capillaries B) micelles : mesenteric capillaries C) chylomicrons : mesenteric capillaries D) chylomicrons : lacteals E) micelles : lacteals

A) lipase

Lipids are disassembled by which of the following? A) lipase B) glucoamylase C) sucrase D) chymotrypsin E) amylase

Which of the following gastrointestinal organs can also be considered endocrine glands?

Liver and pancreas

Physiology and anatomy problem-solving skills can be vital when you're the one taking a patient's history or doing the initial assessment. If you remember the functions of different organs and their anatomical relationships, you can come up with hypotheses and ask the relevant questions to test them. You will also know what to be alert for when caring for the patient later.An overweight forty-three-year-old woman had been having episodes of gripping abdominal pain after fatty meals. One day she ate French onion soup with lots of cheese and suffered severe enough pain that she called in sick. Her supervisor pointed out that she always gets sick after fatty foods. She went to the clinic and the nurse in triage took her vitals and history.The nurse noticed that the whites of her eyes were yellow and that she had tenderness on the right side of her abdomen. Blood pressure and heart rate were normal. Based on this initial assessment, what organs do you think might be involved in this woman's illness, and why?

Liver, because of the yellow coloration in her eyes, which indicates that the liver is not processing bilirubin. Gall bladder, because it stores bile and might be related to the bilirubin buildup in her blood. The combination of trouble with fats and yellow discoloration should make you think about the hepatobiliary system. Now you have a hypothesis about what's wrong with this patient and can make a direct assessment toward supporting or disproving that hypothesis rather than asking a lot of questions that may not be relevant.

Esophagus

Location of upper and lower esophageal sphincters

What factors release glucagon? What organ is the primary target of glucagon? What effect(s) do(es) glucagon produce?

Low plasma glucose or increased plasma amino acids stimulate. Primary target—liver, which increases glycogenolysis and gluconeogenesis.

Liver decreases its protein synthesis.

Low plasma proteins Low plasma osmolarity

Juxtoglomular apparatus made out of what?

Macula Densa & juxtaglomerular cells

In the juxtaglomerular complex, what cells monitor and respond to changes in the NaCl concentration in the renal filtrate?

Macula densa cells

D) sodium-linked secondary active transport

Many individual amino acids enter the epithelial cells that line the lumen of the digestive tract via what transport process? A) paracellular transport B) simple diffusion C) primary active transport D) sodium-linked secondary active transport E) facilitated diffusion

Plasma solute concentration directly or indirectly affects __________.

Mean arterial pressure, changes in plasma volume, and the amount of water in the various body fluid compartments

Define metabolic, anabolic, and catabolic pathways.

Metabolic—all pathways for synthesis or energy production, use, or storage. Anabolic—primarily synthetic; catabolic—break down large molecules into smaller ones.

________ are tiny droplets of fatty acids, monoglycerides, and bile salts

Micelles

Segmentation

Mixing foods in stomach

Which of the following molecules can be found within the membrane or interior of a micelle?

Monoglycerides bile salts cholesterol diglycerides phospholipids free fatty acids

What happens to the resting membrane potential of excitable cells when plasma K+ concentrations decrease? Which organ is most likely to be affected by changes in K+ concentration?

More K+ leaves the cell, and membrane potential becomes more negative (hyperpolarizes). The heart is most likely to be affected

Where in the GI tract would you find all of the following: bicarbonate, amylase, lysozyme, and mucus?

Mouth

Propulsion

Movement

What purposes does motility serve in the gastrointestinal tract? Which types of tissue contribute to gut motility? Which types of contraction do the tissues undergo?

Moves food through the GI tract and helps mix food with secretions. Results from contraction of longitudinal and circular muscle layers to create propulsive peristaltic movements or mixing segmental movements.

Absorption

Moving molecules across a membrane and into circulation

Mrs. F's liver is no longer producing enough plasma protein so her blood has become______________________________ to her cells. This is best illustrated by diagram number_______________

Mrs. F's liver is no longer producing enough plasma protein so her blood has become hypotonic to her cells. This is best illustrated by diagram number 3.

One of the liver's functions is the production of bile, which helps emulsify lipids in the intestinal contents so they can be effectively digested. Is there any reason to think Mrs. F's liver is not performing this function?

Mrs. F's yellowish skin and sclerae indicate that bilirubin is depositing in her skin instead of being made into bile by the liver the yellowish color caused when excessive bilirubin settles in the skin. The liver is not converting that bilirubin into bile, so Mrs. F is also suffering from lack of bile in her digestive system (leading to difficulty digesting lipids). This will also interfere with her absorption of the lipid-soluble vitamins A, D, E, and K.

In a normal kidney, the fluid that leaves the glomerulus and enters the proximal tubule may contain __________.

Na+ and K+ ions and glucose

Which of the following types of transporters found in the cells lining the collecting duct help to maintain acid-base balance by promoting H+ transport into the tubule lumen?

Na+/K+ ATPase Na+/H+ countertransporter and H+ pump H+ pump Na+/H+ countertransporter All answers are correct

If the osmotic pressure in the glomerular capillaries increased from 28 mmHg to 35 mmHg due to dehydration, would net filtration increase or decrease?

Net filtration would decrease.

Clearance of a substance is less than the glomerular filtration rate

Net reabsorption

Clearance of the substance is greater than the glomerular filtration rate

Net secretion

Clearance of the substance is the same as the glomerular filtration rate:

No net reabsorption or secretion

Ions directly regulated by the kidney include all EXCEPT which of the following?

OH- (hydroxide)

A) deacidified

Once the chyme begins to enter the duodenum, it must first be ________ before any of the enzymes in the pancreatic juices can be activated. A) deacidified B) acidified C) dehydrated D) hydrated E) phosphorylated

D) are reassembled into triglycerides and packaged by the Golgi apparatus into chylomicrons.

Once the degradation products of triglycerides have entered the epithelial cells that line the small intestines, they A) cross the basolateral membrane and enter the circulation where they will be embedded into a chylomicron. B) are reassembled into triglycerides and released into the blood where they can be incorporated into chylomicrons. C) are reassembled into triglycerides and packaged by the Golgi apparatus into micelles. D) are reassembled into triglycerides and packaged by the Golgi apparatus into chylomicrons. E) are further degraded to glycerol and a free fatty acid, which are packaged by the Golgi

List five organs and four hormones important in maintaining fluid and electrolyte balance

Organs: kidneys, lungs, heart, blood vessels, digestive tract. Hormones: vasopressin (ADH), aldosterone, atrial natriuretic peptides (ANP), RAS pathway.

What will happen to the osmotic pressure of the extracellular fluid in the body under a hyponatremia state and how could this affect the size and shape of your cells?

Osmotic pressure will decrease; cells will swell.

Para-aminohippuric acid (PAH) is used under clinical conditions to estimate and assess healthy kidney clearance function because __________.

PAH clearance rate equals the renal flow rate of the kidney it is freely filtered at the glomerulus it is not freely reabsorbed by any segment of the nephron the remnants of PAH in renal capillaries are completely secreted all answers are correct

C) glycerol and 3 free fatty acids.

Pancreatic lipase degrades triglycerides into a A) monoglyceride and 1 free fatty acid. B) triglyceride whose free fatty acids have been shortened 2 carbons at a time. C) glycerol and 3 free fatty acids. D) diglyceride and 1 free fatty acid. E) monoglyceride and 2 free fatty acids.

What role do paracrines play in digestion? Give specific examples.

Paracrines help mediate secretion and motility. Examples: serotonin (5-HT) and histamine.

Facilitated diffusion

Passive transport process used by certain large or charged molecules (e.g., Na+, glucose) that are unable to cross through the plasma membrane unaided. Involves movement through channels or movement facilitated by a transport protein.

What are Peyer's patches? M cells of the intestine?

Peyer's patches—nodes of lymphoid tissue. M cells—epithelial cells that transfer information from gut lumen to Peyer's patches.

Trematol is a metabolic poison derived from the white snake root. Cows eating this plant concentrate the poison in their milk. The poison inhibits liver enzymes that convert lactic acid to other compounds for metabolism. Why does physical exertion increase symptoms of poisoning by trematol? Why does the pH of the blood decrease in a person who has digested trematol?

Physical exertion would increase the production of latic acid by fermentation, and the build up of lactic acid decreases blood pH when liver enzymes are blocked.

Mechanical digestion

Physically chewing

Fenestrated

Pierced with one or more small openings.

Draw a section of renal tubule epithelium showing three cells joined by cell junctions. Label the apical and basolateral membranes, the tubule lumen, and the extracellular fluid. Use the following written description of proximal tubule processes to draw a model cell. The proximal tubule cells contain carbonic anhydrase, which promotes the conversion of CO2CO2 and water H+H+ and HCO −3.HCO3 −. Sodium is reabsorbed by an apical Na+-H+Na+-H+ antiporter and a basolateral Na+-K+-ATPase.Na+-K+-ATPase. Chloride is passively reabsorbed by movement through the paracellular pathway. Bicarbonate produced in the cytoplasm leaves the cell on a basolateral Na+-HCO −3Na+-HCO3 − symporter.

Place transporters as described. Cl− moves between the cells.

Ingestion

Placing in mouth (oral cavity)

Briefly describe how podocytes and mesangial cells can regulate GFR.

Podocytes change the size of glomerular filtration slits Wider slits=more surface area for filtration GFR increases Contraction of mesangial cells changes the glomerular capillary surface area available for filtration

Blood flow through the kidney includes a feature seen in only a few organs. What is it?

Portal system

Which of the following statements about the handling of potassium by the renal system is false?

Potassium leak channels are located on the apical membranes of the epithelial cells in the collecting duct.

What do you call the cells of the epithelium of the visceral layer of Bowman's capsule?

Potocytes: which have many food-like processes pedicles. Pedicles: cover the basement membrane of the capillaries leaving only tiny slits for the passage of materials.

Colloid osmotic pressure (COP)

Pressure created in a fluid by large nondiffusible molecules, such as plasma proteins that cannot move across a capillary wall. Such substances tend to draw water to them.

What are the three possible fates for ingested proteins? For ingested fats?

Proteins: protein synthesis, energy, and conversion to fat for storage. Fats: lipid synthesis, energy, and storage as fats.

The majority of reabsorption of water and solutes from the filtrate takes place in which part of the renal tubule?

Proximal tubule As much as 70% of the water and solutes are reabsorbed into the blood at the proximal tubule.

The movement of molecules from the tubule lumen into the peritubular capillaries

Re absorption

In which process can glucose transport reach saturation?

Reabsorption

Loop of Henle Processes

Reabsorption of ions in excess of water to create dilute fluid in the lumen. Countercurrent arrangement contributes to concentrated interstitial fluid in the renal medulla

Which of the following statements regarding the short and long reflexes that regulate GI function is true?

Reflexes that originate and are integrated in the enteric nervous system are called short reflexes.

Of the three buffering mechanisms in the body, which is the strongest?

Renal System Yes, although slow, the renal system is the strongest buffering system in the body. By altering the reabsorption and excretion of hydrogen ions and bicarbonate ions, the kidneys control the pH of body fluids.

What are two systems that compensate for acid-base disturbances?

Renal and respiratory

Urine is carried from the collecting duct to what structure next?

Renal pelvis

What does renin do?

Renin's primary function is to cause an increase in blood pressure, leading to restoration of perfusion pressure in the kidneys

Explain why Sally, a teenage diabetic, may experience deep and gasping breathing from not taking her insulin?

Sally's breathing has become deep and gasping because of the pH imbalance that is ketoacidosis. Metabolic acidosis = high level of H+. The respiratory system compensates for metabolic problem which is why her breathing is heavy. The respiratory system is compensating by breathing off more CO2. (this is hyperventilation)

Appetite for what two substances is important in regulating fluid volume and osmolarity?

Salt and water

The transfer of molecules from the extracellular fluid of the peritubular capillaries into the lumen of the nephron

Secretion

Describe the functional types of epithelium lining the stomach and intestines.

Secretory epithelium (endocrine and exocrine) lines the stomach; absorptive epithelium with a few secretory cells lines the intestines.

_______ involve short segments of intestine that alternately contract and relax. They are responsible for ________

Segmental contractions; mixing

Mixed

Serous and mucous saliva

The woman was given IV pain meds, and blood was drawn to assess liver and pancreatic function, given her gallstone history. Pancreatic enzymes were elevated, and the doctor diagnosed acute hemorrhagic pancreatitis. After her surgery to remove her gallbladder and clear the obstructed bile duct, the woman recovered uneventfully, and her pancreatitis resolved. Now she wants to know what this surgery has done to her ability to produce bile and digest food. What is the answer?

She can still make bile, but she can't store it, so she can't send a large amount into the duodenum to deal with a lot of fat at one time. This patient's liver is undamaged, so she can still produce bile. Without a gall bladder, however, she has no place to store the bile, and no ability to squeeze an extra amount of bile into her duodenum after eating fat. She may have some digestive problems if she eats large amounts of fat. This varies a great deal with the individual, so her best plan will be to be cautious until she discovers how much fat she can handle.

As Mrs. F stands up, you notice that she is limping. She tells you that she banged her foot against a chair several days ago, and it is still sore. When you examine it, you see a swollen, dark purple little toe on her right foot.What is the significance of this sign? What further assessment would be most appropriate for Mrs. F's bruised toe?

She may have trouble absorbing calcium, so you should check to see if the toe is broken. The liver produces bile, which is necessary for the digestion and absorption of fat and of fat-soluble vitamins. One of these is vitamin D. If Mrs. F cannot absorb vitamin D properly, she will also not be able to absorb Ca2+Ca2+ from her diet. That means that she will be likely to develop weak bones. She may have broken her toe when she stubbed it against the chair-and she might break other bones, if she is not treated.Calcium is also a key clotting factor, so inability to absorb it from her diet could be contributing to decreased clotting-leading to her spider nevi and to the extreme discoloration of her stubbed toe.

What are short reflexes? What types of responses do they regulate? What is meant by the term long reflex?

Short reflexes—mediated entirely within the ENS; regulate secretion and motility. Long reflexes—GI reflexes integrated in the CNS.

Describe the myogenic response to increased blood pressure in afferent arterioles. Include the mechanism.

Smooth muscle in arteriole wall stretches due to inc blood pressure => stretch-sensitive ion channels open => depolarization => Boltage-gated Ca2+ channels oopen => vascular sm. muscle contracts. Vasoconstriction = increased resistance = decreased blood flow = decrease filtration pressure in glomerulus

Which of the following molecules are absorbed by the intestinal epithelia by active transport?

Sodium and calcium

What is the primary driving force for glucose transport into proximal tubule cells?

Sodium concentration gradient allows secondary active transport of glucose.

During reabsorption, what causes water to move from renal tubules into the interstitial space around the peritubular capillaries?

Sodium is transported from the tubule cell to the interstitial space, and water follows by osmosis. The pumping of sodium ions from the tubular cell to the interstitial space increases the osmolarity of the interstitial space and water follows by osmosis.

Diagram how sodium moves across the apical and basolateral membranes in the proximal tubule. (Fig. 19.8b)

Sodium moves down its concentration gradient from the tubule lumen into the proximal tubule cell through (symport, antiport, or leak channels) such as NHE (sodium Hydrogen exchanger) and ENaC (epithelial sodium channel) Crosses basolateral membrane to interstitial fluid using a sodium potassium pump

The nurse asked whether she has noticed a change in her stools. The woman answered that they were hard to flush (they float) and kind of gray-looking.Why did the nurse ask about stools?

Someone with difficulty converting bilirubin into bile will have trouble digesting fats, and they will go out in the stool. if bile doesn't enter the duodenum, fat will pass through the intestines without being digested, and as a result it will pass out in the stools. The stools will be pale, perhaps foamy or liquid, because of the fat in them. And fat floats, so the stools will be hard to flush down the toilet! The clinical term for this is "steatorrhea," or fatty diarrhea.This is the sort of sign that patients may be embarrassed to bring up unless you ask them.

Liver decreases its protein synthesis and no longer makes bile effectively, decreasing the absorption of lipids and lipid-soluble vitamins.

Spider Nevi (broken capillaries)

Diagram the micturition reflex. Indicate the components of this reflex that are under conscious control. (Fig. 19.14)

Stretch receptors fire => Parasympathetic neurons fire. Motor neurons stop firing => Sm. Muscle contracts. Internal sphincter is passively pulled open. External sphincter relaxes

Nephron

Structural and functional unit of the kidney.

Solute

Substance that is dissolved in a solution.

A) sucrase

Sucrose is the substrate for which enzyme? A) sucrase B) chymotrypsin C) amylase D) lipase E) glucoamylase

Like breathing, which of the following movements can undergo both conscious and reflexive control?

Swallowing, defecation, and chewing

Which of the following digestive functions is (are) regulated by neurons in the medulla oblongata?

Swallowing, vomiting and saliva secretion

The woman was scheduled for laparoscopic cholecystectomy the next day. But that night she developed severe abdominal pain. The nurse noticed the following: She was pale, sweating, and had cool, clammy skin. Her heart rate was high and so was her blood pressure. She had no detectible bowel sounds. What part of the autonomic system appears to be activated in this client?

Sympathetic −− you can tell because the sympathetic system causes most of her signs and symptoms. This patient's pain and fear have activated her sympathetic system, the response her body uses to survive emergencies. That's what has made her pale and sweaty, increased her heart rate and blood pressure, and decreased her intestinal motility, reducing her bowel sounds.

In neural control of GFR, ___________________ (sympathetic or parasympathetic?) neurons release ______________ (ACh or norepinephrine?) onto _________ (α, β1, or β2?) receptors, causing ___________________ (vasodilation or vasoconstriction?) of renal arterioles.

Sympathetic; Norephinephrine; alpha receptors; vasoconstriction

A) gastrin

The G cells that line the lumen of the stomach are responsible for the secretion of what compound? A) gastrin B) pepsinogen C) ghrelin D) intrinsic factor E) HCl

A) facilitated diffusion

The absorption of fructose across the apical membrane of the lumenal epithelial cells occurs through what process? A) facilitated diffusion B) diffusion C) sodium-linked secondary active transport D) primary active transport E) paracellular transport

C) secondary active transport : facilitated diffusion

The absorption of glucose involves ________ across the apical membrane and ________ across the basolateral membrane. A) facilitated diffusion : facilitated diffusion B) simple diffusion : facilitated diffusion C) secondary active transport : facilitated diffusion D) secondary active transport : secondary active transport E) secondary active transport : primary active transport

A) active : chloride

The absorption of sodium throughout the small intestine is driven by ________ transport and is typically linked to the absorption of ________. A) active : chloride B) passive : bicarbonate C) active : bicarbonate D) active : glucose E) passive : glucose

Primary active transport

The active transport process in which the energy liberated from ATP is transferred directly to the carrier molecule participating in the transport.

Define a kilocalorie. What is direct calorimetry?

The amount of heat required to raise the temperature of 1 L water by 1°C. In direct calorimetry, food is burned to see how much energy it contains.

Mean arterial pressure (MAP)

The average pressure in the systemic arteries over a complete cardiac cycle.

What other compensatory responses may occur, and would they occur earlier or later than the respiratory response?

The body makes chemical buffers such as bicarbonate but these occur much later because buffers need proteins to be formed and it takes longer. Respiratory compensation is the fastest.

Which of the following problems could occur if you rehydrated a patient with cholera using a very dilute saline solution?

The brain could swell due to the disruption of blood osmolarity.

The interstitial fluid in contact with the basolateral side of collecting duct cells has an extremely high osmolarity, and yet the cells do not shrivel up. How can they maintain normal cell volume in the face of such high ECF osmolarity?

The cells concentrate organic solutes to increase their internal osmolarity.

A) parasympathetic

The central component of the long reflex pathway typically involves the ________ nervous system, which acts to promote an increase in gastrointestinal activity. A) parasympathetic B) sympathetic C) somatic D) enteric E) intrinsic

B) stomach : pepsinogen

The digestion of proteins begins in the ________ with the activation of ________. A) small intestine : trypsinogen B) stomach : pepsinogen C) mouth : pepsinogen D) small intestine : chymotrypsinogen E) stomach : procarboxypeptidase

When aldosterone causes sodium reabsorption, why doesn't water automatically follow?

The distal nephron epithelium is impermeable to water without vasopressin

Which of the following statements about the enteric nervous system is true?

The effector cells of the enteric nervous system include exocrine glands.

D) both autonomic neurons and sensory neurons in the wall of the gastrointestinal tract.

The enteric nervous system receives its inputs from A) autonomic neurons only. B) somatic motor neurons only. C) sensory neurons in the wall of the gastrointestinal tract only. D) both autonomic neurons and sensory neurons in the wall of the gastrointestinal tract. E) both somatic motor neurons and sensory neurons in the wall of the gastrointestinal tract.

Plasma

The fluid extracellular matrix of blood; consists of water, proteins, and dissolved solutes.

If you isolate mitochondria and place them in buffer with a low pH they begin to manufacture ATP. Why?

The high external acid concentration causes an increase in H+ in the inter membrane space leading to increased ATP production by ATP synthetase.

Why do patients taking loop diuretics need to take supplemental potassium

The inhibit the reabsorption of potassium as well as sodium in the loop of henle

In a respiratory acidosis the kidney would do which of the following?

The kidney would reabsorb bicarbonate and secrete hydrogen ions. when the kidney secretes hydrogen ions it also generates a bicarbonate ion, which further aids in decreasing the acidity of the plasma.

Mrs. F has a history of mild heart problems, but she has been admitted to the medical-surgical floor for management of her liver failure. She is a white-haired woman in her sixties with a puffy face, overweight, and presents with yellowish skin and sclera, mild generalized edema (swelling), spider nevi (broken capillaries), high blood pressure, increased respiratory rate, and a heart rate of 59 bpm (normal is 60-100 bpm). When you ask whether she has noticed any changes lately, Mrs. F complains about dizziness and lightheadedness. She also mentions weight gain and that her slacks are too tight. She says "They told me this disease would interfere with my digesting lipid, so I thought I'd lose weight!" How is the liver related to lipid digestion?

The liver creates bile, a soapy compound that emulsifies lipids.

Mrs. F's blood work has come back and shows that she has low plasma osmolarity and decreased levels of plasma proteins and plasma K+K+. You can see how plasma proteins are related to her liver failure, since the liver synthesizes plasma proteins. But why does she have low K+K+ levels? Which of the liver functions below is most likely to affect Mrs. F's K+K+ levels?

The liver destroys aldosterone. When you think about K+K+, you should think about aldosterone right away. Aldosterone is the hormone that activates the Na+/K+Na+/K+ exchange in the kidneys, moving 3 Na+Na+ into the blood and 2 K+K+ into the urine. The liver stops this process by destroying the aldosterone.If the liver cannot destroy aldosterone fast enough, the Na+/K+Na+/K+ exchange will continue to run, and K+K+ will be lost in the urine. This is sometimes called "potassium wasting." Because potassium controls nerve and muscle firing, this loss of potassium could cause decreased reflexes, weakness, and-finally-fatal heart arrhythmias.In addition, Mrs. F will be moving too much Na+Na+ from her urine into her blood-and because there is more solute entering her blood than her urine, water will also move from her urine into her blood, increasing her blood volume. That is why she has high blood pressure, and it may also make her edema worse.As you care for Mrs. F, you will need to be aware of these potential problems and watch carefully (so you can catch them early if they develop).

Why is maintaining osmolarity so important to the body?

The membranes of most cell types are freely permeable to water.

Glomerular filtrate

The mix of water and solutes that is filtered out of the blood into the glomerular capsule of the nephron. It is then processed by the renal tubules to form urine.

Which part of the wall of the digestive tract has transporting epithelial cells?

The mucosa

Moderate changes in blood pressure (mean arterial pressure not less than 80 mmHg or greater than 180 mmHg) generally do not affect the glomerular filtration rate due to which known local control processes within the kidney?

The myogenic response and tubule-glomerular feedback

The normal osmolarity of intracellular and extracellular fluid is 300 mOsm, which makes them______________

The normal osmolarity of intracellular and extracellular fluid is 300 mOsm, which makes them isotonic. Normal cell and plasma osmolarity is best illustrated by diagram number 1

Osmolality

The number of solute particles dissolved in one kilogram (1000 g) of water, reflects the solution's ability to cause osmosis.

Osmolarity

The number of solute particles present in one liter of a solution.

If all glucose is normally reabsorbed in the proximal tubule of the nephron, why do people with diabetes have glucose in their urine?

The plasma concentration is greater than the renal threshold.

C) cholecystokinin

The presence of food in the duodenum will cause secretion of what hormone to increase? A) gastrin B) insulin C) cholecystokinin D) secretin E) glucagon

Hydrostatic pressure

The pressure exerted by a fluid on the walls of its container.

B) simple diffusion

The products of fat degradation are absorbed in the small intestines by what transport process? A) sodium-linked secondary active transport B) simple diffusion C) primary active transport D) facilitated diffusion E) paracellular transport

Glomerular filtration rate (GFR)

The rate at which glomerular filtrate is formed by the kidney. It is usually measured in milliliters per minute.

What is the respiratory quotient (RQ)? What is a typical RQ value for an American diet?

The ratio of CO2CO2 produced to O2O2 used in cellular metabolism. Typical RQ is 0.82.

Total peripheral resistance

The resistance that the heart must overcome to push blood through the circulatory system.

B) protein digestion products and fat presence in the duodenum

The secretion of cholecystokinin is predominantly regulated by which of the following? A) carbohydrate digestion products B) protein digestion products and fat presence in the duodenum C) distention of the duodenum D) acid and amino acid content of the chyme E) opening of the pyloric sphincter

D) acid content of the duodenum

The secretion of secretin is predominantly regulated by which of the following? A) carbohydrate digestion products B) protein digestion products and fat C) distention of the duodenum D) acid content of the duodenum E) opening of the pyloric sphincter

Nephron

The structural and functional unit of the kidney that is responsible for forming urine. It consists of the glomerulus, the renal tubule, and the associated blood vessels.

Which of the following statements about the enteric nervous system is FALSE?

The support cells of the neurons within the ENS are more similar to the Schwann cells of the peripheral nervous system than to astroglia of the brain.

Which of the following statements about the enteric nervous system is false?

The support cells of the neurons within the ENS are more similar to the Schwann cells of the peripheral nervous system than to astroglia of the brain.

Filtration membrane

The three-layer barrier that covers the glomerular capillaries and filters the blood to produce glomerular filtrate. The three layers are the fenestrated endothelium, the basement membrane, and the filtration slits (including the slit diaphragms).

Secondary active transport

The transport process that uses the energy of one substance moving down its concentration gradient to transport another substance against its concentration gradient across the membrane. The driving concentration gradient is created by primary active transport. Also called cotransport.

Simple diffusion

The unassisted transport across a plasma membrane of a lipid-soluble or very small particle.

How did Mrs. Adler's kidneys handle QuikSmart?

There has been a net secretion of QuikSmart.

Which of the following statements best describes the condition (hyponatremia) that Lauren was diagnosed with?

There is a low sodium content in the extracellular fluid.

Sally goes to the emergency room (ER) seeking treatment for mild dehydration. The ER nurse mistakenly gives Sally an IV bag of solution that is very hypotonic to normal cells. Which of the following statements about the release of vasopressin reflects what would occur as a result of this mistake?

There is decreased release of vasopressin.

In a diabetic patient with high blood sugar above transport maximum, which would you expect?

There will be increased glucose in urine and increased glucose clearance.

Th e U.S. Food and Drug Administration recently approved a new class of drugs called vasopressin receptor antagonists . Predict the effect these drugs would have on renal function and describe some clinical situations or diseases in which these drugs might be useful.

These drugs decrease ADH-mediated water reabsorption. Useful in people who secrete too much vasopressin (SIADH, or syndrome of inappropriate ADH secretion) or in hyponatremia, such as the woman in this chapter's Running Problem.

What is the function of the interstitial cells of Cajal?

They are the origin of slow waves

Which of the following statements about mitochondria is false?

They contain stacked internal thylakoid membranes.

Why do patients taking loop diuretics need to take supplemental potassium?

They inhibit the reabsorption of potassium as well as sodium in the loop of Henle.

If a person has NORMAL PCO2 (carbon dioxide partial pressure) levels, high H+ (hydrogen) levels, and low pH and bicarbonate levels, what type of disturbance are they suffering from? What may cause it?

They would be suffering from metabolic acidosis. This occurs when there is an excess input of H+ that exceeds H+ excretion. Excess proteins or exercise can cause metabolic acidosis

Mucous

Thick and sticky, binding

Serous

Thin, watery, full of enzymes, used in chemical digestion

Which is NOT true of thirst?

Thirst is a physiological mechanism for maintenance of fluid and electrolyte balance. Drinking is the only normal way to replace lost water.

Which of the following is not true of thirst? Thirst is a physiological mechanism that maintains fluid and electrolyte balance. Thirst can actually increase plasma volume if it leads to ingestion of fluid. Angiotensin II stimulates thirst. High osmolarity stimulates thirst.

Thirst is a physiological mechanism that maintains fluid and electrolyte balance.

Epithelium, lamina, muscularis

Tissues in the Mucosa

Mucosa, submucosa, muscularis externa, serosa,

Tissues of the alimentary canal (deep to superficial)

B) villi : microvilli

To increase the absorptive efficiency of the small intestine, the surface area of the mucosa is increased by the presence of folds in the wall called ________, and projections of the cell membrane called ________. A) microvilli : villi B) villi : microvilli C) lacteals : rugae D) rugae : lacteals E) microvilli : lacteals

What is the primary goal of fasted-state metabolism?

To maintain adequate glucose supply for the brain.

Shunt

To redirect or divert the flow of blood (or other fluid) from its normal route to an alternate route and destination.

Diagram the micturition reflex. How is this reflex altered by toilet training? How do higher brain centers influence micturition?

Toilet training allows higher brain centers to inhibit the reflex until an appropriate time. Higher brain centers can also initiate the reflex.

List and briefly explain th e three forms of biological work.

Transport (moving molecules across membranes), mechanical work (movement of muscles), chemical work (protein synthesis).

After fats are absorbed into the enterocyte, they are reassembled and packaged for transport. Which of the following structures are found within the chylomicron?

Triglycerides Cholesterol Protein

Functionally, why does the kidney filter 180 L/day if 99% of what is filtered is reabsorbed?

Two reasons foreign substances are filtered into the tubule but not reabsorbed into the blood. Helps clear these out rapidly Filtering and either keeping or excreting ions and water is an easy way to manage their concentrations

What prevents urine from leaving the bladder?

Two sphincters Internal sphincter - continuation of bladder wall - smooth muscle - normal tone contracted External sphincter - skeletal muscle - controlled by somatic motor neurons - tonic stimulation from CNS maintains contraction except during urination

What are the two types of diabetes mellitus? How do their causes and basic symptoms differ?

Type 1: absolute lack of insulin. Type 2: cells do not respond normally to insulin. Both: elevated fasting blood glucose levels. Type 1: body uses fats and proteins for fuel. Type 2: not as severe because the cells can use some glucose.

The active transport of which of the following contribute(s) to the medullary osmotic gradient?

Urea, chloride, and potassium

In the absence of ADH, which of the following will decrease?

Urine osmolarity and aquaporin synthesis

Make a list of all the different membrane transporters in the kidney. For each transporter, tell (a) which section(s) of the nephron contain(s) the transporter; (b) whether the transporter is on the apical membrane only, on the basolateral membrane only, or on both; (c) whether it participates in reabsorption only, in secretion only, or in both

Use figures 19.8, 19.12, 20.5b, 20.7d, 20.9, and 20.17 1) ENaC (epithelial sodium channel) a) proximal tubule b) apical side only c) reabsorption 2) Na+-K+-ATPase a) proximal tubule, ascending limb b) basolateral side only c) reabsorption, secretion 3) SGLT (sodium-glucose cotransporter) a) proximal tubule b) apical side only c) reabsorption 4)NaDC (sodium-dicarboxylate cotransporter) a) proximal tubule b) both sides (into the lumen of the cell) c) secretion 5) OAT (organic anion transporter) a) proximal tubule b) basolateral side only c) secretion` 6) NKCC symporter a) ascending limb b) apical side only c) reabsorption

Enteric interneurons that are inhibitory to smooth muscle use which of the following neurotransmitters?

VIP (vasoactive intestinal peptide) both VIP and NO (nitric oxide) are inhibitory to smooth muscle.

The ____ are the long peritubular capillaries that dip in the medulla

Vasa Recta

Name the four main compensatory mechanisms for restoring low blood pressure to normal. why do you think there are so many homeostatic pathways for raising low blood pressure?

Vasoconstriction increased cardiac output water conservation by kidneys thirst If blood pressure falls too low, oxygen supply to the brain will decrease, resulting in damage or death

Which combination of changes would increase the glomerular filtration rate (GFR) the most?

Vasodilation of the afferent arteriole and vasoconstriction of the efferent arteriole

What does vasopressin do?

Vasopressin is a hormone called "anti-diuretic hormone" that is normally secreted by the pituitary gland. Vasopressin acts on the kidneys and blood vessels. Vasopressin helps prevent loss of water from the body by reducing urine output and helping the kidneys reabsorb water into the body.

How would you expect aldosterone and vasopressin levels to change if a person had also run a race and had not taken in any water and produced a copious amount of sweat of 10 mEq/L

Vasopressin would increase while aldosterone would decrease.

Suppose that a comatose patient on a respirator has a pH of 7.5. Should the ventilation rate of the patient be increased or decreased, and why?

Ventilation rate should be decreased in order to increase plasma CO2.

Suppose that a comatose patient on a respirator has a pH of 7.5. Should the ventilation rate of the patient be increased or decreased and why?

Ventilation rate should be decreased in order to increase plasma CO2. Increasing the ventilation rate might increase the O2, but this would be ineffective to correct alkalosis.

C) ileum : complexing with intrinsic factor

Vitamin B12 is absorbed in the ________ through ________. A) jejunum : complexing with intrinsic factor B) duodenum : facilitated diffusion C) ileum : complexing with intrinsic factor D) duodenum : cotransport with sodium E) jejunum : facilitated diffusion

Which of the following substances is not secreted into the filtrate in the distal convoluted tubule?

Water

What is the effect of countercurrent multiplier in the loop of Henle?

Water is reabsorbed in the descending loop of Henle.

Peristalsis

Wave like motion propelling food forward

Serous, Mucous and mixed

What are the 3 types of salivary glands

Left, right, caudate and quadrate

What are the 4 lobes of the liver?

C) contraction of the circular muscle layer and relaxation of the longitudinal muscle in the proximal segment coupled with the contraction of longitudinal muscle and relaxation of the circular muscle in the distal segment

What causes peristalsis? A) contraction of the circular muscle layer and relaxation of the longitudinal muscle at the same segment B) relaxation of the circular muscle layer and contraction of the longitudinal muscle at the same segment C) contraction of the circular muscle layer and relaxation of the longitudinal muscle in the proximal segment coupled with the contraction of longitudinal muscle and relaxation of the circular muscle in the distal segment D) relaxation of the circular muscle layer and contraction of the longitudinal muscle in the proximal segment coupled with the relaxation of longitudinal muscle and contraction of the circular muscle in the distal segment E) contraction of both the circular muscle layer and longitudinal muscle in the proximal segment coupled with the relaxation of both the circular muscle layer and longitudinal muscle in the distal segment

B) parietal

What cells function to secrete hydrogen ions into the lumen of the stomach? A) goblet B) parietal C) chief D) neck E) G

D) chief

What cells secrete pepsinogen into the lumen of the stomach? A) goblet B) G C) parietal D) chief E) neck

B) bilirubin

What degradation by-product of hemoglobin is removed from the blood by the liver, conjugated, and then secreted into the bile? A) lipoproteins B) bilirubin C) iron D) urea E) uric acid

D) both hydrogen ions and pepsinogen

What do parietal cells secrete? A) hydrogen ions only B) pepsinogen only C) intrinsic factor only D) both hydrogen ions and pepsinogen E) both hydrogen ions and intrinsic factor

The amount of bile that goes into the duodenum

What does the hepatopancreatic sphincter control?

D) lipases

What enzymes, secreted by the exocrine pancreas, degrade fats? A) proteases B) endopeptidases C) amylases D) lipases E) nucleases

Cell in the liver

What is a hepatocyte?

D) a peristaltic-like wave in the large intestine that propels the contents toward the rectum

What is a mass movement? A) a segmentation-like wave in the large intestine that causes the elimination of contents as feces B) a peristaltic-like wave in the large intestine that causes the elimination of contents as feces C) a segmentation-like wave in the large intestine that mixes the contents D) a peristaltic-like wave in the large intestine that propels the contents toward the rectum

Hepatic portal vein, hepatic artery and common bile duct

What is located in the liver?

The hepatic artery and vein enter, and the right and left hepatic bile ducts exits

What is located in the porta hepatis

Simple squamous epithelium

What is serosa made up of?

Makes gallbladder contact, increase secretion of pancreatic enzymes, decrease gastrin

What is the function of cholecystokinin?

E) mixing the chyme

What is the function of segmentation in the small intestines? A) the opening of the pyloric valve B) releasing bile from the gall bladder C) releasing pancreatic juices D) propelling the chyme E) mixing the chyme

The functional unit of the porta hepatis

What is the hepatic lobule?

Right

What is the largest lobe in the liver?

Liver

What is the longest internal organ

C) secretin

What is the main hormone that stimulates bile secretion in the liver? A) cholecystokinin B) insulin C) secretin D) glucagon E) gastrin

Opening to the liver

What is the porta hepatis?

B) reduce the volume of the chyme

What is the primary function of the colon? A) further digest fats within the chyme B) reduce the volume of the chyme C) further digest protein within the chyme D) further absorb carbohydrates E) regulate absorption of carbohydrates and amino acids

Gallbladder, pancreas and stomach

What is the target of cholecystokinin

Stomach

What is the target of gastrin

C) gastroileal reflex

What reflex stimulates the motility of the ileum in response to the presence of chyme in the stomach? A) intestino-intestinal reflex B) ileogastric reflex C) gastroileal reflex D) colonocolonic reflex E) gastrocolic reflex

D) distention of the rectum

What stimulates the defecation reflex? A) increase in osmolarity of the rectum B) peristaltic contraction of the sigmoid colon C) pressure on the internal anal sphincter D) distention of the rectum E) distention of the descending colon

B) on the apical membrane of enterocytes

Where are enzymes that breakdown disaccharides located? A) in the lumen of the small intestine B) on the apical membrane of enterocytes C) inside enterocytes D) on the basolateral membrane of enterocytes E) in the lumen of the large intestine

Large intestine

Where are the anal sphincters located?

Stomach

Where is the cardiac valve located?

It is located on the inferior and posterior surface of the liver in the RUQ.

Where is the gallbladder located

Small intestine

Where is the illeocecal valve located?

Stomach

Where is the pyloric valve located

E) muscularis externa

Where would the myenteric plexus be found? A) mucosa B) submucosa C) serosa D) lamina propria E) muscularis externa

C) amylase

Which enzyme catalyzes reactions with polysaccharides? A) sucrase B) lipase C) amylase D) chymotrypsin E) glucoamylase

D) gastrin

Which hormone stimulates gastric secretion and motility? A) inhibin B) cholecystokinin C) secretin D) gastrin E) glucose-dependent insulinotropic peptide

B) stimulation of gallbladder contraction and relaxation of the hepatopancreatic sphincter

Which of the following are mechanisms by which cholecystokinin (CCK) facilitates digestion of fats? A) stimulation of lipase and bile secretion B) stimulation of gallbladder contraction and relaxation of the hepatopancreatic sphincter C) stimulation of lipase secretion and gallbladder contraction D) stimulation of bile secretion and liver contraction E) stimulation of bile secretion and contraction of the sphincter of Oddi

A) distension of the stomach

Which of the following increases gastric motility? A) distension of the stomach B) cholecystokinin and secretin C) increased osmolarity of duodenal contents D) distension of the intestines E) increased fat content in the lumen of the duodenum

B) filtration

Which of the following is NOT a basic process of the gastrointestinal system? A) digestion B) filtration C) absorption D) secretion E) motility

D) activate vitamin B12

Which of the following is NOT a normal function of the high acidity of the stomach? A) activate pepsin B) denature proteins C) destroy foodborne bacteria D) activate vitamin B12 E) protect against illness

D) secretion of enzymes for digestion

Which of the following is NOT one of the functions of the liver? A) secretion of bile B) synthesis of albumin C) removal of aged erythrocytes D) secretion of enzymes for digestion E) synthesis and modification of hormones

C) distension of the stomach and the presence of protein digestion products in the lumen of the stomach

Which of the following is a gastric-phase stimulus for acid secretion? A) distension of the stomach and the act of swallowing B) chewing and the act of swallowing C) distension of the stomach and the presence of protein digestion products in the lumen of the stomach D) chewing, swallowing, and the presence of protein digestion products in the lumen of the stomach E) the presence of fat and protein digestion products in the lumen of the stomach

E) salivary glands, pancreas, and liver

Which of the following secretes a bicarbonate-rich fluid? A) salivary glands only B) pancreas only C) liver only D) both salivary glands and pancreas E) salivary glands, pancreas, and liver

Which set of diagnostic features offers the best information that your patient has the most common type of diabetes mellitus?

Which set of diagnostic features offers the best information that your patient has the most common type of diabetes mellitus?

D) Hydrogen : chloride

_______ ions are produced within the parietal cells and transported across the apical membrane via an active transporter while, at the same time, ________ moves across the apical membrane through ion channels. A) Hydrogen : sodium B) Bicarbonate : chloride C) Bicarbonate : hydrogen D) Hydrogen : chloride E) Hydrogen : bicarbonate

A glomerulus is

a "knot" of capillaries that lies within the Bowman's capsule.

Suppose that your company is developing a new drug that blocks aldosterone synthesis. You would expect that patients given this drug in clinical tests would respond with __________.

a higher concentration of sodium in their urine than in patients given a placebo

Suppose that your company is developing a new drug that blocks aldosterone synthesis. You would expect that patients given this drug in clinical tests would respond with __________.

a higher concentration of sodium in their urine than in patients given a placebo The absence of aldosterone would lead to greater rates of sodium than water loss in the urine, which should lead to decreased blood pressure.

A drug that blocks the action of carbonic anhydrase in parietal cells would result in ________

a higher pH during gastric digestion

acid-base balance is ___________

a pattern of slow contractions sweeping along the GI tract

What is the value of the renal plasma flow? a) 625 mL/min b) 440 mL/min c) 140 mL/min d) 125 mL/min

a) 625 mL/min

Make a table that specifies the following for each substance listed: hormone or enzyme? steroid or peptide? produced by which cell or tissue? target cell or tissue? target has what response? a) ANP b) aldosterone c) renin d) ANG II e) vasopressin f) angiotensin-converting enzyme

a) ANP - peptide from atrial myocardial cells. Causes Na+ and water excretion; inhibits ADH secretion b) Aldosterone - steroid from adrenal cortex. Increases distal nephron Na_ reabsorption and K_ excretion c) Renin - enzyme from JG cells. Converts plasma and angiotensinogen to ANG I d) ANG II - peptide hormone made from ANG I. Increases blood pressure by actions on arterioles, brain, and adrenal cortex e) vasopressin - hypothalamic peptide. Increases distal nephron water reabsorption f) ACE - enzyme on vascular endothelium. Converts ANGI to ANG II

The kidneys also function in in acid-base balance by controlling which of the following ions? a) H+ and HCO3- b) H+ and Na+ c) Cl- and HCO3- d) H+ and Cl-

a) H+ and HCO3-

Intestinal transport of the amino acid analog MIT (monoiodotyrosine) can be studied using the "everted sac" preparation. A length of intestine is turned inside out, filled with a solution containing MIT, tied at both ends, and then placed in a bath containing nutrients, salts, and an equal concentration of MIT. Changes in the concentration of MIT are monitored in the bath (mucosal or apical side of the inverted intestine), in the intestinal cells (tissue), and within the sac (serosal or basolateral side of the intestine) over a 240-minute period. The results are displayed in the graph shown here. (Data from Nathans et al., Biochimica et Biophysica Acta 41: 271-282, 1960.) Based on the data shown, is the transepithelial transport of MIT a passive process or an active process? Which way does MIT move: (1) apical to tissue to basolateral, or (2) basolateral to tissue to apical? Is this movement absorption or secretion? Is transport across the apical membrane active or passive? Explain your reasoning. Is transport across the basolateral membrane active or passive? Explain your reasoning.

a) MIT started out with equal concentrations in both solutions, but by the end of the experiment MIT was more concentrated on the serosal side. Therefore, MIT must be moving by active transport. (b) MIT moves apical to basolateral, which is absorption. (c) Transport across the apical membrane goes from bath into tissue. Tissue MIT is more concentrated than bath. Therefore, this must be active transport. (d) Transport across the basolateral membrane goes from tissue into the sac. Tissue MIT is more concentrated than sac fluid, so this must be passive transport.

Which of the following is the only substance reabsorbed in the descending limb of the loop of Henle? a) water b) sodium c) potassium d) bicarbonate

a) water

Trace the path of an erythrocyte through the kidney circulation: a) afferent arteriole b) efferent arteriole c) glomerular capillary d) vasa recta e) renal vein

a, c, b, d, e

Put the events of lipid breakdown in the correct order, starting with large fat globules: a) large fat globules b) micelles c) chylomicrons d) fat droplets e) monoglycerides

a, d, b, e, c

Dehydration may cause some ions to become concentrated. If a person was suffering from severe hyperkalemia, you would expect

abnormal cardiac rhythms.

Processes that occur within the gastrointestinal tract include __________.

absorption and digestion digestion motility secretion absorption all answers are correct

Functions of the large intestine include:

absorption of water and compaction of feces.

Which of the following is NOT considered to be one of the four basic functions of the digestive system?

acid-base balance

In response to a steak dinner, certain secretions are needed to aid digestion. What cells in the pancreas would provide these secretions?

acinar cells in response to a high fat and protein meal, CCK would be stimulated and in turn would stimulate an enzyme-rich secretion from the pancreas.

The exocrine portion of the pancreas consists of lobules called ________, which secrete ________

acini, digestive enzymes

Which is NOT true about angiotensin II?

activates parasympathetic output

In addition to waste removal and regulation of cellular osmolarity, the kidneys' other functions may include __________.

activation of vitamin D3 secretion of erythropoietin gluconeogenesis secretion of erythropoietin and gluconeogenesis all answers are correct

Secretion into the nephron is __________ because the direction is __________ the concentration gradient.

active; against

order of which blood passes

afferent arteriole->glomerulus_>efferent arteriole-?peritubular capillary

A hormone that helps to regulate the sodium ion concentration of the blood is _______

aldosterone

When the pH rises above 7.42, a state of ________ exists

alkalosis

The functions of the liver include __________.

all answers are correct

Compare and contrast the terms: b) renin, ANG II, aldosteronce, ACE

all are parts of the RAS system. Renin and ACE - enzymes; ANG II and aldosterone - hormones

Which noncarbohydrate molecules can be made into glucose? What are the pathways called through which these molecules are converted to glucose?

amino acids and glycerol, gluconeogenesis.

Write the equation that relates excretion to the three processes of the kidney. (Fig. 19.3)

amount filtered - amount reabsorbed + amount secreted = amount excreted

The enzyme that digests starch into disaccharides is _______

amylase

Where in the GI tract would you find all of the following: bicarbonate

amylase, lysozyme, and mucus?,mouth

Rugae, plicae, and villi are all features that accomplish _____

an increase in surface area

The enzyme renin is responsible for the activation of

angiotensin I

ACE converts _________

angiotensin I to angiotensin II

Hormones that influence arteriolar resistance and GFR include ______________, a potent vasoconstrictor, and ______________, which act as vasodilators.

angiotensin II; prostaglandins

When ventilation increases, what happens to arterial Pco2? To plasma pH? To plasma H+ concentrations?

arterial Pco2 decreases, pH increases, and plasma H+ concentration decreases

The structures that regulate blood flow into single capillaries within a tissue are ________.

arterioles

Where does the reabsorption of ions occur in the nephron to produce a hyposmotic solution in the tubules?

ascending limb of the loop of Henle

Which part of the nephron always has low permeability to water, regardless of hormone levels?

ascending loop of Henle The water permeability of the collecting duct is variable. It is high when ADH is present and low when ADH is absent.

Which hormone acts as a form of reflex (systemic) control to increase the glomerular filtration rate?

atrial natriuretic peptide Angiotensin II is a vasoconstrictor, which reduces the glomerular filtration rate.

What is the movement of filtered solutes and water from the lumen of the tubules to the plasma fluid compartment called? a) secretion b) reabsorption c) excretion d) filtration

b) reabsorption

High plasma PCO2 would not __________.

be a result of hyperventilation

High plasma PCO2 would NOT __________.

be a result of hyperventilation Urine would be more acidic as H+ is secreted.

Explain the term retroperitoneal

behind the peritoneal cavity, behind the peritoneum

In response to the hormone secretin, the pancreas secretes a fluid that contains ________

bicarbonate

What is the most important extracellular buffer system?

bicarbonate produced from CO2

The secretion in the large intestine consists of which of the following?

bicarbonate- and potassium-rich mucus an alkaline mucus secretion of bicarbonate and potassium protects the large intestinal wall from acids produced by resident bacteria.

When red blood cells are old or injured, the spleen breaks them down and converts the heme from their hemoglobin into

bilirubin

Which of the following substances is not normally found in filtrate?

blood cells and large particles Yes, both blood cells and large particles, such as proteins, are not allowed to filter through a healthy glomerular membrane.

Which of the following substances is not normally found in filtrate?

blood cells and large particles (blood cells and large particles, such as proteins, are not allowed to filter through a healthy glomerular membrane)

Which of the following substances is not found in normal renal filtrate?

blood cells and macromolecules

The force for glomerular filtration is the ____

blood pressure in the glomerular capillaries

The force of glomerular filtration rate is the

blood pressure in the glomerular capillaries

A rise in angiotensin II levels would result in increased ___

blood pressure, blood volume, water retention, and retention of sodium ions at the kidney. (all of these effects)

Result of respiratory alkalosis

body retains less carbon dioxide

Compare and contrast the terms: e) respiratory alkalosis and metabolic alkalosis, including causes and compensations

both - pH goes about 7.42 metabolic - may be caused by excessive ingestion of bicarbonate-containing antacids or vomiting respiratory - hyperventilation metabolic compensation - decrease ventilation, decreased renal H+ excretion, increased HCO3- excretion respiratory compensation - decreased renal H+ excretion, increased HCO3- excretion

Compare and contrast the terms: a) principal cells and intercalated cells

both are in the distal neprhon. P cells are associated with aldosterone -mediated Na+ reabsorption; I cells are involved with acid-base regulation

The disorder known as anorexia nervosa is characterized by _______.

both psychological and physiological causes

Digestion refers to the _______

breakdown of food into particles small enough to cross epithelial cells.

The surface area of the small intestine is increased dramatically by the presence of villi and the __________.

brush border

For most nutrients, which two processes are not regulated? Which two are continuously regulated? Why do you think these differences exist? Defend your answer.

bsorption and digestion; secretion and motility. By not regulating absorption and digestion, the body ensures that it will always absorb the maximum available nutrients.

Some drugs and waste products do not pass through the filtration membrane of the glomerulus into the filtrate. How do the kidneys ensure that these waste products get excreted in urine?

by utilizing secretion Secretion moves selected substances from the blood into the filtrate for excretion.

Which of the following is NOT one of the three barriers that the filtrate must cross before entering Bowman's capsule? a) capillary endothelial cell layer b) surrounding epithelial cell layer c) apical membrane of tubular cells d) basement membrane

c) apical membrane of tubular cells

Metabolic acidosis is a decrease in plasma pH due to failure of body systems to compensate for H+ ions acquired from metabolic processes or the diet or from a loss of bicarbonate ions, as from diarrhea. Compensatory mechanisms work to prevent a change in plasma pH or to bring it back to homeostasis if it decreases. Put the following compensatory mechanisms in order of fastest to slowest response: a) changes in ventilation rate b) renal transport of H+ and HCO3- c) buffering by HCO3- or HPO42-

c, a, b

Metabolic acidosis is a decrease in plasma pH due to failure of body systems to compensate for H+ ions acquired from metabolic processes or the diet or from a loss of bicarbonate ions, as from diarrhea. Compensatory mechanisms work to prevent a change in plasma pH or to bring it back to homeostasis if it decreases. Put the following compensatory mechanisms in order of fastest to slowest response: a) changes in ventilation rate b) renal transport of H+ and HCO3- c) buffering by HCO3- or HPO42-

c, a, b

Put the following enzymes involved in the digestion of proteins in the correct order: a) trypsin b) carboxypeptidase c) pepsin d) intracellular proteases

c, a, b, d

Put the following enzymes involved in the digestion of proteins in the correct order:a) trypsin b) carboxypeptidase c) pepsin d) intracellular proteases

c, a, b, d

Put the following in the appropriate sequence to indicate the pathway a substance must travel to be reabsorbed: (a) basolateral membrane of the tubular epithelial cells (b) peritubular space (c) luminal membrane of the tubular epithelial cell (d) capillary pore of the peritubular capillary

c, a, b, d

Trace the path of fluid through the nephron: (a) collecting duct (b) proximal convoluted tubule (c) glomerulus (d) loop of Henle

c, b, d, a

Most digested nutrients are absorbed into the __________ of the __________ system, delivering nutrients to the __________ (organ). However, digested fats go into the __________ system because intestinal capillaries have a(n) __________ around them that most lipids are unable to cross.

capillaries, hepatic portal system, liver, lymphatic, basement membrane (basal lamina)

Factors that favor filtration at the glomerulus include __________.

capillary blood pressure and the concentration of proteins in Bowman's capsule

The most important factor affecting the pH of plasma is the concentration of

carbon dioxide

The enzyme that catalyzes the conversion of H2O and CO2 to H2CO3 is called _________

carbonic anhydrase

Which enzyme is responsible for producing bicarbonate in pancreatic acini and in duodenal cells? Hint: It is the same enzyme that is active in erythrocytes

carbonic anhydrase

An enzyme that will digest proteins into amino acids is ________

carboxypeptidase

What are mesangial cells? (Fig. 19.5c)

cells near glomerular capillaries that have cytoplasmic bundles of actin-like filaments that enable them to contract and alter blood flow through capillaries Also secrete cytokines assoc. with immune and inflammatory processes

If sodium increases in the ECF, water will move from

cells to the ECF, and cells will shrink.

If your stomach starts growling when you smell french fries, this is an example of __________ phase control of gastrointestinal function.

cephalic

HCl secretions convert pepsinogen to the active hormone pepsin. What cells in the gastric pits produce pepsinogen?

chief chief cells produce pepsinogen, the inactive form of pepsin.

When you eat fat, the cells of your small intestine release the hormone

cholecystokinin

stimulates bile release :

cholecystokinin

After processing in the stomach, the gastric contents are referred to as _______

chyme

Which of the following would best help a clinician understand how well the kidney was able to remove a substance from the body?

clearance of the substance

Glucose and amino acids are reabsorbed in the proximal tubule by

co-transport with Sodium

Filtrate becomes urine at the end of the __________.

collecting duct

The filtrate is subject to no further modification and is therefore first called urine at the end of the __________.

collecting duct

section where chyme is processed to remove water and electrolytes, leaving waste products of digestion?

colon

List and explain the significance of the five characteristics of urine that can be found by physical examination.

color (concentration), odor (infection or excreted substances), clarity (presence of cells), taste (presence of glucose), and froth (presence of proteins)

Describe competition in a mediated transport system, using the renal secretion of penicillin and probenecid as your example.

competition occurs due to broad specificity of OAT transporters Penicillin is super good at being secreted. 80% is excreted in urine after 4 hours Probenecid is a synthetic chemical made to block OAT channels so the penicillin stays in the blood.

The juxtaglomerular apparatus __________.

consists of both granular cells and specialized epithelial cells called the macula densa is the site where the distal convoluted tubule is in contact with the nephron's afferent and efferent arterioles is important in the regulation of blood pressure and volume all answers are correct

The outer layer of the kidney is the _____________ and the inner layer is the _____________. (Fig. 19.1c)

cortex; medulla

The anatomical arrangement of the kidney that allows transfer of solutes from one blood vessel to another is called the _________

countercurrent exchange system

What solute that is normally present in the body is used to estimate GFR in humans?

creatinine

Which of the following is NOT one of the basic renal exchange processes that take place within the nephrons? a) filtration b) secretion c) reabsorption d) excretion

d) excretion

Which effect would a decrease in pH have on the amount of potassium ion in the urine?

decrease

A poison that inhibits the Na+-K+-ATPase in the proximal tubule would likely __________ in this segment.

decrease glucose reabsorption Glucose reabsorption requires a sodium gradient, which results from the activity of the Na+-K+-ATPase. The first step of glucose reabsorption is secondary active transport (symport with sodium) into the cells of the proximal tubule.

Effect of decrease in pH on the amount of potassium ion in the urine?

decrease in amount of potassium in urine

In a normal kidney, which of the following conditions would cause an increase in GFR?

decrease in the concentration of plasma proteins in the cells

Poisoning the Na-K-ATPase in the proximal tubule would likely __________.

decrease the amount of glucose reabsorption in this segment

Relative to temperature regulation in humans, "shivering thermogenesis" would likely occur at the same time as _______.

decreased blood flow to the skin

During prolonged starvation, the central nervous system utilizes ketone bodies released from the liver's oxidation of fatty acids as fuel molecules. This increases the likelihood of _______.

decreased blood pH due to ketoacidosis

Dehydration leads to ________ blood volume, ________ blood pressure, and ________ osmolarity

decreased; decreased; increased

When filtrate flow through the renal tubule increases, reabsorption of sodium chloride by the tubule

decreases

Hyponatremia _____the secretion of ADH/vasopressin and ____ the secretion of aldosterone.

decreases; increases

Angiotensin II alters kidney function by __________.

decreasing the glomerular filtration rate and increasing sodium reabsorption along the distal tubules Neither of these activities is under hormonal regulation in these locations.

During the fasting state, the central nervous system _______.

depends on gluconeogenesis by the liver and kidneys

The ________ limb of the loop of Henle is permeable only to ________, which is reabsorbed all along the length because the fluid outside is ________ concentrated deeper in the medulla.

descending; water; more

What are the symptoms associated with a cholera infection?

diarrhea and dizziness

Water reabsorption is controlled by hormones in the __________.

distal convoluted tubule and collecting duct ********FURTHER EXPLANATION OF THE ANSWER ****** Water reabsorption is controlled by hormones in the distal convoluted tubule and the collecting duct. Anti-diuretic hormone (vasopressin) is released from the posterior pituitary gland in response in low blood pressure or elevated blood osmolarity. This hormone stimulates the insertion of aquaporins in the apical membrane of the principal cells of the late distal convoluted tubule and collecting duct, which permits water to exit the filtrate. Aldosterone is released from the adrenal cortex and stimulates sodium reabsorption and potassium secretion along the distal convoluted tubule and the collecting duct. Due to the movement of these ions, the osmolarity of the peritubular space increases, this also promotes water reabsorption. No hormones influence water reabsorption in the proximal convoluted tubule. No hormones influence water reabsorption in the loop of Henle. Aldosterone and anti-diuretic hormone stimulate water reabsorption from the distal convoluted tubule. There is a better answer. Aldosterone and anti-diuretic hormone stimulate water reabsorption from the collecting duct. There is a better answer. No hormones influence water reabsorption in the loop of Henle. Aldosterone and anti-diuretic hormone stimulate water reabsorption from the distal convoluted tubule.

Distal nephron composition and processes

distal tubules + collecting duct Regulated reabsorption of ions and water for salt and water balance and pH homeostasis

Motility in the small intestine is increased by __________.

distension and gastrin

During defecation, ________

distension in the rectal wall activates a spinal reflex

Normal removal of excess water in urine is known as

diuresis

Filtration ______________ (does or doesn't?) exhibit saturation. (Fig. 19.10a)

does not

ACE inhibitors

drugs that treat hypertension that prevent the conversion angiotensin I -> angiotesion II

What is the first area that food contacts as it moves from the stomach to the small intestines?

duodenum (it goes duodenum > jejunum > ileum)

The three sections of the small intestine, in order according to movement of its contents, are __________

duodenum, jejunum, ileum

Put the following events in the correct order: a) Angiotensin I is converted to angiotensin II. b) Renin is secreted by the granular cells. c) Aldosterone is released from the adrenal glands. d) Renin converts angiotensinogen into angiotensin I. e) Cells in the macula densa detect a decrease in the flow of fluid (or ion concentration) in the distal tubule.

e,b,d,a,c

An obstruction in a glomerulus would affect the flow of blood into the

efferent arteriole

An explorer has been lost in the desert for two days with very little water. As a result, you would expect to observe ______

elevated vasopressin levels

Which of the following terms can be associated with bile?

emulsification and amphipathic

bile salts aid in the digestion of fats by _______ large fat droplets.

emulsifying

Materials which flow into the glomerulus are filtered thru what?

endothelial -capsular membrane

Short reflexes of the digestive system are integrated in the ________ nervous system.

enteric

Regulation of the GI tract may involve the ____________ system.

enteric nervous endocrine enteric nervous and autonomic nervous autonomic nervous central nervous all answers are correct

The gastric phase of gastric secretion is triggered by the ___________

entry of food into the stomach

In response to the hormone cholecystokinin, the pancreas secretes a fluid that contains _______

enzymes

Normal plasma glucose concentrations are __________ (>, =, or < ? ) the renal threshold for glucose. Therefore, normally all glucose filtered is ______________ (excreted, secreted, or reabsorbed?). (Fig. 19.10b)

equal; reabsorbed

All of the following will normally be found in the filtrate EXCEPT

erythrocytes

Which of the following sequences is listed in the correct anatomical order?

esophagus > gastric antrum > pyloric sphincter > duodenum > ileum

The walls of the gastrointestinal tract are constituted entirely of smooth muscle

except for which of the following?,esophagus and anus

The plasma can gain or lose water and/or solutes by __________.

exchange with the lumen of the GI tract, respiration, and exchange with extracellular connective tissue, such as bone

When the plasma concentration of a substance exceeds its renal concentration, more of the substance will be ..

excreted.

Which equation correctly describes the relationship between excretion, filtration, reabsorption, and secretion?

excretion = filtration - reabsorption + secretion

In and around the urinary bladder, skeletal muscle can be found in the __________.

external urethral sphincter

Myogenic response is the intrinsic ability of vascular smooth muscle to respond to pressure changes

false

Decreased levels of bile salts in the bile would interfere with digestion of ______________

fat

Salivation is controlled almost entirely by the nervous system. Which of the following stimuli would inhibit salivation?

fear Yes, fear, sleep, fatigue, and dehydration all inhibit salivation.

Which of the following leads to the lowest water loss during a day?

feces

Long reflexes that originate completely outside the digestive system include ________ reflexes and ________ reflexes, which are called ________ reflexes

feedforward, emotional, cephalic

What is the function of the renal portal system? (Do you remember the other two portal systems in the body?)

filter fluid out of the blood and into the lumen of the nephron at the glomerular capillaries, then to reabsorb fluid from the tubule lumen back into the blood at the peritubular capillaries

GFR = 130 mls/min PX = 25 mmole/L UX = 125 mmole/L V = 4 mls/min From the above data, we can determine conclusively that substance X is __________.

filtered and reabsorbed

The rate of excretion is equal to __________.

filtered load + secretion rate - reabsorption rate

What is the term for the movement from the glomerulus to the nephron lumen?

filtration

How do you calculate the filtration rate (filtered load)?

filtration load of X = [X] plasma x GFR

What two things affect excretion rate

filtration rate of the substance and whether the substance is reabsorbed

When can you use a clearance rate to estimate GFR?

for any substance that is freely filtered but neither reabsorbed nor secreted, its clearance is equal to FGR

The primary products of protein digestion are ________, ________, and ________

free amino acids, dipeptides, tripeptides

Contraction of the forces the into the where it emulsifies fats and aids in their digestion.

gallbladder bile duodenum

To achieve coordinated activity within a given section of the small intestine, adjacent smooth muscle cells have ________.

gap junctions

To achieve coordinated activity within a given section of the small intestine, adjacent smooth muscle cells have __________.

gap junctions

stimulates insulin release :

gastric inhibitory peptide

Which hormone stimulates the release of insulin in response to glucose in the intestinal lumen?

gastric inhibitory peptide (GIP)

G cells of the stomach secrete __________

gastrin

Of the currently known enterogastrones (GI hormones), only one, ___________, stimulates gastric motility and secretion.

gastrin

Secreted by cells in the stomach :

gastrin

Hormones secreted directly by the gastrointestinal tract (excluding the secondary organs) include __________.

gastrin, secretin, and cholecystokinin (CCK)

A high-protein meal stimulates release of ______, which in turn stimulates secretion of ____________.

gastrin; hydrochloric acid

A high-protein meal stimulates release of __________, which in turn stimulates secretion of __________.

gastrin; hydrochloric acid

Organs involved in calcium balance include __________.

gastrointestinal tract skin kidneys bones and kidneys bones all answers are correct

How does renal compensation by H+ excretion work?

gets rid of excess H+

Three filtration barriers

glomerular capillary endothelium, basal lamina, epithelium of Bowman's capsule

The amount of filtrate entering the proximal tubules of the kidneys each minute is the ________ rate.

glomerular filtration

net glomerular filration pressure

glomerular hydrostatic pressure - ( plasma oncotic pressure + fluid pressure in bowman)

Glomerular filtration is a process of bulk flow driven by

glomerular hydrostatic pressure.

Glomerular filtration rate is determined by the net filtration pressure and the filtration coefficient. Which two factors determine the filtration coefficient?

glomerular surface area and the permeability of the filtration barrier

Where does filtration take place in the kidneys?

glomerulus

In kidney renal tubules, all of the following substances normally are excreted into urine as it forms except __________.

glucose

Which of the following carbohydrates can be absorbed in the human GI tract?

glucose

Which of the following molecules is(are) moved across the apical membranes of intestinal epithelial cells by cotransport mechanisms with sodium?

glucose and amino acids

List some molecules that are transported using Na+-linked secondary active transport.

glucose, amino acids, ions, organic metabolites

Normally found in the filtrate

glucose, potassium, urobilinogen

The excretion of glucose in the urine is called

glucosuria

In what forms is excess energy stored in the body?

glycogen and adipose tissue fat

Which of the following is a pair of processes that typically occur at the same time?

glycogenolysis and lipolysis

What is the point of secreting a substance in addition to filtering it?

helps with homeostatic regulation of those ions or organic compounds. Makes excretion more effecient

Red blood cells contain the protein

hemoglobin

Most absorbed nutrients first enter the blood of the ______ system

hepatic portal

What would cause osmoreceptors in the hypothalamus to shrink and what effect would that have?

high plasma osmolarity, release of vasopressin

the _____ the pH concentration, the lower the ____.

higher ; H+

The functions of the liver include __________.

hormone secretion, protein synthesis and dysfunctional or aged red blood cell destruction hormone secretion dysfunctional or aged red blood cell destruction cholesterol synthesis protein synthesis ALL THE ANSWER ARE CORRECT

The parietal cells of the stomach synthesize and secrete __________, and its chief cells synthesize and secrete __________.

hydrochloric acid; pepsinogen

The ability of the nephron to filter substances based on size and charge is NOT due to the __________.

hydrostatic pressure

The ability of the nephron to filter substances based on size and charge is not due to the __________.

hydrostatic pressure

What is the primary driving force that produces glomerular filtration?

hydrostatic pressure of blood (blood pressure)

What is the primary driving force (pressure) that produces glomerular filtration?

hydrostatic pressure of blood (blood pressure) Yes, the hydrostatic pressure of blood forces fluid out of the glomerular capillaries.

Which of the following is UNLIKELY to be a consequence of kidney disease?

hyperglycemia

An increase in plasma potassium levels is properly called _________

hyperkalemia

The ion imbalance known as ________ initially leads to ________ in excitable cells

hyperkalemia; depolarization

An ACE (angiotensin converting enzyme) inhibitor would be used to treat __________.

hypertension since ACE inhibition will result in decreased secretion of anti-diuretic hormone and aldosterone

An ACE (angiotensin-converting enzyme) inhibitor would be used to treat __________.

hypertension, since ACE inhibition will result in decreased secretion of vasopressin The ACE inhibitor might actually further reduce the blood pressure, and the lack of angiotensin II should decrease, not increase, the secretion of vasopressin.

Osmoreceptors are located in the ____

hypothalamus

The primary osmoreceptors are located in the _____

hypothalamus

How can you determine renal handling of a substance by comparing its clearance rate to the clearance rate of inulin or creatinine?

if less of the substance appears in the urine than was filtered, net reabsorption occurred If more substance appears in the urine, it was filtered If the substance is handled like inulin, it is neither reabsorbed nor secreted

In which non-digestive function does the digestive tract play a surprisingly large role?

immunity

Atrial natriuretic peptide

increase GRF and inhibits the release of renin

Intestinal crypts ___________________

increase the surface area of the mucosa of the small intestine and produce new cells for the mucosa of the small intestine

The effects of angiotensin II on the central nervous system are to __________.

increase thirst and increase the cardiac output and peripheral vasoconstriction Angiotensin II increases vasopressin secretion.

The effects of angiotensin II on the central nervous system are to __________.

increase thirst, increase cardiac output, and cause peripheral vasoconstriction

Dehydration triggers a/an _____ in osmolarity and secretion of ______.

increase; ADH/vasopressin

In response to a loss of blood, the sympathetically-induced (increase / decrease) in renal vascular resistance will cause glomerular filtration rate to (increase / decrease).

increase; decrease

Which of the following directly caused Lauren's weight gain?

increased absorption of water in the digestive system and an increased reabsorption of water by the collecting duct

In response to a rapid increase of organic acid in the body, you would expect to observe __________

increased alveolar ventilation

Aldosterone secretion increases in response to ________ and causes ________.

increased angiotensin II; sodium reabsorption

In skeletal muscle fibers, arrival of the pancreatic hormone insulin causes _______.

increased glucose transport into skeletal muscles

Saliva secretion is primarily a result of ______

increased parasympathetic sitimulation

Diagram how GFR is controlled by the resistance at the renal arterioles. What happens to GFR when resistance increases or decreases at afferent arterioles? Efferent arterioles? (Fig. 19.6d, e)

increased resistance at afferent arteriole = hydrostatic pressure decreases on glomerular side = decrease GFR Increased resistance at efferent arteriole = blood dams up in front of the constriction = hydrostatic pressure increases = increase GFR

Which of the following would NOT result in an increase in GFR?

increased resistance in the afferent arteriole

Which of the following would not result in an increase in GFR?

increased resistance in the afferent arteriole

Which of the following is appropriately linked to increased renin secretion?

increased sodium reabsorption in the distal nephron Angiotensinogen is made constitutively by the liver, whether or not renin is present.

Which of the following factors does NOT stimulate insulin secretion?

increased sympathetic activity

High ECF K+ stimulates __________.

increased synthesis and secretion of aldosterone

High ECF K+ stimulates __________.

increased synthesis and secretion of aldosterone Vasopressin does not influence potassium secretion.

If a patient had damage to the posterior pituitary that resulted in decreased secretion of vasopressin, which of the following would you expect to happen?

increased urine output and decreased blood volume

Filtrate flow through the renal tubule increases when glomerular filtration rate

increases

When filtrate flow through the renal tubule increases, the concentration of sodium chloride remaining in the filtrate

increases

Atrial natriuretic peptide ________

increases GFR (glomerular filtration rate) and inhibits release of renin

Because fats are excluded by aqueous solutions, they are difficult to digest. Fat droplets entering into the small intestine need to be modified so that they don't join together and form larger, indigestible structures. Bile salts found in bile secreted by the liver keep the fat droplets apart in small, stable structures that can be more easily accessed by digestive enzymes. What is the role of bile?

increases the surface area of fats so that they are easier to digest

Which of the following best describes the direct effect of vasopressin?

increases water reabsorption at the collecting duct of the kidney

In a dehydrated person, the kidneys compensate for changes in extracellular fluid volume and osmolarity by __________.

increasing the amount of water that is reabsorbed from the collecting duct A dehydrated person would not benefit from making more filtrate because that would likely increase the volume of excreted urine and worsen the dehydration.

When insufficient water intake leads to dehydration, compensatory mechanisms to preserve plasma volume and homeostatic osmolarity include __________.

increasing the amount of water that is reabsorbed from the filtrate in the collecting duct

What causes the internal urethral sphincter to open?

increasing urine pressure due to contraction of the full bladder

Angiotensin II leads to an alteration in kidney function by __________.

increasing water reabsorption from the collecting ducts and increasing sodium reabsorption along the distal tubules

How is cholera transmitted from the environment to the human body?

ingested through the digestive system and brought into the enterocyte

When venous return is increased, stretch receptors in the atria of the heart are activated. This results in

inhibition of ADH secretion

When baroreceptors in the carotid and aortic bodies register increased blood pressure, this results in

inhibition of vasopressin secretion

When venous return is increased, stretch receptors in the atria of the heart are activated. This results in ____

inhibition of vasopressin secretion

What is the mechanism of action of vasopressin?

insertion of AQP-2 into the apical membrane of collecting duct cells

What is the mechanism of action of vasopressin?

insertion of AQP-2 into the apical membrane of distal tubule cells

Most body water is located

inside cells

The pancreatic hormone that is more prevalent after a meal is __________, whereas the pancreatic hormone more characteristic of the fasting state is __________.

insulin; glucagon

A blockage of the ducts from the parotid glands would _______

interfere with carbohydrate digestion in the mouth

________ are pacemakers for slow wave activity

interstitial cells of Cajal

During which phase in the control of the digestive system would bicarbonate and bile be stimulated?

intestinal phase food in the intestines initiates a reflex that stimulates secretions of bicarbonate, digestive enzymes, and bile.

The clearance of __________ is the most precise way to measure GFR.

inulin

In a normal kidney in a healthy individual, glucose __________.

is freely filtered at the glomerulus

Fluid entering Bowman's capsule is nearly ________________ osmotic with plasma.

isoosmotic about 300 mOsM

What is the osmolarity of the filtrate at the end of the proximal tubule?

isotonic - 300 mOsm Yes, the osmolarity of the filtrate would be about 300 mOsm because both solutes and water are reabsorbed in the proximal tubule. As particles are reabsorbed, water follows osmotically.

How does respiratory compensation by hypoventilation work?

it keeps in CO2

Granular cells

juxtaglomerular cells or JG cells secrete renin - an enzyme involved in salt and water balance

How does renal compensation by H+ reabsorption work?

keeps in H+

Abnormal fat and amino acid metabolism may lead to the condition called ______

ketoacidosis

Filtration an reabsorption of blood occurs in the

kidney

Urine is produced by the ___?

kidney

Excess potassium ions are eliminated from the body by the ________

kidneys

The organ(s) that regulate water loss to maintain water balance is (are) the __________.

kidneys

The ________ is a significant site of absorption of water and electrolytes, but NOT of nutrients

large intestine

Why is a decrease in GFR when blood pressures fall below normal an adaptive response?

less blood filtered means less chance of water loss in the urine

Enzymatic digestion of fats involves ________, which breaks down ________

lipase; triglycerides

Which of the following gastrointestinal organs can also be considered endocrine glands?

liver and pancreas

Blood draining from the intestines flows directly to the ________

liver for filtering

Bile is produced in the ________ and stored in the ________

liver; gallbladder

Describe tubuloglomerular feedback as a result of increased GFR. What role does NaCl play in the tubuloglomerular feedback mechanism? (Fig. 19.7)

local control pathway - fluid flow through tubule influences GFR Distal most portion of distal tubule passes through afferent and efferent arterioles and communicates called the juxtaglomerular apparatus consists of macula densa and granular cells NaCl past the macula densa inc due to increased GFR macula densa sends a paracrine message to afferent arteriole to constrict and decrease GFR

Digestive reflexes originating in the CNS are called ________ reflexes

long

The hairpin-shaped segment of the nephron is the

loop of Henle

The segment of the nephron between the proximal and distal tubules that loops down into the medulla of the kidney and returns back to the cortex is called the ________

loop of Henle

Structure that is not a part of the blood circulation through the kidney?

loop of henle

Stimuli for the activation of the RAS pathway include: ____

low blood pressure in arterioles in the nephron and a decrease in fluid flow through the distal tubule.

Stimuli for the activation of the Renin Angiotensin pathway include

low blood pressure in the arterioles in the nephron and decrease in fluid flow through the distal tube

Filtrate leaving the loop of Henle is __________ in volume and __________ to the fluid entering the loop.

lower; hypo-osmotic

Arterial blood pH is regulated by the __________.

lungs and kidneys

Where do chylomicrons go after they are transported out of the enterocyte, and why do they go there?

lymphatic system, because they are too large to enter into the blood

Most products of fat digestion are absorbed by ___________

lymphatic vessels

The largest collection of ________ tissue in the body is the gut-associated lymphoid tissue (GALT)

lymphoid

The signals controlling ADH release come from __________.

macula densa, osmoreceptors in the hypothalamus, and arterial baroreceptors

The primary function of the vasa recta is to __________.

maintain the medullary concentration gradient

In the mouth, amylase digests starch to ______, which is further digested to _____ in the small intestine by the action of the enzyme ________.

maltose; glucose; maltase

In the mouth, amylase digests starch to __________, which is further digested to __________ in the small intestine by the action of the enzyme __________.

maltose; glucose; maltase

By the end of the proximal tubule, approximately 70% of the water and solutes have been reabsorbed. However, the fluid in the tubule is still iso-osmotic to plasma. This is because within the proximal tubule __________.

many solutes are actively transported, and water follows the solutes by passive diffusion

What is plasma osmolarity?

measurement of the body's electroltye-water balance. measure of the hydration status.

Conditions inside the GI tract are monitored by __________.

mechanoreceptors, chemoreceptors, and osmoreceptors

The swallowing center in the central nervous system that coordinates the muscular reflexes is located in the ________.

medulla oblongata of the brain stem

The swallowing center in the central nervous system that coordinates the muscular reflexes is located in the __________.

medulla oblongata of the brain stem

The swallowing center in the brain, which coordinates the muscular reflexes, is located in the ____________

medulle oblangata

Diarrhea can lead to which acid/base disturbance? Assuming compensation, would it be a metabolic or respiratory compensation?

metabolic acidosis with respiratory compensation the loss of bicarbonate ions, this is a metabolic acidosis. If compensated, it would be a respiratory compensation.

A patient is admitted to the hospital with the following plasma values: pH = 7.2, pCO2 = 25 mmHg, and HCO3 −− = 18 mEq/L. What is the acid base imbalance?

metabolic acidosis with respiratory compensation the low bicarbonate indicates this is a metabolic acidosis. Since CO2 is below the normal range, this indicates there is a respiratory compensation.

Starvation would cause which of the following acid-base conditions? Also, determine what type of compensation (metabolic or respiratory) there would be.

metabolic acidosis with respiratory compensation this is known as a ketosis (due to the breakdown of fat for metabolism), and the respiratory system would increase respiration to compensate for the excess hydrogen ions.

Prolonged vomiting of the stomach's contents can result in _______

metabolic alkalosis

Vomiting will cause which type of acid-base disturbance?

metabolic alkalosis Yes, loss of HCL acid from the stomach would cause a metabolic alkalosis.

Ingesting too much antacid would cause which of the following acid-base disturbances?

metabolic alkalosis ingestion of an antacid would increase the bicarbonate, causing a metabolic alkalosis.

A patient is admitted to the hospital with the following plasma values: pH = 7.5, pCO2 = 45 mmHg, and HCO3 −− = 30 mEq/L. What is the acid-base imbalance?

metabolic alkalosis with no compensation since HCO3¯ is 30 (above the normal range of 22 to 26 mEq/L), this is definitely a metabolic alkalosis. There is no respiratory compensation since CO2 is within the normal range.

Which is NOT an important route of water loss from the body?

metabolic breakdown of water molecules

The sum of all of the biochemical processes going on within the human body at any given instant is called _______.

metabolism

Describe nephrons and distinguish between cortical nephrons and juxtamedullary nephrons. (Fig. 19.1f-i)

microscopic tubules cortical nephrons - are completely within the cortes Juxtamedullary nephrons - dip into the medulla

Formal term for urination

micturition

A buffer ________

moderates changes in pH

macula densa

modified tubule epithelium in the juxtaglomerular apparatus

smooth muscle of duodenum is a target of ________

motilin

Carbohydrate digestion begins here:

mouth

Chemical digestion of food begins in the _______

mouth

Which of the following is the sequence of layers from the lumen to the outer wall of the digestive tract?

mucosa, submucosa, muscularis externa, serosa

Contraction of the ________ alters the surface area by moving villi

muscularis mucosae

What is the primary stimulus for natriuretic peptide release?

myocardial cell stretch

The kidneys filter approximately 180 L of plasma per day, but only excrete about 1.5 L of urine in that time, regardless of most changes in mean arterial pressure. This is because of __________.

myogenic regulation sympathetic innervation to the afferent and efferent arterioles tubular reabsorption all answers are correct *******FURTHER EXPLANATION FOR EACH ANSWER**** The kidneys filter approximately 180 L of plasma per day, but only excrete about 1.5 L of urine in that time, regardless of most changes in mean arterial pressure. This is because of tubular reabsorption, myogenic regulation and sympathetic innervation to the afferent and efferent arterioles. First, tubular reabsorption occurs when fluid moves from the nephron tubules to the peritubular capillaries and finally, the circulation. This process is critical in preventing the loss of filtered water, ions and molecules in the urine. Second, myogenic regulation of GFR occurs through a similar mechanism that regulates blood flow in other areas of the body. The smooth muscle in the afferent arteriole monitors stretch and responds by contracting. If there is an increase in blood pressure, the arteriole will constrict, which prevents blood from entering the glomerulus. As a result, the glomerular capillary pressure and GFR remain constant. A similar mechanism occurs when blood pressure decreases. Third, sympathetic input on smooth muscle in the afferent and efferent arterioles causes them to contract, which increases the resistance in the vasculature of the kidney, preventing blood from entering the glomerulus. As a result, glomerular filtration rate decreases. The increase in resistance in the renal vasculature decreases renal blood flow and promotes total peripheral resistance, thereby increasing mean arterial blood pressure. Thus, sympathetic input works to maintain mean arterial blood pressure despite factors that would alter arterial blood pressure. Independent of changes in mean arterial pressure, the kidneys filter approximately 180 L of plasma per day, but only excrete 1.5 L of urine in that time. One factor that contributes to this phenomenon is tubular reabsorption. Tubular reabsorption occurs when fluid moves from the nephron tubules to the peritubular capillaries and finally, the circulation. This process is critical in preventing the loss of filtered water, ions and molecules in the urine. There is a better answer. Independent of changes in mean arterial pressure, the kidneys filter approximately 180 L of plasma per day, but only excrete 1.5 L of urine in that time. One factor that contributes to this phenomenon is myogenic regulation of GFR. This regulation of GFR occurs through a similar mechanism that regulates blood flow in other areas of the body. The smooth muscle in the afferent arteriole monitors stretch and responds by contracting. If there is an increase in blood pressure, the arteriole will constrict, which prevents blood from entering the glomerulus. As a result, the glomerular capillary pressure and GFR remain constant. A similar mechanism occurs when blood pressure decreases. There is a better answer. Independent of changes in mean arterial pressure, the kidneys filter approximately 180 L of plasma per day, but only excrete 1.5 L of urine in that time. One factor that contributes to this phenomenon is sympathetic input on renal vasculature. This input on smooth muscle in the afferent and efferent arterioles causes them to contract, which will increase the resistance in the vasculature of the kidney, preventing blood from entering the glomerulus. As a result, glomerular filtration rate decreases. The increase in resistance in the renal vasculature decreases renal blood flow and promotes total peripheral resistance, thereby increasing mean arterial blood pressure. Thus, sympathetic input works to maintain mean arterial blood pressure despite factors that would alter arterial blood pressure. There is a better answer.

What proteins are in the filtration slits of podocytes

nephrin and podocin

The functional unit of the kidney is the ____

nephron

List the major structures of the urinary system in their anatomical sequence, from the kidneys to the urine leaving the body. Describe the function of each structure.

nephrons through ureters to urinary bladder (storage), leaving through the urethra

If the osmotic pressure in the glomerular capillaries increased from 28 mm Hg to 35 mm Hg, would net filtration increase or decrease?

net filtration would decrease (because osmotic pressure opposes filtration, increasing osmotic pressure would decrease net filtration)

If the osmotic pressure in the glomerular capillaries increased from 28 mm Hg to 35 mm Hg, would net filtration increase or decrease?

net filtration would decrease Yes, because osmotic pressure opposes filtration, increasing osmotic pressure would decrease net filtration.

If clearance of a substance is greater than GFR, the substance is __________.

net secreted by the kidney

Which of the following is the correct classification of vasopressin?

neurohormone

Bicarbonate secretion ________

neutralizes acid entering from the stomach into the duodenum and is secreted by apical Cl--HCO3- exchanger

In normal kidneys, blood cells and plasma proteins are

not filtered

The hormone that regulates water reabsorption by the kidneys (vasopressin).....

only increases water permeability in certain portions of the kidney tubules

Cell volume (and therefore cell function) in most cells is dependent upon careful regulation of _____

osmolarity of extracellular fluid

cell volume dependent on careful regulation of

osmolarity of extracellular fluid

Water reabsorption by the kidneys is a result of

osmosis

Water reabsorption in the kidneys is a result of

osmosis

The higher the H+ concentration, the lower the _____

pH

Indicate the pH, HCO3- (bicarbonate), and PCO2 levels when ketoacidosis has just developed:

pH = decreased HCO3- = decreased PCO2 = unchanged

Indicate the pH, HCO3- (bicarbonate), and PCO2 levels when respiratory compensation occurs:

pH = increased HCO3- = decreased PCO2 = decreased

List and briefly explain three reasons why monitoring and regulating ECF pH are important. What three mechanisms does the body use to cope with changing pH?

pH alters protein structure (enzyme activity, membrane transporters, neural function). Buffers, renal and respiratory compensation

Indicate the pH, HCO3- (bicarbonate), and PCO2 levels when renal compensation occurs:

pH: increased HCO3- : increased PCO2: unchanged

Gastric motility is inhibited by __________.

pain and glucose-dependent insulinotropic peptide (GIP)

Organ that adds secretions to the duodenum via a duct ??

pancreas

In the digestive system, HCl is released by ________, whereas HCO3- is secreted primarily from the ________

parietal cells of the stomach, pancreas

podocytes

part of the epithelium of Bowman's capsule. long cytoplasmic extensions called foot processes that wrap around glomerular capillaries and create filtration slits closed by a semiporous membrane.

Describe the juxtaglomerular apparatus. (Fig. 19.1g)

part where the final part of the ascending limb of the loop of henle passes between the afferent and efferent arterioles proximity of ascending limb and arterioles allows paracrine communication allowing for kidney autoregulation

The movement of water across the late distal tubule and collecting duct is __________.

passive, through channels called aquaporins, and driven by the medullary osmotic gradient

Urea is ..

passively reabsorbed in the proximal tubule.

List three solutes secreted into the tubule lumen.

penicillin, K+, and H+

Chief cells secrete ______

pepsinogen

Movements that propel the luminal contents forward into the GI tract include __________.

peristalsis and swallowing

Peristalsis differs from segmentation in that ________.

peristalsis promotes net forward movement of the bolus

Tubular reabsorption __________.

permits reclamation of filtered items from the plasma, occurs in the loop of Henle, and is responsible for the movement of amino acids, CA2+, Cl- and water at the proximal convoluted tubule

Which chemical can stimulate ventilation by binding both peripheral and central chemoreceptors?

plasma CO2

The urine of an individual will contain glucose when __________.

plasma glucose concentration has reached the renal threshold

The renal corpuscle is the site where __________.

plasma is filtered from the glomerulus into Bowman's capsule

The most potent stimulus for vasopressin release is ______

plasma osmolarity

People suffering from central diabetes insipidus (a disorder of the hypothalamus or pituitary gland) may have increased __________.

plasma osmolarity and urine volume

Filtrate entering proximal tubule has a [Na+] similar to that of ______________, which is ______________ (higher or lower?) than the [Na+] inside tubular cells.

plasma; higher

In hemostasis, vasoconstriction is rapidly followed by mechanical blockage of the hole by a _______.

platelet plug

Which unique epithelial cells of Bowman's capsule are involved in filtration?

podocytes

Which of the following structures is the site of release for vasopressin?

posterior pituitary

Change in homeostasis of ________ has a bigger impact on acid-base balance than the other options presented in this question

potassium ions

A vasopressin receptor antagonist would __________.

prevent membrane recycling in collecting duct cells

A vasopressin receptor antagonist would __________.

prevent membrane recycling in collecting duct cells This effect of vasopressin would be blocked; therefore, blood pressure would be lowered.

The primary role of the carbonic acid-bicarbonate buffer system is to

prevent pH changes caused by organic and fixed acids.

H+ is low in these 3 acid-base imbalances:

prolonged vomiting, antacid overdose, hyperventilation

Peristalsis differs from segmentation in that peristalsis __________.

promotes net forward movement of the bolus

Aldosterone _______

promotes sodium retention in the kidneys

Mucus functions in _____

protection and lubrication

Which substance would be found in higher concentration if the membrane were damaged?

protein

Which substance would be found in higher concentration if the membrane were damaged?

protein Yes, large proteins are not normally filtered by a healthy glomerular membrane.

In terms of chemical digestion of organic molecules, the stomach primarily digests ________

proteins

Most reabsorption of fluid from the filtrate back into the blood occurs from the ______.

proximal convoluted tubule

Most reabsorption of fluid from the filtrate back into the blood occurs from the __________.

proximal tubule

majority of chloride, sodium, amino acids and lactate are reabsorbed in the

proximal tubule

Compare and contrast the terms: d) water reabsorption in proximal tubule, distal tubule, and ascending limb of the loop of Henle

proximal tubule - not regulated distal nephron - regulated by vasopressin ascending limb - impermeable to water

Location of a smooth muscle band that prevents premature emptying of the stomach ??

pylorus

Glycolysis leads to the production of ____________ and two molecules of ATP. In the absence of oxygen, fermentation leads to the production of ______________. Glycolysis plus the citric acid cycle can convert the carbons of glucose to _________ , storing the energy as ATP, _____________ and ___________.

pyruvate, lactic acid, CO2, NADH, FADH2

At the end of glycolysis, each molecule of glucose has yielded 2 molecules of _______, 2 molecules of ________, and a net of 2 molecules of _________.

pyruvate; NADH; ATP

Define clearance, give its units, and write out the general equation for calculating clearance.

rate at which a substance disappears from the body through excretion or metabolism clearance of X = excretion rate of X (mg/min)/[X] plasma (mg/mL plasma)

If solute C moves passively across the basolateral membrane and is actively transported into the epithelial cell across the apical membrane, then C is (reabsorbed / secreted) across this epithelium.

reabsorbed

In the kidney, potassium may be __________.

reabsorbed secreted filtered, reabsorbed and excreted excreted filtered all answers are correct

What happens to the fluid that doesn't leave in the urine?

reabsorbed into the blood

What is the term for the movement from the nephron lumen to the blood?

reabsorption

The primary function of the proximal tubule is ...

reabsorption of ions, organic molecules, and water.

The primary function of the proximal tubule is ________

reabsorption of ions, organic molecules, and water.

The proximal tubule is where __________.

reabsorption of solutes and water takes place the reabsorption of the majority of ions, organic molecules, and vitamins takes place

Describe the mechanism(s) used to reabsorb filtered proteins in the proximal tubule.

receptor mediated Endocytosis in the proximal tubule Proteins, once inside cells are digested by lysosomes Amino acids then transported across the basolateral membrane and absorbed into the blood

The glucostatic theory of appetite regulation suggests that _______.

regions of the hypothalamus monitor the glucose concentration in the cerebrospinal fluid and increase activity in the feeding center when glucose levels decrease

The kidneys function to produce hormones, excrete wastes, regulate blood pressure, and maintain ion balance. Which of the following is another function of the kidneys?

regulate osmolarity

Functions of the kidneys include all but one of the following. Identify the exception.

regulation of blood protein levels

List and explain the six major kidney functions.

regulation of extracellular fluid volume (to maintain adequate blood pressure), regulation of osmolarity, maintenance of ion balance (neuron function), regulation of pH (proteins denature if pH not maintained), excretion of wastes and foreign substances (to prevent toxic effects), and production of hormones (that regulate RBC synthesis, Ca2+Ca2+ and Na+Na+ balance)

The functions of the kidneys include __________.

regulation of plasma pH maintenance of body fluid osmolarity regulation of plasma volume regulation of plasma volume, maintenance of body fluid osmolarity and regulation of plasma ion composition regulation of plasma ion composition all answers are correct ********FURTHER EXPLANATION OF THE ANSWER ****** The functions of the kidney are diverse and mandatory for the body to maintain homeostasis. The kidney partners with the lungs to regulate plasma pH. The kidneys can vary the amount of water released in the urine in order to maintain appropriate plasma volume. Plasma volume directly influences blood volume and blood pressure. By varying the solutes excreted in the urine, the kidneys maintain appropriate body fluid osmolarity and regulate the plasma ion concentrations. It is mandatory that plasma ion concentrations are normal in order for normal excitability of nerves and muscle. The kidney partners with the lungs to regulate plasma pH. There is a better answer. The kidneys can vary the amount of water released in the urine in order to maintain appropriate plasma volume. Plasma volume directly influences blood volume and blood pressure. There is a better answer. By varying the solutes excreted in the urine, the kidneys maintain appropriate body fluid osmolarity. There is a better answer. By varying the solutes excreted in the urine, the kidneys regulate the plasma ion concentrations. There is a better answer. The kidneys can vary the amount of water released in the urine in order to maintain appropriate plasma volume. Plasma volume directly influences blood volume and blood pressure. By varying the solutes excreted in the urine, the kidneys maintain appropriate body fluid osmolarity. By varying the solutes excreted in the urine, the kidneys regulate the plasma ion concentrations. There is a better answer.

The process of filtration in the kidney is most accurately described as

relatively nonspecific

Angiotensin II stimulates the __________.

release of aldosterone and release of ADH

Secretion refers to the ________

release of substances into the lumen of the gut

When filtrate flow through the renal tubule increases, the macula densa responds by

releasing vasoconstrictors

What are two systems that compensate for acid-base disturbances?

renal and respiratory The digestive system can affect acid-base balance (e.g., vomiting causes metabolic alkalosis) but does not regulate or compensate for acid-base changes.

The ________ branch off the ________ and supply blood to the kidneys.

renal arteries, abdominal aorta

Trace a drop of blood through the nephron from a renal artery to a renal vein. (Fig. 19.1d-h)

renal artery => afferent arteriole => glomerulus => efferent arteriole => peritubular capillaries => renal vein

The renal filtrate is formed in the kidney's outer shell, called the __________.

renal cortex The outer portion of any sphere-like structure is its cortex, while the inner portion is called the medulla.

The final volume and osmolarity of the urine depend on _________________.

renal handling what substances are reabsorbed, secreted, and filtered

Respiratory Acidosis/akalosis compensates through the _____

renal system

The plasma concentration at which all of the renal carriers for a substance are saturated is the

renal thershold

For a particular substance, the plasma concentration at which that substance first appears in the urine is known as the ________________________. (Figs. 19.9, 19.10)

renal threshold

The ____ carries blood from the kidneys back to the ____

renal vein, inferior vena cava

Granular cells secrete ______

renin

The RAS pathway begins with secretion of ________

renin

A person who suffers from emphysema will exhibit signs of ___________

respiratory acidosis

A patient is admitted to the hospital with the following plasma values: pH = 7.2, pCO2 = 55 mmHg, and HCO3 −− = 30 mEq/L. What is the acid base imbalance?

respiratory acidosis with metabolic compensation this is a respiratory acidosis with metabolic compensation. Since pCO2 is above the normal range, it is the cause of the acidosis. HCO3 −− is increased to compensate, which is a metabolic compensation.

Emphysema can lead to which acid/base disturbance? What would be the compensation?

respiratory acidosis; kidneys will retain more HCO3 −− and excrete H+ a patient with emphysema would retain CO2, leading to a respiratory acidosis. The kidneys would compensate by retaining HCO3 −− and excreting H+

A person who suffers from hyperventilation will exhibit signs of ________

respiratory alkalosis

Severe anxiety would cause which type of acid-base disturbance? What would be the compensation?

respiratory alkalosis; kidneys would excrete HCO3 during hyperventilation caused by severe anxiety, carbonic acid is excreted from the lungs in the form of CO2, thus causing an alkalosis. Kidneys would compensate by excreting HCO3.

metabolic acidosis compensates through the _____

respiratory system

Amylases, the enzymes used to digest carbohydrates, are secreted by _____

salivary glands into the mouth and by the pancreas into the intestine

A decrease in fluid osmolarity may result from ingesting too much water. What mechanism exists to counteract the drinking of too much water?

salt appetite

List the three characteristics of mediated transport.

saturation specificty competition

The transport process(es) utilized by the proximal tubule to reabsorb glucose include __________.

secondary active transport and facilitated diffusion

goblet cells:

secrete mucus

If solute A is actively transported across the basolateral membrane into the epithelial cell and then passively crosses the apical membrane, then is A (reabsorbed / secreted) across this epithelium.

secreted

Bile is ______________

secreted by hepatocytes only

Inhibits gastric emptying :

secretin

Which of the following GI hormones promotes a pancreatic juice rich in bicarbonate ions?

secretin secretin (from S cells in the duodenum) causes both the liver and pancreas to secrete bicarbonate into the small intestine.

An increase in HCl (hydrochloric acid) arriving in the duodenum would stimulate which hormone that would help to counteract the effects of HCl?

secretin secretin stimulates the duct cells in the pancreas and liver to secrete a bicarbonate-rich solution that will bind hydrogen ions and increase the pH.

In response to the acid in the duodenum, the blood levels of _________

secretin rise

The secretion of pancreatic juice is stimulated by __________.

secretin, cholecystokinin (CCK) and H+ ions in the duodenum

Contents in the peritubular capillaries are actively transported in the proximal and distal tubules through a process known as

secretion

Contents in the peritubular capillaries are actively transported into proximal and distal convoluted tubules in a process known as:

secretion

What is the term for the movement from the peritubular capillaries to the nephron lumen?

secretion

Which kidney process is always active and always requires energy to occur?

secretion

Which of these processes uses membrane proteins to move molecules across the tubule epithelium?

secretion

Which word means to selectively move molecules from the blood and add them to the tubule lumen?

secretion

Secretion differs from filtration in that __________.

secretion is more selective secretion is selective because it is carrier-mediated

Atrial natriuretic peptide (ANP) decreases __________.

secretion of aldosterone secretion of renin reabsorption of sodium all answers are correct

In addition to waste removal and regulation of cellular osmolarity, the kidneys' other functions may include __________.

secretion of erythropoietin and gluconeogenesis activation of vitamin D3 gluconeogenesis secretion of erythropoietin all answers are correct

Contents in the peritubular capillaries are actively transported into proximal and distal convoluted tubules in a process known as

secretion.

Glucose reabsorption from the renal filtrate is due to __________.

sequential active transport and facilitated diffusion by the cells of the proximal tubule

Metabolic acidosis can be caused by __________.

severe diarrhea and a diet high in protein and fat

Erica's baby, Justin, has had a severe bout of diarrhea and is now dehydrated. Is his blood more likely to be acidotic or alkalotic? Why?

severe diarrhea→lossdiarrhea→loss of small intestine HCO −3→HCO3 −→ metabolic acidosis

Osmoreceptors depolarize after they _____ in response to ____ plasma osmolarity.

shrink; increased

The cephalic phase of digestion can be initiated by ________.

sight and smell of food, and sounds associated with food

The cephalic phase of digestion can be initiated by the __________.

sight and smell of food, and sounds associated with food preparation

Given the chemical nature of free fatty acids and monoglycerides, how are these absorbed across the epithelial border of the small intestine?

simple diffusion

Slow, spontaneous graded depolarizations that occur in the circular layer of smooth muscle in the GI tract are called __________.

slow waves

Kidneys respond relatively ______ to changes in blood volume

slowly

Carbohydrate digestion is completed here:

small intestine

Digestion is essentially completed in the ________

small intestine

Fat digestion is completed here:

small intestine

In the body, 80% of all lymphocytes, a type of immune system cell, are thought to be present in the ______

small intestine

Nutrient absorption occurs primarily in the _______

small intestine

Organ where most digestion occurs?

small intestine

Protein digestion is completed here:

small intestine

Which of the following molecules are absorbed by the intestinal epithelia by active transport?

sodium and calcium

SGLT

sodium glucose cotransporter on apical membrane

How is glucose moved over the basolateral membrane

sodium potassium pump allows glucose to diffues through facilitated diffusion GLUT transporter

If someone eats a large quantity of salty potato chips, to maintain homeostasis the kidneys may excrete urine that is higher in __________.

solute concentration and osmolarity

Once a GFR is known, what two factors must be measured to allow you to analyze renal handling of a substance? How would you use these measurements to determine renal handling? (Fig. 19.13)

solutes plasma concentration and excretion rate

During micturition __________.

somatic motor neuron input to the external urethral sphincter is inhibited and stretch receptor activity leads to smooth muscle contraction

What cellular compartment becomes acidic (high concentration of hydrogen ions) during mitochondrial electron transport?

space between inner and outer mitochondrial membranes

The hormone vasopressin

stimulates the kidneys to conserve water.

An example of a feedforward endocrine reflex in the digestive system is the __________.

stimulation of insulin release by GIP, preventing a rise in plasma glucose

Fat digestion begins here:

stomach

Protein digestion begins here:

stomach

chyme is released from here ??

stomach

If blood flow through the afferent arterioles increases, __________ ?

stretch reflexes trigger vasoconstriction to reduce the flow

The neuronal networks of the enteric nervous system are found within the __________.

submucosa and muscularis externa

Below saturation, the rate of transport is proportional to __________________. (Fig. 19.9)

substrate concentration

Two hours before major surgery is to begin, the patient experiences "jitters," an elevated heart rate and blood pressure. These symptoms are the result of

sympathetic activation

Neural control of GFR is mediated by ________ that innervate ________ receptors on vascular smooth muscle causing ________

sympathetic neurons, alpha, vasoconstriction

Decreased ECF volume causes ___________

sympathetic output from the cardiovascular control center to increase

Norepinephrine is the neurotransmitter released by which fibers?

sympathetic postganglionic fibers norepinephrine is the neurotransmitter of the sympathetic postganglionic fibers. An increase in the sympathetic system would decrease digestion

Match each substance with its primary mode of transport across the kidney epithelium. A. transcytosis B. active transport C. symport with a cation D. passive reabsorption/diffusion - glucose

symport with a cation

Glucose and amino acids are reabsorbed by ..

symport with sodium.

Micturition

the action of urinating.

In which of the following locations is the osmolarity of the filtrate the greatest?

the base of the loop of Henle The osmolarity decreases as the fluid moves up the ascending limb as ions are "pumped" into the interstitial space, thus reducing the osmolarity inside the ascending limb.

Obesity can lead to diabetes because __________.

the cells of an obsese person become resistant to insulin and fat cells secrete hormones that lead to insulin resistance

A drug that blocks the action of the hormone cholecystokinin would affect ___________

the composition of pancreatic secretions

In order to measure the clearance of a substance, one would need to know which of the following?

the concentration of the substance in the plasma, the concentration of the substance in the urine, and the urine flow rate

By the end of the proximal tubule in a normal kidney, __________.

the filtrate glucose concentration = 0 the filtrate is iso-osmotic to plasma the filtrate volume is reduced by 70% all answers are correct

The renal corpuscle consists of two parts, the glomerular capillaries and __________.

the glomerular capsule

What forces cause fluid reabsorbed from the kidney tubule to move into the peritubular capillaries? (Fig. 19.11)

the hydrostatic pressure of the peritubular capillaries is less than the colloid osmotic pressure, so the net pressure gradient favors reabsorption

The nephron is the functional unit of the kidney because __________.

the nephron filters the plasma, reabsorbs critical molecules, and excretes the rest This is essentially what the kidney does; therefore, the nephron is the functional unit of the kidney.

The proximal and distal tubules differ in that __________.

the proximal tubule is "leakier" than the distal tubule, distal tubule cells possess a higher concentration of hormone receptors compared to proximal tubule cells, and proximal tubule cells have larger numbers of mitochondria

Which of the following statements best describes clearance?

the rate at which a solute disappears from the plasma by either excretion or metabolism

retroperitoneal

the space between the peritoneum and the posterior abdominal wall that contains especially the kidneys and associated structures, the pancreas, and part of the aorta and inferior vena cava.

An example of a feedforward endocrine reflex in the digestive system is ________.

the stimulation of insulin release by GIP, preventing a rise in plasma glucose

Which of the following events does NOT occur when you swallow?

the upper esophageal sphincter closes

Urine is carried to the urinary bladder by

the ureters

Urine is carried to the urinary bladder by____

the ureters

During the cephalic phase of gastric secretion, _________

there is an increased flow of action potentials along the vagus nerve to the stomach.

Sally goes to the emergency room (ER) seeking treatment for mild dehydration. The ER nurse mistakenly gives Sally an IV bag of solution that is very hypotonic to normal cells. Which of the following statements about the release of vasopressin reflects what would occur as a result of this mistake?

there is decreased release of vasopressin

Angiotensin II stimulates _______

thirst, vasoconstriction, and synthesis and release of aldosterone

Intrinsic control of glomerular filtration rate (GFR) can occur __________.

through myogenic regulation and via tubuloglomerular feedback

GI contractions that are sustained for minutes or hours are called ________ contractions

tonic

The rate of transport at saturation is also known as the ___________________. (Fig. 19.9)

transport maximum

Glucose reabsorption utilizes transport proteins in the proximal convoluted tubule cells to move glucose into the blood. The maximum amount of glucose that can be transported when all of the transport proteins are full is known as the ___________. Any remaining glucose will be excreted in the urine.

transport maximum The transport maximum is determined by the number of transport proteins available to move glucose into the blood.

In the myogenic response, the macula densa cells send a paracrine message to the neighboring afferiole

true

In tubuloglomerular feedback, stretch-sensitive ion channels open, resulting in depolarization of smooth muscle cells

true

Myogenic response is a paracrine signaling mechanism

true

The (tubular epithelial cell / peritubular capillary endothelial cell) forms the primary barrier for reabsorption of most solutes.

tubular epithelial cell

One substance has no membrane transporters to move it but can diffuse freely through open leak channels if there is a concentration gradient. Initially, this substance's concentrations in the filtrate and extracellular fluid are equal. Later, however, the active transport of Na+ and other solutes creates a gradient by removing water from the lumen of the tubule where it is located. What substance is this?

urea

Urine is carried out of the body by the

urethra

Urine is carried to the external environment by the ____

urethra

The primary route for ion loss from the body is the ______ system

urinary

The primary route for water loss from the body is ____ system.

urinary

Which system helps control pH by eliminating H+ from the body?

urinary

The two organ systems that work together to regulate most aspects of the body's water balance are

urinary and cardiovascular

After it is formed, urine is temporarily stored in the ____

urinary bladder

What is micturition?

urination

The characteristic yellow color of urine is attributed to the presence of

urobilinogen

When filtrate flow through the renal tubule increases, the afferent arteriole responds by

vasoconstricting

Function of Angiotensin II

vasoconstriction, stimulates thirst, stimulate production and release of aldosterone from the adrenal cortex

Lower than normal NaCl concentration in the ascending limb of the nephron loop will cause the afferent arterioles to __________.

vasodilate

The hormone that directly controls water reabsorption by the kidneys is

vasopressin

Most cell types are ________ permeable to water. An exception is cells of the ________, which can be either impermeable or permeable, depending upon the absence or presence of ________

very; distal nephron; aquaporins

Microbes in the colon can synthesize __________, which is (are) absorbed luminally via __________.

vitamin K; simple diffusion

Microbes in the colon can synthesize __________, which is/are absorbed luminally via __________.

vitamin K; simple diffusion

Substances secreted into the filtrate as it moves along the nephron tubule include all of the following EXCEPT __________.

water

Functions associated with the loop of Henle include __________.

water conservation and creation of an osmotic gradient

Kidneys regulate

water loss (not water gain)

The development of hyperthermia can be counteracted by _______.

wetting the skin

Which type of blood lipoprotein has been implicated in the formation of atherosclerotic plaques

which can cause coronary artery disease?,LDL

When a body is dehydrated, water in the urinary bladder

will still be expelled from the body in the urine

When the body is dehydrated, water in the urinary bladder

will still be expelled from the body in the urine

Does looking at the composition of urine tell us if a substance has been filtered? _________ Reabsorbed? ______________ Secreted? ______________ Excreted? ______________

yes to secreted and excreted, check on others

Mrs. F has spider nevi, broken skin capillaries. Normally, broken skin capillaries do not cause visible blemishes, but hers are bleeding more than usual. Could this be due to her liver problems?

~The liver makes bile, which is needed to absorb lipid-soluble vitamins. ~The liver creates many plasma proteins. The liver is vital for proper blood clotting. First, the liver synthesizes plasma proteins, including many clotting factors. Second, if the liver fails to produce bile, fat and fat-soluble vitamins cannot be absorbed effectively from the diet. Vitamin K is an essential clotting factor. And there is a third reason...


Set pelajaran terkait

Biology B2: Organisms in the environment

View Set

النظام الاقتصادي في الإسلام

View Set

Commercial Property Insurance Practice Questions

View Set

Mastering Biology Practice Questions Chapters 40, 45, 46, 47, 48, 49, 54

View Set

The patient with endocrine health problems

View Set

Business Ethics Final ND - Class 1-3

View Set